Sie sind auf Seite 1von 254

WNM

Tax 1 Ring (Fall 2017)

THE TAX SYSTEM........................................................................................................................................................................2


GROSS INCOME...........................................................................................................................................................................6
Benefits in Kind..............................................................................................................................................................................6
In General...................................................................................................................................................................................6
Fringe Benefits............................................................................................................................................................................8
Imputed Income.........................................................................................................................................................................11
Recovery of Capital.......................................................................................................................................................................13
Introduction to Basis..................................................................................................................................................................13
Damage Payments.....................................................................................................................................................................14
Tax Benefit Doctrine..................................................................................................................................................................19
Annuities...................................................................................................................................................................................20
Life Insurance...........................................................................................................................................................................22
Realization....................................................................................................................................................................................25
The Realization Requirement......................................................................................................................................................25
Benefit of Deferral.....................................................................................................................................................................26
Discount Obligations.....................................................................................................................................................................28
Capital Gains............................................................................................................................................................................32
Factors of Motivation....................................................................................................................................................................34
Windfalls...................................................................................................................................................................................34
Gifts......................................................................................................................................................................................... 34
Prizes & Awards........................................................................................................................................................................41
Scholarships & Fellowships........................................................................................................................................................42
Social Welfare Payments.............................................................................................................................................................43
Relationship of Categories..........................................................................................................................................................43
Offsetting Liabilities.....................................................................................................................................................................44
Cancellations of Indebtedness.....................................................................................................................................................44
Claim of Right & Subsequent Repayment..................................................................................................................................... 46
Embezzled Funds.......................................................................................................................................................................47
Tax Expenditures & Concept of Income........................................................................................................................................49
Tax Exempt Bonds......................................................................................................................................................................49
Tax Expenditure Budget..............................................................................................................................................................50
Concept of Income.....................................................................................................................................................................50
Alternative Tax Base..................................................................................................................................................................51
DEDUCTIONS.............................................................................................................................................................................52
Deductions....................................................................................................................................................................................52
Profit Seeking Expenditures..........................................................................................................................................................53
Ordinary and Necessary Business Expense...................................................................................................................................53
Capital Expenditures..................................................................................................................................................................56
Capital Recovery....................................................................................................................................................................... 58
Losses..................................................................................................................................................................................59
Bad Debts............................................................................................................................................................................61
Depreciation.........................................................................................................................................................................61
Business/Personal Borderline........................................................................................................................................................63
Childcare..................................................................................................................................................................................63
Clothing....................................................................................................................................................................................64
Travel & Moving Expenses......................................................................................................................................................... 65
Travel..................................................................................................................................................................................65
Moving Expenses..................................................................................................................................................................66
Travel & Entertainment..............................................................................................................................................................66
Hobby Losses............................................................................................................................................................................67
Legal Expenses..........................................................................................................................................................................68
Education Expenses................................................................................................................................................................... 68
Approaches...............................................................................................................................................................................69
Personal Deductions......................................................................................................................................................................69
Interest.....................................................................................................................................................................................69
Taxes........................................................................................................................................................................................70
Casualty Losses.........................................................................................................................................................................70
Medical Expenses...................................................................................................................................................................... 72
Charitable Contributions............................................................................................................................................................72
COMPUTATION OF TAX............................................................................................................................................................75
Tax Rates......................................................................................................................................................................................75
Leverage.......................................................................................................................................................................................77
Nonrecourse Borrowing.............................................................................................................................................................77
CHOICE OF TAXPAYER.............................................................................................................................................................80
Capital Gains & Losses.................................................................................................................................................................81

1
WNM
Tax 1 Ring (Fall 2017)

THE TAX SYSTEM


Policy Objectives of the Tax System:
0 Raise revenue for government functions
1 Supplement fiscal policy objectives: stimulate the economy by increasing or decreasing
taxes
2 Affect income and wealth distribution: reduce income disparities (Medicare, welfare,
Medicaid)
3 Provide financial assistance to achieve social goals
4 Achieve economic goals: encourage investments in particular economic activities (i.e.
investments in clean energy technology)
5 Regulation: imposing taxes on specific activities deemed undesirable or harmful (i.e.
pollution)
Sources of Tax Law:
0 Internal Revenue Code (IRC): statute regulating federal taxation
0.0 Interpretation: Courts look to the underlying intent of Congress with respect to
the IRC as a whole and try to determine which interpretation is the most
consistent with objectives of the IRC as a whole.
0.1 §1 of IRC governs taxes imposed upon individuals
0.2 §11 of IRC governs taxes imposed upon corporations
1 Treasury Regulation:
1.0 Treasury department adopts regulations that concern the income tax; the
Internal Revenue Service (branch of Treasury Dep’t) usually writes the first
draft of the regulation. It is then approved by Congress.
1.1 Courts tend to give a lot of deference to the agency’s interpretation
1.0 Chevron – established that courts allowed to interpret the Tax Code
1.1 Add detail to the statutory scheme
1.2 Two Types of Regulation:
2.0 Interpretive Regulation → Under §7805(a) Treasury Dept. has
authority to issue regs. that interpret the IRC. These interpretive regs.
are given significant deference by the courts - so long as
interpretations are reasonable because the Treasury Dep’t. is the
expert.
2.1 Legislative Regulation → In the IRC, Congress authorized the
Secretary of the Treasury to promulgate tax regulations. Therefore,
Court give the IRS significant deference – so long as it does not act
outside the scope of its authority
1.0 Ex] totally arbitrary and capricious interpretation
1.1 treated as a statutory adoption
2.2 Legislative History
2.0 Congressional Reports: tax bills begin in the House (House
Ways & Means Committee issues a report). The bill then goes
to the Senate (Senate Financial Committee issues a report)
2.1 If the House and Senate issue two conflicting proposals then
Conference Committee (members of House & Senate) issue a
compromising report
2 Courts: Court can look to the look to the Economic Substance to determine what’s
going on – unique to U.S.
2.0 3 Court → 3 Outcomes:
0.0 U.S. District Court:
0.0 π must pay the tax and then sue for a refund
0.1 Jury trial (sympathetic facts → favorable result?)
0.2 Applies law of and appeals to the circuit court in which the
district your’e in
0.1 Tax Court
1.0 π does not pay the tax; stay is put in place to prevent the IRS
from attaching assets
1.1 Results in a bench trial (can handle more complicated matters?)
1.2 Applies law of and appealed to the circuit court in which the
taxpayer (TP) resides
1.3 Experts in tax
0.2 Federal Claims Courts
2
WNM
Tax 1 Ring (Fall 2017)

0 TP must pay the tax and then sue for a refund


1 must pay tax then sue for refund, bench trial,
2 Court applies law of & appealed to federal circuit of appeals
0 Effect on Penalties
0 It’s important when advising a client to consider which court they will
bring their case in because failure to pay a tax can result in a 20%
penalty UNLESS the TP is able to show that there was substantial
authority for the position taken (§6662(d)(2)(B))
0 ex: if 2 court cases - 1 that supports and 1 that doesn’t; can point
to the 1 that supports and use that as substantial authority so
long as reasonable reasoning (not totally laughable)
0 usually regarded as about 40% chance of success.
1 failure to advise client re: potential applicability of
penalties is grounds for malpractice
2 Negligence – Standard: realistic possibility of success on
the merits
– 33%
3 Substantial underpayment – $5k or 10% deficiency – 40%
4 Tax Shelters – Standard: success more likely than not –
50%+
5 Lawyers / Preparers - §6694 – Fine of $1k or 50% of fee –
penalties
where TP doesn’t even have them – Congress
reaffirming role as gatekeepers
IRS Rulings
Revenue Ruling/Published Rulings → memorandum of law prepared by IRS
attorney and issued by the treasury dept; treasury dept is estopped from
denying the validity of the revenue ruling so long as the law upon which
the ruling is based on hasn’t changed.
Private Rulings → TP can ask IRS for a private ruling about what it thinks
the treatment should be; not used often b/c can take 6 months; IRS
will only honor that ruling to the TP to whom that ruling was issued;
attorneys should still look at these not b/c client can rely on them but
b/c gives good idea as to what the appropriate solution is; courts can’t
cite these as precedent.
0 Process & Litigation
0 File tax return → Not filing a Tax Return (TR) is a criminal offense (with no SOL).
0 under §6012 every individual who has gross income in excess of a certain level
must file a tax return
1 Audit/Inquiry → some returns are audited – 1-2% - (especially if in cash flow
business - ex: sole proprietor, run small business, etc) or inquiry if W2s & 1099s that
IRS receive don’t match w/ tax return; IRS may just ask you to make correction or
may come to your home or place of business (in
that case want to see your standard of living - might be claim of criminal conduct)
0 Amended Return → if mistake or client has lied they should file an amended return
before IRS files criminal complaint b/c then harder for IRS to prove beyond a
reasonable doubt that TP intended to defraud the gov’t (oftentimes IRS won’t press
criminal charges)
1 Administrative Appeals Process → after audit, IRS issues report called Revenue Agent
Report (RAR) stating that TP owes additional amounts of money & issue a 30 day letter;
TP must either pay amount or w/i 30 days file a formal document w/ the IRS appeals office
(which states facts & law).
1.0 During hearing, the appeals officer can settle the case based on statistical
outcome of case - ex: if TP has 60% chance of winning, will reduce tax owed
by 60%; but if TP has less than 20% chance IRS won’t settle.
1.1 Most disputes w/ IRS are resolved in the appeals office
2 File complaint → if don’t file w/ appeals office w/i 30 days and don’t pay tax, TP
receives a 90 day letter - a formal statutory notice of deficiency - have 90 days to pay
tax or IRS will take the tax (ie: seize property); TP can either pay tax or file petition in
one of the 3 courts
2.0 after file complaint, TP has opportunity to meet w/ appeals office or
opposing attorney to negotiate claim, again based on statistical outcome.
3 Statute of Limitations → normally 3 years from due date of tax return; but 6 years if it
turns out client has a substantial underpayment (25% of tax due); and if lied (ie:
fraud) then no statute of limitations (can even go after TP’s estate after death)
3
WNM
Tax 1 Ring (Fall 2017)

ĀȀĀᜀĀᜀĀᜀĀᜀĀᜀĀᜀĀĀȀ⸀ĀᜀĀᜀĀᜀĀᜀĀᜀĀᜀĀᜀĀᜀĀĀĀĀ0 Burden of Proof


Ȁ
ĀЀĀĀȀĀȀ⠀⠀⤀ĀЀĀĀЀĀᜀĀᜀĀᜀĀᜀЀĀȀ⸀ĀᜀĀ⨀ĀЀĀȀȀ⸀ĀĀĀȀऀऀЀĀȀЀ⸀ĀᜀĀᜀĀᜀĀᜀĀᜀĀ㄀Āᜀ
Ā∀ĠᜀĀᜀĀᜀĀᜀĀᜀĀᜀĀᜀ‫؀‬ĀȀ⸀ĀЀĀȀĀ⸀ऀऀЀĀȀȀ⸀ĀĀȀȀ⸀ĀЀĀȀЀ⸀ĀȀĀȀԀ⸀ĀᜀĀᜀĀᜀĀᜀĀ㌀ĀᜀĀ∀Ġ
ᜀĀᜀĀᜀĀᜀĀᜀĀᜀĀᜀĀĀȀ⸀ĀЀĀȀĀ⸀ĀȀĀȀȀ⸀ĀĀȀȀ⸀ĀᜀĀᜀĀᜀĀᜀĀᜀĀᜀĀ㐀ĀᜀĀ∀ĠᜀĀᜀĀᜀĀᜀĀ
ᜀĀᜀĀᜀĀᜀĀ㔀ĀᜀĀᜀĀᜀĀᜀĀᜀĀᜀĀᜀĀᜀĀᜀĀ∀ĠᜀĀᜀĀᜀĀᜀĀᜀĀᜀĀᜀĀᜀༀЀĀĀĀᜀĀᜀĀᜀĀᜀĀᜀĀ
ᜀĀᜀĀᜀĀᜀĀ∀ĠᜀĀᜀĀᜀĀᜀĀᜀĀᜀĀᜀĀᜀༀЀĀĀĀᜀĀᜀĀᜀĀᜀĀᜀĀᜀĀᜀĀᜀༀЀĀĀĀᜀĀᜀĀᜀĀᜀĀᜀĀ
ᜀĀᜀĀᜀĀᜀĀ㘀ĀᜀĀᜀĀᜀĀᜀĀᜀĀᜀĀᜀĀᜀ㄀ ĀĀĀᜀĀᜀĀᜀĀᜀĀᜀĀᜀĀᜀĀᜀĀĀȀ⸀ĀЀĀȀĀ⸀ĀᜀĀᜀĀ
ᜀĀᜀĀᜀĀᜀĀᜀఀ㄀ ĀĀĀᜀĀ∀ĠᜀĀᜀĀᜀĀᜀĀᜀĀᜀĀᜀĀĀĀЀЀĀȀĀ⸀ĀЀĀĀȀĀĀĀȀĀᜀĀᜀĀᜀĀᜀĀᜀఀȀĀȀ⸀ĀЀĀȀĀ⸀ĀЀĀĀȀĀ
ᜀĀᜀĀᜀĀᜀĀᜀĀЀĀĀ⌀ЀĀȀĀ⸀ĀᜀĀᜀĀᜀĀᜀĀᜀĀᜀĀᜀఀȀЀĀȀ⸀ऀऀЀĀȀĀ⸀ĀᜀĀᜀĀᜀĀᜀĀᜀĀᜀĀᜀ㄀ ĀĀĀ
ᜀĀ∀ĠᜀĀᜀĀᜀĀᜀĀᜀĀᜀĀᜀĀᜀĀ∀༠ЀĀĀĀĀᜀĀᜀĀᜀĀᜀĀᜀĀᜀĀᜀĀĀȀ⸀ĀЀĀȀĀ⸀ĀȀĀȀȀ⸀ĀĀȀȀ⸀Ā
ᜀĀᜀĀᜀĀᜀĀᜀĀĀĀĀЀĀĀĀ⌀ЀĀȀȀ⸀ĀĀȀȀ⸀ĀȀȀȀ⠀Ѐ⤀ĀᜀĀᜀĀᜀĀᜀఀȀĀȀ⸀ĀЀĀȀĀ⸀ऀऀЀĀȀȀ⸀ĀĀȀȀ⸀ĀȀĀĀЀĀ
ᜀĀᜀĀᜀĀᜀༀĀĀĀᜀĀᜀĀᜀĀᜀĀᜀĀᜀĀᜀĀᜀĀĀȀ⸀ĀЀĀȀĀ⸀ĀȀĀȀȀ⸀ĀĀȀȀ⸀ĀᜀĀᜀĀᜀĀᜀĀᜀሀĀĀĀᜀ
ĀᜀĀᜀĀᜀĀᜀĀᜀĀᜀĀᜀĀᜀĀ∀ĠᜀĀᜀĀᜀĀᜀĀᜀĀᜀĀᜀĀᜀༀЀĀĀĀᜀĀᜀĀᜀĀᜀĀᜀĀᜀĀᜀĀᜀĀᜀĀ꜀ĀᜀĀ
ᜀĀᜀĀᜀĀᜀĀᜀĀᜀĀᜀĀᜀĀ∀ĠᜀĀᜀĀᜀĀᜀĀᜀĀᜀĀᜀĀᜀༀЀĀĀĀᜀĀᜀĀᜀĀᜀĀᜀĀᜀĀᜀĀᜀఀȀĀȀ⸀ĀЀĀȀĀ⸀ĀȀĀȀȀ⸀ĀĀȀȀ⸀Ā
ᜀĀᜀĀᜀĀᜀĀᜀጀĀĀĀᜀĀᜀĀᜀĀᜀĀᜀĀᜀĀᜀĀᜀĀᜀĀ∀ĠᜀĀᜀĀᜀĀᜀĀᜀĀᜀĀᜀĀᜀ᐀ĀĀĀᜀĀ∀ĠᜀĀᜀĀ
ᜀĀᜀĀᜀĀᜀĀᜀༀЀĀĀĀᜀĀᜀĀᜀĀᜀĀᜀĀᜀĀᜀĀᜀĀĀĀȀЀĀȀĀ⸀ĀȀĀȀȀ⸀ĀĀȀȀ⸀ĀЀĀȀЀ⸀ĀȀĀĀԀĀ
ᜀĀᜀĀᜀ‫؀‬ĀȀ⸀ĀЀĀȀĀ⸀ĀȀĀȀȀ⸀ĀĀȀȀ⸀ĀЀĀȀЀ⸀ऀऀЀĀȀԀ⸀ĀᜀĀᜀĀᜀĀЀĀȀ⸀ĀᜀĀᜀĀᜀĀᜀĀᜀĀᜀĀᜀ
ĀᜀᔀĀĀĀᜀĀ꜀ĀᜀĀᜀĀᜀĀᜀĀᜀĀᜀĀᜀĀȀĀȀ⸀ĀĀȀĀ⸀ĀЀĀȀȀ⸀ĀȀĀȀȀ⸀ĀĀȀЀ⸀ĀᜀĀᜀĀᜀĀᜀᜀĀĀĀ
ᜀĀ∀ĠᜀĀᜀĀᜀĀᜀĀᜀĀᜀĀᜀ㄀ĀȀ⸀ĀЀĀȀĀ⸀ĀȀĀȀȀ⸀ĀĀȀȀ⸀ĀЀĀȀЀ⸀ĀᜀĀᜀĀᜀĀᜀЀЀĀȀ⸀ĀȀĀȀĀ⸀ĀĀȀȀ⸀Ā
ᜀĀᜀĀᜀĀᜀĀᜀĀᜀ᠀ĀĀĀᜀĀᜀĀᜀĀᜀĀᜀĀᜀĀᜀĀᜀĀĀȀ⸀ĀЀĀȀĀ⸀ĀȀĀȀȀ⸀ĀᜀĀᜀĀᜀĀᜀĀᜀĀᜀᤀĀĀĀ
ᜀĀ∀ĠᜀĀᜀĀᜀĀᜀĀᜀĀᜀĀᜀĀĀȀ⸀ĀЀĀȀĀ⸀ĀЀĀĀȀĀᜀĀᜀĀᜀĀᜀĀᜀĀᜀԀĀȀ⸀ĀЀĀȀĀ⸀ĀᜀĀᜀĀᜀĀ
ᜀĀᜀĀᜀĀᜀఀ㄀ ĀĀĀᜀĀ케ĥᜀĀ쬀ĥᜀĀꀀĥᜀĀᜀĀᜀĀᜀĀᜀĀĀĀĀЀĀĀĀĀȀĀĀȀЀĀȀȀ⸀ĀᜀĀᜀĀᜀĀᜀĀ
ᜀĀĀĀĀЀĀĀĀȀȀĀȀȀ⸀ĀĀĀȀĀᜀĀᜀĀᜀĀᜀĀᜀ‫܀‬ĀȀ⸀ĀЀĀȀĀ⸀ĀȀĀȀȀ⸀ĀĀĀȀĀᜀĀᜀĀᜀĀᜀĀᜀȀĀȀ⸀Ā
ᜀĀᜀĀᜀĀᜀĀᜀĀᜀĀᜀĀᜀ㄀ ĀĀĀᜀĀᜀĀᜀĀᜀĀᜀĀᜀĀᜀĀᜀ㄀ĀȀ⸀ĀЀĀȀĀ⸀ĀȀĀȀȀ⸀ĀᜀĀᜀĀᜀĀᜀĀᜀ
ĀᜀĀĀĀȀЀĀȀĀ⸀ĀЀĀĀȀĀĀĀȀĀЀĀĀЀĀᜀĀᜀĀᜀĀᜀఀȀĀȀ⸀ĀЀĀȀĀ⸀ऀऀЀĀȀȀ⸀ĀĀȀȀ⸀ĀЀĀȀЀ⸀ĀᜀĀ
ᜀĀᜀĀᜀȀȀĀȀ⸀ĀᜀĀᜀĀᜀĀᜀĀᜀĀᜀĀᜀĀᜀἀĀĀĀᜀĀᜀĀᜀĀᜀĀᜀĀᜀĀᜀĀᜀЀĀȀ⸀ĀЀĀȀĀ⸀ĀȀĀȀȀ⸀ĀĀȀȀ⸀Ā
ᜀĀᜀĀᜀĀᜀĀᜀĀĀȀ⸀ĀЀĀȀĀ⸀ĀȀĀȀȀ⸀ĀĀȀȀ⸀ĀᜀĀᜀĀᜀĀᜀĀᜀ℀ĀĀĀᜀĀᜀĀᜀĀᜀĀᜀĀᜀĀᜀĀᜀĀĀĀЀЀĀȀĀ⸀ĀȀĀȀȀ⸀ĀĀ
ᜀĀᜀĀᜀĀᜀఀȀĀȀ⸀ĀЀĀȀĀ⸀ĀȀĀȀȀ⸀ĀĀĀȀĀЀĀĀЀĀᜀĀᜀĀᜀĀᜀ∀ĀĀĀᜀĀᜀĀᜀĀᜀĀᜀĀᜀĀᜀĀᜀĀĀȀ⸀ĀЀĀȀĀ⸀ĀĀȀȀ⸀Ā
ᜀĀᜀĀᜀĀᜀĀᜀĀᜀĀЀĀĀЀȀĀȀĀ⸀ĀᜀĀᜀĀᜀĀᜀĀᜀĀᜀĀᜀఀȀЀĀȀ⸀ĀȀĀȀĀ⸀ĀᜀĀᜀĀᜀĀᜀĀᜀĀᜀĀᜀ⌀
ĀĀĀᜀĀ∀ĠᜀĀᜀĀᜀĀᜀĀᜀĀᜀĀᜀༀЀĀĀĀᜀĀ꜀ĀᜀĀᜀĀᜀĀᜀĀᜀĀᜀĀᜀĀᜀĀ∀༠ЀĀĀĀĀᜀĀᜀĀᜀĀᜀĀᜀ
ĀᜀĀᜀĀᜀĀ∀༠ЀĀĀĀĀᜀĀᜀĀᜀĀᜀĀᜀĀᜀĀᜀ─ĀĀĀᜀĀᜀĀᜀĀᜀĀᜀĀᜀĀᜀĀᜀЀЀĀȀ⸀ĀȀĀȀĀ⸀ĀᜀĀ
ᜀĀᜀ0 criminal fraud → beyond reasonable doubt (gov’t has burden)
ĀЀĀĀȀĀȀȀ⠀⠀⤀ĀЀĀĀЀĀᜀĀᜀĀᜀĀᜀЀĀȀ⸀ĀᜀĀ⨀ĀЀĀȀȀ⸀ĀĀĀȀऀऀЀĀȀЀ⸀ĀᜀĀᜀĀᜀĀᜀĀᜀĀ㄀
ĀᜀĀ∀ĠᜀĀᜀĀᜀĀᜀĀᜀĀᜀĀᜀ‫؀‬ĀȀ⸀ĀЀĀȀĀ⸀ऀऀЀĀȀȀ⸀ĀĀȀȀ⸀ĀЀĀȀЀ⸀ĀȀĀȀԀ⸀ĀᜀĀᜀĀᜀĀᜀĀ㌀
ĀᜀĀ∀ĠᜀĀᜀĀᜀĀᜀĀᜀĀᜀĀᜀĀĀȀ⸀ĀЀĀȀĀ⸀ĀȀĀȀȀ⸀ĀĀȀȀ⸀ĀᜀĀᜀĀᜀĀᜀĀᜀĀᜀĀ㐀ĀᜀĀ∀ĠᜀĀ
ᜀĀᜀĀᜀĀᜀĀᜀĀᜀĀᜀĀ㔀ĀᜀĀᜀĀᜀĀᜀĀᜀĀᜀĀᜀĀᜀĀᜀĀ∀ĠᜀĀᜀĀᜀĀᜀĀᜀĀᜀĀᜀĀᜀༀЀĀĀĀᜀĀ
ᜀĀᜀĀᜀĀᜀĀᜀĀᜀĀᜀĀᜀĀ∀ĠᜀĀᜀĀᜀĀᜀĀᜀĀᜀĀᜀĀᜀༀЀĀĀĀᜀĀᜀĀᜀĀᜀĀᜀĀᜀĀᜀĀᜀༀЀĀĀĀᜀ
ĀᜀĀᜀĀᜀĀᜀĀᜀĀᜀĀᜀĀᜀĀ㘀ĀᜀĀᜀĀᜀĀᜀĀᜀĀᜀĀᜀĀᜀ㄀ ĀĀĀᜀĀᜀĀᜀĀᜀĀᜀĀᜀĀᜀĀᜀĀĀȀ⸀ĀЀĀȀĀ⸀Ā
ᜀĀᜀĀᜀĀᜀĀᜀĀᜀĀᜀఀ㄀ ĀĀĀᜀĀ∀ĠᜀĀᜀĀᜀĀᜀĀᜀĀᜀĀᜀĀĀĀЀЀĀȀĀ⸀ĀЀĀĀȀĀĀĀȀĀᜀĀᜀĀᜀ
ĀᜀĀᜀఀȀĀȀ⸀ĀЀĀȀĀ⸀ĀЀĀĀȀĀĀȀȀ⸀ĀᜀĀᜀĀᜀĀᜀĀᜀĀЀĀĀ⌀ЀĀȀĀ⸀ĀᜀĀᜀĀᜀĀᜀĀᜀĀᜀĀᜀఀȀЀĀȀ⸀
ऀऀЀĀȀĀ⸀ĀᜀĀᜀĀᜀĀᜀĀᜀĀᜀĀᜀ㄀ ĀĀĀᜀĀ∀ĠᜀĀᜀĀᜀĀᜀĀᜀĀᜀĀᜀĀᜀĀ∀༠ЀĀĀĀĀᜀĀᜀĀᜀĀᜀ
ĀᜀĀᜀĀᜀĀĀȀ⸀ĀЀĀȀĀ⸀ĀȀĀȀȀ⸀ĀĀȀȀ⸀ĀᜀĀᜀĀᜀĀᜀĀᜀĀĀĀĀЀĀĀĀ⌀ЀĀȀȀ⸀ĀĀȀȀ⸀ĀȀȀȀ⠀Ѐ⤀Ā
ᜀĀᜀĀᜀĀᜀఀȀĀȀ⸀ĀЀĀȀĀ⸀ऀऀЀĀȀȀ⸀ĀĀȀȀ⸀ĀȀĀĀЀĀᜀĀᜀĀᜀĀᜀༀĀĀĀᜀĀᜀĀᜀĀᜀĀᜀĀᜀĀᜀĀᜀĀĀȀ⸀ĀЀĀȀĀ⸀ĀȀĀȀȀ⸀ĀĀȀ
ᜀĀᜀĀᜀĀᜀĀᜀሀĀĀĀᜀĀᜀĀᜀĀᜀĀᜀĀᜀĀᜀĀᜀĀᜀĀ∀ĠᜀĀᜀĀᜀĀᜀĀᜀĀᜀĀᜀĀᜀༀЀĀĀĀᜀĀᜀĀᜀĀ
ᜀĀᜀĀᜀĀᜀĀᜀĀᜀĀ꜀ĀᜀĀᜀĀᜀĀᜀĀᜀĀᜀĀᜀĀᜀĀᜀĀ∀ĠᜀĀᜀĀᜀĀᜀĀᜀĀᜀĀᜀĀᜀༀЀĀĀĀᜀĀᜀĀᜀ
ĀᜀĀᜀĀᜀĀᜀĀᜀఀȀĀȀ⸀ĀЀĀȀĀ⸀ĀȀĀȀȀ⸀ĀĀȀȀ⸀ĀᜀĀᜀĀᜀĀᜀĀᜀጀĀĀĀᜀĀᜀĀᜀĀᜀĀᜀĀᜀĀᜀĀᜀ
ĀᜀĀ∀ĠᜀĀᜀĀᜀĀᜀĀᜀĀᜀĀᜀĀᜀ᐀ĀĀĀᜀĀ∀ĠᜀĀᜀĀᜀĀᜀĀᜀĀᜀĀᜀༀЀĀĀĀᜀĀᜀĀᜀĀᜀĀᜀĀᜀĀᜀ
ĀᜀĀĀĀȀЀĀȀĀ⸀ĀȀĀȀȀ⸀ĀĀȀȀ⸀ĀЀĀȀЀ⸀ĀȀĀĀԀĀᜀĀᜀĀᜀ‫؀‬ĀȀ⸀ĀЀĀȀĀ⸀ĀȀĀȀȀ⸀ĀĀȀȀ⸀ĀЀĀȀЀ⸀
ऀऀЀĀȀԀ⸀ĀᜀĀᜀĀᜀĀЀĀȀ⸀ĀᜀĀᜀĀᜀĀᜀĀᜀĀᜀĀᜀĀᜀᔀĀĀĀᜀĀ꜀ĀᜀĀᜀĀᜀĀᜀĀᜀĀᜀĀᜀĀȀĀȀ⸀ĀĀȀĀ⸀ĀЀĀȀȀ⸀ĀȀĀȀȀ⸀ĀĀ
ᜀĀᜀĀᜀĀᜀᜀĀĀĀᜀĀ∀ĠᜀĀᜀĀᜀĀᜀĀᜀĀᜀĀᜀ㄀ĀȀ⸀ĀЀĀȀĀ⸀ĀȀĀȀȀ⸀ĀĀȀȀ⸀ĀЀĀȀЀ⸀ĀᜀĀᜀĀ
ᜀĀᜀЀЀĀȀ⸀ĀȀĀȀĀ⸀ĀĀȀȀ⸀ĀᜀĀᜀĀᜀĀᜀĀᜀĀᜀ᠀ĀĀĀᜀĀᜀĀᜀĀᜀĀᜀĀᜀĀᜀĀᜀĀĀȀ⸀ĀЀĀȀĀ⸀ĀȀĀȀȀ⸀Ā
ᜀĀᜀĀᜀĀᜀĀᜀĀᜀᤀĀĀĀᜀĀ∀ĠᜀĀᜀĀᜀĀᜀĀᜀĀᜀĀᜀĀĀȀ⸀ĀЀĀȀĀ⸀ĀЀĀĀȀĀᜀĀᜀĀᜀĀᜀĀᜀĀᜀ
ԀĀȀ⸀ĀЀĀȀĀ⸀ĀᜀĀᜀĀᜀĀᜀĀᜀĀᜀĀᜀఀ㄀ ĀĀĀᜀĀ케ĥᜀĀ쬀ĥᜀĀꀀĥᜀĀᜀĀᜀĀᜀĀᜀĀĀĀĀЀĀĀĀĀȀĀĀȀЀĀȀȀ⸀Ā
ᜀĀᜀĀᜀĀᜀĀᜀĀĀĀĀЀĀĀĀȀȀĀȀȀ⸀ĀĀĀȀĀᜀĀᜀĀᜀĀᜀĀᜀ‫܀‬ĀȀ⸀ĀЀĀȀĀ⸀ĀȀĀȀȀ⸀ĀĀĀȀĀᜀĀ
ᜀĀᜀĀᜀĀᜀȀĀȀ⸀ĀᜀĀᜀĀᜀĀᜀĀᜀĀᜀĀᜀĀᜀ㄀ ĀĀĀᜀĀᜀĀᜀĀᜀĀᜀĀᜀĀᜀĀᜀ㄀ĀȀ⸀ĀЀĀȀĀ⸀ĀȀĀȀȀ⸀Ā
ᜀĀᜀĀᜀĀᜀĀᜀĀᜀĀĀĀȀЀĀȀĀ⸀ĀЀĀĀȀĀĀĀȀĀЀĀĀЀĀᜀĀᜀĀᜀĀᜀఀȀĀȀ⸀ĀЀĀȀĀ⸀ऀऀЀĀȀȀ⸀ĀĀȀȀ⸀ĀЀĀȀЀ⸀Ā
ᜀĀᜀĀᜀĀᜀȀȀĀȀ⸀ĀᜀĀᜀĀᜀĀᜀĀᜀĀᜀĀᜀĀᜀἀĀĀĀᜀĀᜀĀᜀĀᜀĀᜀĀᜀĀᜀĀᜀЀĀȀ⸀ĀЀĀȀĀ⸀ĀȀĀȀȀ⸀ĀĀȀȀ⸀Ā
ᜀĀᜀĀᜀĀᜀĀᜀĀĀȀ⸀ĀЀĀȀĀ⸀ĀȀĀȀȀ⸀ĀĀȀȀ⸀ĀᜀĀᜀĀᜀĀᜀĀᜀ℀ĀĀĀᜀĀᜀĀᜀĀᜀĀᜀĀᜀĀᜀĀᜀĀĀĀЀЀĀȀĀ⸀ĀȀĀȀȀ⸀ĀĀ
ᜀĀᜀĀᜀĀᜀఀȀĀȀ⸀ĀЀĀȀĀ⸀ĀȀĀȀȀ⸀ĀĀĀȀĀЀĀĀЀĀᜀĀᜀĀᜀĀᜀ∀ĀĀĀᜀĀᜀĀᜀĀᜀĀᜀĀᜀĀᜀĀᜀĀĀȀ⸀ĀЀĀȀĀ⸀ĀĀȀȀ⸀Ā
ᜀĀᜀĀᜀĀᜀĀᜀĀᜀĀЀĀĀЀȀĀȀĀ⸀ĀᜀĀᜀĀᜀĀᜀĀᜀĀᜀĀᜀఀȀЀĀȀ⸀ĀȀĀȀĀ⸀ĀᜀĀᜀĀᜀĀᜀĀᜀĀᜀĀ
ᜀ⌀ĀĀĀᜀĀ∀ĠᜀĀᜀĀᜀĀᜀĀᜀĀᜀĀᜀༀЀĀĀĀᜀĀ꜀ĀᜀĀᜀĀᜀĀᜀĀᜀĀᜀĀᜀĀᜀĀ∀༠ЀĀĀĀĀᜀĀᜀĀᜀĀ
ᜀĀᜀĀᜀĀᜀĀᜀĀ∀༠ЀĀĀĀĀᜀĀᜀĀᜀĀᜀĀᜀĀᜀĀᜀ─ĀĀĀᜀĀᜀĀᜀĀᜀĀᜀĀᜀĀᜀĀᜀЀЀĀȀ⸀ĀȀĀȀĀ⸀Ā
ᜀĀᜀĀᜀ1 civil fraud → clear and convincing evidence that taxpayer acted fraudulently (75%
penalty) (gov’t has burden)
ĀЀĀĀȀĀȀȀ⠀⠀⤀ĀЀĀĀЀĀᜀĀᜀĀᜀĀᜀЀĀȀ⸀ĀᜀĀ⨀ĀЀĀȀȀ⸀ĀĀĀȀऀऀЀĀȀЀ⸀ĀᜀĀᜀĀᜀĀᜀĀᜀĀ㄀Āᜀ
Ā∀ĠᜀĀᜀĀᜀĀᜀĀᜀĀᜀĀᜀ‫؀‬ĀȀ⸀ĀЀĀȀĀ⸀ऀऀЀĀȀȀ⸀ĀĀȀȀ⸀ĀЀĀȀЀ⸀ĀȀĀȀԀ⸀ĀᜀĀᜀĀᜀĀᜀĀ㌀ĀᜀĀ∀Ġ
ᜀĀᜀĀᜀĀᜀĀᜀĀᜀĀᜀĀĀȀ⸀ĀЀĀȀĀ⸀ĀȀĀȀȀ⸀ĀĀȀȀ⸀ĀᜀĀᜀĀᜀĀᜀĀᜀĀᜀĀ㐀ĀᜀĀ∀ĠᜀĀᜀĀᜀĀᜀĀ
ᜀĀᜀĀᜀĀᜀĀ㔀ĀᜀĀᜀĀᜀĀᜀĀᜀĀᜀĀᜀĀᜀĀᜀĀ∀ĠᜀĀᜀĀᜀĀᜀĀᜀĀᜀĀᜀĀᜀༀЀĀĀĀᜀĀᜀĀᜀĀᜀĀᜀĀ
ᜀĀᜀĀᜀĀᜀĀ∀ĠᜀĀᜀĀᜀĀᜀĀᜀĀᜀĀᜀĀᜀༀЀĀĀĀᜀĀᜀĀᜀĀᜀĀᜀĀᜀĀᜀĀᜀༀЀĀĀĀᜀĀᜀĀᜀĀᜀĀᜀĀ
ᜀĀᜀĀᜀĀᜀĀ㘀ĀᜀĀᜀĀᜀĀᜀĀᜀĀᜀĀᜀĀᜀ㄀ ĀĀĀᜀĀᜀĀᜀĀᜀĀᜀĀᜀĀᜀĀᜀĀĀȀ⸀ĀЀĀȀĀ⸀ĀᜀĀᜀĀ
ᜀĀᜀĀᜀĀᜀĀᜀఀ㄀ ĀĀĀᜀĀ∀ĠᜀĀᜀĀᜀĀᜀĀᜀĀᜀĀᜀĀĀĀЀЀĀȀĀ⸀ĀЀĀĀȀĀĀĀȀĀᜀĀᜀĀᜀĀᜀĀᜀఀȀĀȀ⸀ĀЀĀȀĀ⸀ĀЀĀĀȀĀĀ
ᜀĀᜀĀᜀĀᜀĀᜀĀЀĀĀ⌀ЀĀȀĀ⸀ĀᜀĀᜀĀᜀĀᜀĀᜀĀᜀĀᜀఀȀЀĀȀ⸀ऀऀЀĀȀĀ⸀ĀᜀĀᜀĀᜀĀᜀĀᜀĀᜀĀᜀ㄀ ĀĀĀ
ᜀĀ∀ĠᜀĀᜀĀᜀĀᜀĀᜀĀᜀĀᜀĀᜀĀ∀༠ЀĀĀĀĀᜀĀᜀĀᜀĀᜀĀᜀĀᜀĀᜀĀĀȀ⸀ĀЀĀȀĀ⸀ĀȀĀȀȀ⸀ĀĀȀȀ⸀Ā
ᜀĀᜀĀᜀĀᜀĀᜀĀĀĀĀЀĀĀĀ⌀ЀĀȀȀ⸀ĀĀȀȀ⸀ĀȀȀȀ⠀Ѐ⤀ĀᜀĀᜀĀᜀĀᜀఀȀĀȀ⸀ĀЀĀȀĀ⸀ऀऀЀĀȀȀ⸀ĀĀȀȀ⸀ĀȀĀĀЀĀ
ᜀĀᜀĀᜀĀᜀༀĀĀĀᜀĀᜀĀᜀĀᜀĀᜀĀᜀĀᜀĀᜀĀĀȀ⸀ĀЀĀȀĀ⸀ĀȀĀȀȀ⸀ĀĀȀȀ⸀ĀᜀĀᜀĀᜀĀᜀĀᜀሀĀĀĀᜀ
ĀᜀĀᜀĀᜀĀᜀĀᜀĀᜀĀᜀĀᜀĀ∀ĠᜀĀᜀĀᜀĀᜀĀᜀĀᜀĀᜀĀᜀༀЀĀĀĀᜀĀᜀĀᜀĀᜀĀᜀĀᜀĀᜀĀᜀĀᜀĀ꜀ĀᜀĀ
ᜀĀᜀĀᜀĀᜀĀᜀĀᜀĀᜀĀᜀĀ∀ĠᜀĀᜀĀᜀĀᜀĀᜀĀᜀĀᜀĀᜀༀЀĀĀĀᜀĀᜀĀᜀĀᜀĀᜀĀᜀĀᜀĀᜀఀȀĀȀ⸀ĀЀĀȀĀ⸀ĀȀĀȀȀ⸀ĀĀȀȀ⸀Ā
ᜀĀᜀĀᜀĀᜀĀᜀጀĀĀĀᜀĀᜀĀᜀĀᜀĀᜀĀᜀĀᜀĀᜀĀᜀĀ∀ĠᜀĀᜀĀᜀĀᜀĀᜀĀᜀĀᜀĀᜀ᐀ĀĀĀᜀĀ∀ĠᜀĀᜀĀ
ᜀĀᜀĀᜀĀᜀĀᜀༀЀĀĀĀᜀĀᜀĀᜀĀᜀĀᜀĀᜀĀᜀĀᜀĀĀĀȀЀĀȀĀ⸀ĀȀĀȀȀ⸀ĀĀȀȀ⸀ĀЀĀȀЀ⸀ĀȀĀĀԀĀᜀ
ĀᜀĀᜀ‫؀‬ĀȀ⸀ĀЀĀȀĀ⸀ĀȀĀȀȀ⸀ĀĀȀȀ⸀ĀЀĀȀЀ⸀ऀऀЀĀȀԀ⸀ĀᜀĀᜀĀᜀĀЀĀȀ⸀ĀᜀĀᜀĀᜀĀᜀĀᜀĀᜀĀᜀĀ
ᜀᔀĀĀĀᜀĀ꜀ĀᜀĀᜀĀᜀĀᜀĀᜀĀᜀĀᜀĀȀĀȀ⸀ĀĀȀĀ⸀ĀЀĀȀȀ⸀ĀȀĀȀȀ⸀ĀĀȀЀ⸀ĀᜀĀᜀĀᜀĀᜀᜀĀĀĀᜀ
Ā∀ĠᜀĀᜀĀᜀĀᜀĀᜀĀᜀĀᜀ㄀ĀȀ⸀ĀЀĀȀĀ⸀ĀȀĀȀȀ⸀ĀĀȀȀ⸀ĀЀĀȀЀ⸀ĀᜀĀᜀĀᜀĀᜀЀЀĀȀ⸀ĀȀĀȀĀ⸀ĀĀȀȀ⸀Ā
ᜀĀᜀĀᜀĀᜀĀᜀĀᜀ᠀ĀĀĀᜀĀᜀĀᜀĀᜀĀᜀĀᜀĀᜀĀᜀĀĀȀ⸀ĀЀĀȀĀ⸀ĀȀĀȀȀ⸀ĀᜀĀᜀĀᜀĀᜀĀᜀĀᜀᤀĀĀĀ
ᜀĀ∀ĠᜀĀᜀĀᜀĀᜀĀᜀĀᜀĀᜀĀĀȀ⸀ĀЀĀȀĀ⸀ĀЀĀĀȀĀᜀĀᜀĀᜀĀᜀĀᜀĀᜀԀĀȀ⸀ĀЀĀȀĀ⸀ĀᜀĀᜀĀᜀĀ
ᜀĀᜀĀᜀĀᜀఀ㄀ ĀĀĀᜀĀ케ĥᜀĀ쬀ĥᜀĀꀀĥᜀĀᜀĀᜀĀᜀĀᜀĀĀĀĀЀĀĀĀĀȀĀĀȀЀĀȀȀ⸀ĀᜀĀᜀĀᜀĀᜀĀ
ᜀĀĀĀĀЀĀĀĀȀȀĀȀȀ⸀ĀĀĀȀĀᜀĀᜀĀᜀĀᜀĀᜀ‫܀‬ĀȀ⸀ĀЀĀȀĀ⸀ĀȀĀȀȀ⸀ĀĀĀȀĀᜀĀᜀĀᜀĀᜀĀᜀȀĀȀ⸀Ā
ᜀĀᜀĀᜀĀᜀĀᜀĀᜀĀᜀĀᜀ㄀ ĀĀĀᜀĀᜀĀᜀĀᜀĀᜀĀᜀĀᜀĀᜀ㄀ĀȀ⸀ĀЀĀȀĀ⸀ĀȀĀȀȀ⸀ĀᜀĀᜀĀᜀĀᜀĀᜀ
ĀᜀĀĀĀȀЀĀȀĀ⸀ĀЀĀĀȀĀĀĀȀĀЀĀĀЀĀᜀĀᜀĀᜀĀᜀఀȀĀȀ⸀ĀЀĀȀĀ⸀ऀऀЀĀȀȀ⸀ĀĀȀȀ⸀ĀЀĀȀЀ⸀ĀᜀĀᜀ
ĀᜀĀᜀȀȀĀȀ⸀ĀᜀĀᜀĀᜀĀᜀĀᜀĀᜀĀᜀĀᜀἀĀĀĀᜀĀᜀĀᜀĀᜀĀᜀĀᜀĀᜀĀᜀЀĀȀ⸀ĀЀĀȀĀ⸀ĀȀĀȀȀ⸀ĀĀȀȀ⸀Ā
ᜀĀᜀĀᜀĀᜀĀᜀĀĀȀ⸀ĀЀĀȀĀ⸀ĀȀĀȀȀ⸀ĀĀȀȀ⸀ĀᜀĀᜀĀᜀĀᜀĀᜀ℀ĀĀĀᜀĀᜀĀᜀĀᜀĀᜀĀᜀĀᜀĀᜀĀĀĀЀЀĀȀĀ⸀ĀȀĀȀȀ⸀ĀĀȀ
ᜀĀᜀĀᜀĀᜀఀȀĀȀ⸀ĀЀĀȀĀ⸀ĀȀĀȀȀ⸀ĀĀĀȀĀЀĀĀЀĀᜀĀᜀĀᜀĀᜀ∀ĀĀĀᜀĀᜀĀᜀĀᜀĀᜀĀᜀĀᜀĀᜀĀĀȀ⸀ĀЀĀȀĀ⸀ĀĀȀȀ⸀Ā
ᜀĀᜀĀᜀĀᜀĀᜀĀᜀĀЀĀĀЀȀĀȀĀ⸀ĀᜀĀᜀĀᜀĀᜀĀᜀĀᜀĀᜀఀȀЀĀȀ⸀ĀȀĀȀĀ⸀ĀᜀĀᜀĀᜀĀᜀĀᜀĀᜀĀᜀ⌀
ĀĀĀᜀĀ∀ĠᜀĀᜀĀᜀĀᜀĀᜀĀᜀĀᜀༀЀĀĀĀᜀĀ꜀ĀᜀĀᜀĀᜀĀᜀĀᜀĀᜀĀᜀĀᜀĀ∀༠ЀĀĀĀĀᜀĀᜀĀᜀĀᜀĀᜀ
ĀᜀĀᜀĀᜀĀ∀༠ЀĀĀĀĀᜀĀᜀĀᜀĀᜀĀᜀĀᜀĀᜀ─ĀĀĀᜀĀᜀĀᜀĀᜀĀᜀĀᜀĀᜀĀᜀЀЀĀȀ⸀ĀȀĀȀĀ⸀ĀᜀĀ
ᜀĀᜀ2 plain old tax liability - preponderance of evidence that TP was correct in what he did
(TP has burden); TP can shift burden if cooperated w/ IRS during the audit, if produced
some “credible evidence” that supports position and if met certain reqs of maintaining
adequate records.
History & Constitutional Issues
0 1800s: Income taxes did not exist; the government’s revenue came from customs and duties
1 1860s: An income tax was instituted to raise revenue during the Civil War
2 1894: Congress passed a 2% income tax on individuals. The tax was deemed unconstitutional in
Pollock v. Farmer Loan and Trust. The Constitution prevented Congress from levying a direct tax
unless apportioned among the states so that each state's share of the total income tax would be
proportional to the population of that state (almost impossible to manage)
2.0 Pollock v. Farmers’ Loan & Trust (1895) → SC struck down the entire income tax as a direct
tax not apportioned among the states
3 1909: A corporate income tax appears. An income tax on corporations was not unconstitutional
because it was not seen as a direct tax on income. It was instead viewed as a tax on the corporation’s
privilege to conduct business as a corporation;
th
4 1913: 16 Amendment is passed in response to the critique in Pollock, allowing for an income tax.
Shortly thereafter, Congress imposes an income tax.
4.0 Congress imposed only low rates on high incomes so very few people were actually
impacted by the tax at its initial inception.
5 1940s: Through WWII and after, the nature of the income tax changed; evolved into a tax on the
people, not only the ultra-wealthy.
5.0 Scope of income tax changes → broad scope, higher rate
6 Now: Contemporary IRS is very accountable on a global scale; low corruption and
high professionalism
6.0 By using multiple taxes, including federal, state income, city/local, foreign, sales, excise
(gas, tobacco, alcohol), estate and gift, property, social security on wages, import/export
tariffs, Value Added Tax – can reduce rates and fraud
6.0.0 Vs. 1 central tax would require higher rates, with more potential fraud
7 Other income taxes not regarded as direct taxes → Sixteenth Amendment → allowed Congress to
tax income “from whatever source derived” without apportionment among the states.
7.0 U.S. doesn’t have wealth tax (as in Europe) could be regarded as a direct tax - 16th
Amendment allows for income tax, doesn’t refer to wealth tax; our estate & gift tax treated
as an assessment on the right to transfer property
7.1 Wealth transfer tax not seen as direct tax b/c not taxing income but the right to transfer
Marginal Tax Rate
a. Generally → marginal rate is the rate that applies to additional dollars of taxable income and thus is
the rate that affects tax decisions at the margin; in a progressive rate schedule, marginal tax rates
increase as income increases; the average tax rate is the total tax liability divided by income.
HYPO: taxable income is 20K; first 10K is 1000 (times 10%) next 10K is in next bracket so that’s
taxed at 20% which is 2000; their marginal tax rate is 20% b/c last dollar she earned was taxed at
20%; paying 3000 of taxes on 20K of taxable income so this person’s average tax rate is
3000/20K = 15%.
0 Principles of Tax Law:
a. Horizontal Equity → people in the same economic condition should be taxed the same;
i. How do we define similarity? In an income tax - determine which persons are the same by
their income; in a consumption tax (ex: sales tax) - determine similarity by consumption.
0 Vertical Equity → if people are different must make appropriate distinction b/t how you treat them;
0 How does our system achieve VE? Progressive income tax — the more you earn →
the higher the tax rate (VE met because distinction is as you earn more you pay more)

4
WNM
Tax 1 Ring (Fall 2017)

0 rationale for progressive tax — diminishing marginal returns; the more


you have of something the less utility it has for you; tax that millionth
dollar at a higher rate than first dollar because it means less to you. –
Also distributive justice
0 Violations of VE → Poll tax - regardless of how much you earn, everyone
pays the same amount; Flat tax - same % of tax applied regardless of what
you earn.
c. Efficiency
Administrative efficiency → simple tax system; easy to apply and to understand
ĀĀᜀĀᜀĀᜀĀᜀĀᜀĀᜀĀᜀĀᜀĀĀȀ⸀ĀЀĀȀĀ⸀ĀȀĀȀȀ⸀ĀĀȀȀ⸀ĀᜀĀᜀĀᜀĀᜀ0 Alloc
ational efficiency → system is not causing someone to do something they
wouldn’t do (i.e. allocate their resources differently) if the system was not in
place
0.0 Poll tax is allocationally efficient; consumption tax is not.
0.1 income / substitution effect
ĀĀᜀĀᜀĀᜀĀᜀĀᜀĀᜀĀᜀĀᜀĀĀȀ⸀ĀЀĀȀĀ⸀ĀȀĀȀȀ⸀ĀĀȀȀ⸀ĀᜀĀᜀĀᜀĀᜀ1 Tradeof
f → usually tradeoff between efficiency & equity which is about distributive
1.0 justice - how we treat similar and different people; ie: the more
equitable the system, the less efficient it is.
23 Simplicity
23 Tax rules are inefficient because TPs must divert time from other activities in
order to calculate their taxes (or earn the money to pay for professional tax
assistance) and because the government must maintain a large agency to interpret
these complex rules and to ensure that taxes are calculated correctly. Also, tax
liability should not hinge on individual’s ability to understand and manipulate
tax ambiguities.
5
WNM
Tax 1 Ring (Fall 2017)

GROSS INCOME
Benefits in Kind
In General
23 The Sixteenth Amendment allows the government to impose taxes on it’s citizens. IRC
§61 exercises this power.
24 The tax imposed on individuals and corporations by §§1 and 11 of the IRC,
respectively, are imposed on “taxable income.”
25 Income = undeniable accessions to wealth, clearly realized, and over which the
1
taxpayers have complete dominion.
§61(a): except as otherwise provided, gross income (GI) means all income from
whatever source derived, including but not limited to
5888 Compensation for services, including fees, commissions, fringe benefits, and
similar items*
5889 Gross income derived from business*
5890 Gains derived from dealings in property
5891 Interest*
5892 Rents
5893 Royalties
5894 Dividends*
5895 Alimony and separate maintenance payments*
5896 Annuities*
5897 Income from life insurance and endowment contract*
5898 Pensions
5899 Income from discharge of indebtedness (COD)*
5900 Distributive share of partnership gross income
5901 Income in respect of a decedent
5902 Income from an interest in an estate or trust
5889 covered in class so far

23 Benefits in kind = compensation for services in something other than case (e.g., meals,
lodging, etc.)
23 include the fair market value (FMV) of the benefit in income
0 Reginald Turner2
24 Fair Market Value (FMV) = Fair market value is the price at which the property would
change hands between a willing buyer and a willing seller, neither being under any
compulsion to buy or to sell and both having reasonable knowledge of relevant facts.
23 When income is received in the form of goods or services, the in-kind payments
must be assigned a money value under Treas. Reg. §1.61–2(d)(1) → usually the
FMV.
0 Reginald Turner
23 what is the difference between wealth and income?
23 Wealth  existing pool of accumulated assets
24 Income  new flows over the
2
course of the year Reginald Turner v.
Commissioner – what is FMV?
25 Facts: Turners won tickets in a radio contest for a cruise to Buenos Aires. (Turner originally won
two first-class tickets, but traded them for four coach tickets so he could go with his whole family).
Turner reported the tickets as income in the amount of $520, but the tickets had a retail price of
$2,200. IRS determined a deficiency in income tax from Turners for 1948.
26 Holding: Tax court determined that the Turners should have reported a more substantial
amount of the retail price ($1,400 – half the retail value). The Turners were unlike people who
voluntarily bought steamship tickets, so the value of the tickets to the Turners was not their retail
value because they were not transferable or salable and there were other restrictions on their
use. But they did accept the tickets and received some benefit from them, so the court basically
just split the difference between what the IRS wanted and what Turner reported.
6
WNM

Tax 1 Ring (Fall 2017)

23 Windfall

Lottery winnings or other windfalls are included in gross income


23
Haverly v. United States3
24 (Free Textbooks): Tax system does not allow for double benefits
(exclusion from income + deduction)
24 Found Property
23 the finder of treasure trove is in receipt of taxable income
24 taxable only in the year of discovery
25 Compensation for Services and Indirect Payments
23 Cash payments for services are includible in the income of the recipient.
24 There are other forms of compensation, besides cash payments, that may or may not be
included under a TP’s gross income.
23 Old Colony Trust Co. v. Commissioner:
23 Facts: Old Colony paid the federal income tax owed on the
salaries of some of its executives. The IRS said that the payment
of the executives’ taxes was additional income to the executives
and was therefore taxable.
24 Decision: The court upheld the IRS’s ruling because the
payment of the executive’s taxes was made in consideration of
the services rendered by the employee for his labor, so the
payment constituted income to the employee.
25 Notes: decision established the proposition that gross income under
§61(a) is not limited to situations in which cash is actually received
by a taxpayer. The receipt of economic benefit from a third party is
income.
4 TP must treat the economic benefit resulting from the
payment of their taxes in the same way as they would
treat the receipt for cash that they would then use to
pay off their taxes.
5 Policy: Any other approach would make it too simple to
avoid taxation by restructuring cash payment into
payments to third parties that discharge the taxpayer's
obligation.
6 Pyramid taxation, such as this, is not never-ending;
taxable income can be calculated.
7 Formula: 2 ( ) =( ) –
3 = −[ ( )× ]
4 = −[ × ]

5 = 1−
6 = ABC@

25 Tax. Reg. 1.61–2(d)(1): if services are paid for other than in cash, the fair
market value of the property or services taken in payment must be included in
income.
23 If the services were rendered at a stipulated price, such price will be
presumed to be the fair market value of the compensation received in the
absence of evidence to the contrary.
26 Rev.Rul. 79–24: goods or services received in exchange for services are income to the
recipient. Gain derived from labor is income regardless of the form of payment.
27 Income in kind is not limited to compensation for services. (see §61)
28 Almost any kind of income can be received in kind.
23 Dean v. Commissioner: a majority shareholder of a corporation who by virtue
of her status as a shareholder resided in a home owned by the corporation
realized dividend income as a result of the rent free use of the home.
23 Haverly v. United States (Free Textbooks)
23 Facts: school principal receives free samples of textbooks from publisher, SP donated the books to
his school library, and claimed a charitable contribution deduction.
24 Holding: the receipt of the books was an accession of wealth. Double tax benefit is realized
because SP did not include the books in his income AND he claimed the books as a charitable
contribution. If SP wants to deduct the books as a charitable contribution, he must first report them
as income.
7
WNM

Tax 1 Ring (Fall 2017)

Fringe Benefits
Compensation for services in something other than cash (e.g., meals, lodging, etc.). ER’s commonly provide benefits to EE in addition to their
23
compensation.

23 [Payment in kind is made in consideration of the services rendered by the employee for his labor, so the payment constituted income to the employee,

but it can be excluded under certain circumstances]

§132(a): Exclusion from gross income.


24
Gross income shall not include any fringe benefit which qualifies as a—
23
no-additional-cost service,
23
qualified employee discount,
24
working condition fringe,
25
de minimis fringe,
26
qualified transportation fringe,
27
qualified moving expense reimbursement,
28
qualified retirement planning services, or
29
23 qualified military base realignment and closure fringe.
Nondiscrimination Rules: §132(j)(1): If an employee is considered “highly compensated” (i.e., is a top paid employee) and he receive benefits that aren’t
25
otherwise made available to everyone equally, the EE cannot claim a tax exemption for that benefit.
ND rules only apply to no-additional-cost services and certain qualified employee discounts → other benefits in §132 can discriminate between
23
regular and high-level employees
Line of business limitation:
26
To be excluded, a service must be the (1) same type of service that is sold to the public in the (2) ordinary course of the line of business
23
of the employer in which the employee is performing services.
Ex] If an airline provides free seats to its pilots, those seats are not income to the pilots and thus they can be excluded from the
23
pilots’ tax returns.
Ex] If an airline buys a soup manufacturer and then provides its pilots with free soup, the pilots cannot exclude the soup
24
from income of their tax returns. The soup is income! The soup with be taxed!
Policy: looking to avoid competitive imbalances between small businesses and large conglomerates, which could provide employees with a
24
variety of tax-free benefits.

Benefit Definition and § Examples & Cases Comments

Employer §119(a)(1): excludable from GI if: Ex] a short lunch hour, Treas. Reg. 1.119-1(a)(2): The

Provided Meals ● For the convenience of the employer insufficient eating meals cannot have a “substantially

● Furnished in kind on the business facilities in the vicinity, compensatory purpose.” Cash
and the need to be on reimbursements for meals bought
premises
call for emergencies. elsewhere do not count.

§119(b)(4):
if more than 50% of the meals Benaglia (Exc.)4 The IRS will consider whether the ER’s

5
furnished to employees on an employer’s Kowalski (Inc.) policy of providing meals on site are

23 Benaglia v. Commissioner – § 119 (meals or lodging furnished for the convenience of the
employer)
23 Facts: Benaglia was a manager of several luxury hotels in Hawaii. To be able to do his job and
for the convenience of the hotel, Benaglia and his wife were required to live in and receive meals
from the hotels. Benaglia did not report the FMV of his room and board on his tax returns. The
IRS assessed that each return was short $7,845/year.
24 Decision: The court overruled the IRS and held that the meals and rooms furnished by
Benaglia’s employer were not part of his GI. Benaglia was able to show what his duties at the
hotel were and why his residence in the hotel was necessary to be able to do his job and was
a condition to his employment when he took the job.
23 Kowalski:
23 Facts: State trouper given $ for means while on duty. The amount of $ increased as your rank
went up. Got it during days off. ST did not report the $ as income. ST is taking the $ and eating
away from the station.
24 Issue: is it income? or does it fall under the §119 exception?
25 Holding: $ for meals ≠ meals furnished
26 Notes: what is the premises of the ER? The station? The state of NJ? What about constraints on
usage?
5888 It’s $ not meals! Seems more like compensation. Increasing with
rank is thus problematic. o The premises question has not been entirely
answered here.
8
WNM
Tax 1 Ring (Fall 2017)

premises meet the requirement, all of them do. reasonably related to the needs of the
ER’s business. These needs must be
apart from a desire to provide
additional compensation to its EE. The
IRS will also consider whether these
policies are in fact followed in the actual
conduct of the business. If reasonable
procedures are adopted and applied
and they preclude the EE from
obtaining a meal away from the ER’s
business premises during a reasonable
meal period §119 will apply.
Employer §119(a)(2): excludable from GI if: Benaglia (Exc.) Business premises7: the place where
Provided Lodgings ● For the convenience of the employer the employee performs any significant
● Furnished on the business premises HYPO: Dean R. gets portion of his duties
housing on campus. Treas. Reg. 1.119-1(a)(2): Cannot
● Employee was required to accept the
have a “substantially compensatory
lodgings as a condition of
purpose”
employment6 (because acceptance Treas. Reg. 1.119-1(a)(2)(i): If the
is required to enable the employee to test for lodging is met, meals provided
properly perform his duties) are automatically excluded from
taxation too.
Cafeteria Plans Allows an EE to choose among a buffet of cash HYPO: $100,000 salary Must be nondiscriminatory.
[not about food!] and non-cash benefits. AND the opportunity to
●Cash → Tax choose from $10,000 in Cafeteria plans are great because they
●Non-cash → No Tax cash or a buffet of cash do not force an EE to accept a benefit
benefits. he/she does not want.
●If Ω takes the $ →
taxed Note: Cafeteria plans dramatically
●If ß takes the non- increased the use of tax-exempt
$ option → not benefits to compensate EEs. Once
taxed Congress implemented §125 the
number of cafeteria plans grew
significantly! Gov’t. dislikes because
results in lost revenues.

Lodging for §107: Rental value of home or housing allowance provided to “ministers of No requirement for it to be on the
“ministers of the the gospel8” is not taxable. premises or a condition of employment
gospel” Applies to any religious leader9
No-additional cost §132(b): excludable from GI if: ex] pilots and §132(j): allows for reciprocal
service ● No substantial additional cost to the stewardesses fly for agreements
employer free; the exclusion ● ex] if airline worker A wants
● Benefit is the same service the would not apply if airline to fly standby on airline B, B
employer provides to the public had to bump regular is exempt from paying taxes
● Benefit is part of the service the customers off to
on the ticket so as long as A
accommodate pilot and and B have an agreement
employee performs his family because the and B incurs no substantial
airline is then incurring additional cost.
additional cost

Alternatives:
23 Reimbursements
24 Designated list of lunch places (contract with the restaurants)
25 Grossing-up: increasing the EE's salary. Could be problematic: constraints on salary that might
not exist for other benefits
26 Vouchers: way of navigating $ v. meals.
23 vouchers might have constraints
23 Does the voluntary voucher system (res. making a donation to the gov't.) really resolve the
issue? Or are we
back to a §119 compensation question?
0 Employment contracts are not determinative. Contract does not prove that the lodgings are not
compensation. They do not prove that there was a need to house the EE on the premises. (Risk of
abuse; adding language to an employment contract is not hard).
7
Raises lots of litigation for housing. Is it really a business premises? (hotel EE example)
8
Does not apply to other 501(c)3 entities.
9
Possible rationale: Religious leaders provide a social service/community benefit, while making little
money.
9
WNM
Tax 1 Ring (Fall 2017)

10
§132(h)(3): Special rule for parents in Only applies to services, never anything
the case of air transportation. Any use of tangible (goods).
air transportation by a parent of an employee
(determined without regard to paragraph (1)(B)) Non-discrimination requirement
shall be treated as use by the employee.

Qualified §132(c): excludable from GI if: Policy: it benefits the employer


employee ● Property or service is offered for sale by the employer to non- because it helps the employee better
discount employee customers advertise and sell the employer’s
● Property or service is offered in the ordinary course of the good/benefit; if the EE has used the
employer’s line of business product themselves, they can better
● Employee works in that specific line of business advertise the product.

Employee Discount (difference between price offered to employees and


price offered to customers) that does not exceed:
● property → the gross profit % of the price which is being offered
to customers OR gross profit % is the markup; (diff b/t customer
price & cost/customer price)
○ Ex] shirt sells for $100 in the store; it costs $50; the
gross profit % (100-50/100) = 50%
● services → 20% of the price at which the services are offered to
customers
○ ex] if law firm drafts wills for clients for $1000, can
draft wills for employees in the firm for $800

Residual Statutory Fringes (§13211)


Catch-all ––– replaces any remaining common law fringe benefits or administrative rules that have been issued. These
are the only things →
nothing else!!

Working §132(d): excludable from GI if: Ex] office supplies See §162
Conditions Fringe ● expenses which, if the employee had
paid for himself, the would be HYPO: ER provides EE Treas. Reg. 1.132-5 (a)(1)(v) (v): A
claimable as a business deduction with copy paper. cash payment made by an employer to
under §162 or 167 §132(d) avoids a two- an employee will not qualify as a
● Extends to partners and independent step deduction. Without working condition fringe unless the
§132(d), you would employer requires the employee to:
contractors Reg. 1.132-1(b)(2) have to report the paper (A) Use the payment for expenses in
as income and then connection with a specific or pre-
deduct it as a business arranged activity or undertaking for
deduction. which a deduction is allowable under
section 162 or 167,
(B) Verify that the payment is actually
used for such expenses, and
(C) Return to the employer any part of
the payment not so used.

De Minimis Fringe §132(e): If the FMV of any property/service ex] free coffee at work Policy rationale:
that would otherwise be included in GI is so in the mornings; holiday ● Building into the statute
small that accounting for it would be turkey; birthday cakes administrative ease (not
unreasonable or administratively impracticable worth having the TP account
→ the value is excluded. Ex] meal $ for junior for this––statutorily ignoring
associates working long these small expenditures)
The term “de minimis fringe” means any hours on a case. (Note: ● Designed to allow ERs to
property or service the value of which is (after might not be as provide their EE’s with these
taking into account the frequency with infrequent in many small benefits, while also
which similar fringes are provided by the settings, such a junior avoiding compensation
employer to the employer’s employees) so small associate in a law firm.) disguised as de minimis
as to make accounting for it unreasonable or fringe.
administratively impracticable.
Has no topical constraints –– other than
Reg. 1.132-6: Occasional meal money or frequency.
local transportation fare.
Meals, meal money or local transportation fare Equity problems → somewhat
provided to an employee is excluded as a de violates horizontal equity because one
minimis fringe benefit if the benefit provided is employee pays for coffee and other

Airline lobby was extremely successful here! We went from fringe benefits as being a necessity for
employment to giving free tickets to your parents!
Codified common law fringes. Anything not mentioned in §132 is not a fringe benefit.
10
WNM
Tax 1 Ring (Fall 2017)

reasonable and is provided in a manner that gets it free at work; not really vertical
satisfies the following three conditions: equity problem because its so minimal.
(A) Occasional basis.
(B) Overtime. Note: If it’s something too valuable or
(C) Meal money. consumed during the it happens too frequently, it’s not de
period that the employee works minimis
overtime.
Qualified Excluded if ER provided transportation, such as qualified parking, transit Anti-discrimination rules do not
Transportation passes, commuter highway vehicle use, etc. apply. Executives may be provided free
Benefits parking while rank and file workers are
not.
Qualified Tuition §117(d): excludable from GI if it’s a tuition Ex] BC professor’s son Cannot discriminate in favor of
Reduction discount given by an education employer to its receives free tuition highly compensated individuals
employee below the graduate level.

Athletic Facilities §132(j)(4): excludable from GI if: HYPO: IBM sets up a Contrast to no-additional cost services.
● Offered on the premises of the gym/country club nexts
employer to its parking lot and all Policy: in the ERs interest to have a
● Operated by the employer EEs have free access. healthy workforce.
● Substantially all of the use is by HYPO: what if IBM
employees (including spouses and rents a room at Planet
dependents) Fitness? –– gets murky
Others12 Employee life insurance; certain debt benefits; health plans; provisions for members of the armed forces; education
benefits; childcare benefits

Why does the tax system view benefits in kind (e.g., getting paid by your employer in the
form of a Cadillac) as income? What would happen if we did not?
If the tax system did not tax benefits in kind then instead of paying salaries, which are taxable to the
employee (EE), employers (ER) would simply pay their EEs in kind. For example, instead of paying an
EE a salary, the ER might pay for his house and his car. The car is consideration for the services
rendered by the employee for his labor, so the payment constituted income to the employee.

If your employer gives you a Cadillac with a FMV of $27,000 how are you treated?
The Cadillac with a FMV of $27,000 is income to the EE because it is consideration for the services render
by the employee.

What happens for tax purposes if you sell the car 30 days later for $28,000?
If ER gives EE a car at an FMV of $27,000, EE will be taxed on the $27,000 as if it were cash income.
Therefore, EE can count the $27,000 as his basis. It would be no different than if EE had gone out an
purchased the car with $ 27,000 in cash. Therefore, when EE sells the car of $28,000, the GI = AR – B
= $28,000 - $27,000 = $1,000. EE would be taxed on $1,000 in gain from the sale.

Maybe if the car was furnished to the EE for some business purpose? What if EE was a traveling
salesman?

Imputed Income
0 Income earned from using/doing things for yourself → NOT TAXED
1 Policy: Why isn’t imputed income taxed?13
0 Valuation → Assessing the value of goods/services is extremely difficult
1 Administrative burden → The additional administrative burden to taxpayers and the
IRS of calculated imputed income
2 Negative impacts on spending → disincentives ownership
5888 Frequent Flyer Miles: The ER is buying the ticket for business use, but the EE gets the benefit of
using the miles on those tickets for personal use. There is no explicit statutory provisions for these. But you
might turn to the de minimis fringe (might be tough to argue).
5889 Why might we choose to tax imputed income?
5888 It’s fair, we want everyone playing a fair share of taxes
5889 More accurate to do so
5890 Ways in which it might influence behavior. (might make more people want to go out an
work)
5891 Distributional effects on socioeconomic groups – gender divide

11
WNM
Tax 1 Ring (Fall 2017)

23 Intrusion → Concerns about the invasion of privacy of having the government


reach into a private home to charge taxes
24 Liquidity → would require making sure each individual TP has saved enough for
taxes
5888 Benefit of using your own assets
5888 Ex] using your own washing machine instead of going to the dry cleaners,
owning your own car
5889 HYPO: Renting v. living in your own house. A buys a house for $100,000.
A rents the house to C for $10,000 in rental income. B buys an identical house for
100,000 lives in it.
i. Outcomes:
5888 A’s income = $10,000 of rental income.
5889 B’s income = $0
5890 We tax market rental transactions but we do not tax
imputed income because it is not a market transaction. Even
though, as the above example illustrates, A and B are
identical in almost every way, they made identical investments in identical homes. A
has taxable income and B does not.
ii. Ways to resolve the disparity:
1. Give A a deduction on his rent.
14
2. Give A a tax credit to encourage rentals
4. Benefits of providing services for yourself:
a. HYPO: A is a lawyer. A earns $10,000. A hires B for housework and pays him $5,000. How much
taxable income exists in this world? $15,000.
b. HYPO: A and B get married. They are now one taxable unit. B’s services are provided within the
home. Any transfer of cash is invisible. The services B provides are no longer in the market. How
much taxable income exists in the world? $10,000.
c. HYPO: housework v. market pay
Pre-Tax After Tax
Home Services $8,000 $8,000
Market for CPs $10,000 $7,000 (at a 30% tax rate)
5. Bartering: Goods or services are used as a means of exchange with third parties in lieu of using cash.
a. HYPO: A is a plumber. B is a roofer. A goes over to B’s to do the plumbing and B goes over A’s to
fix his roofs after a terrible winter storm.
b. HYPO: TP is a news reporter by day earning $800/day. TP needs to paint her house. Painting services
cost her $500.
i. Tax-Free: If there is no tax in this world, what should the TP do? Spend the day working,
while paying someone to paint the house.
5888 Taxed: If there is a 50% income tax in this world, what should the
TP do? Stay at home and paint the house herself. TPs income after tax is
only $400. So, she would be saving $100 by staying home.
← HYPO: Suppose A could earn $2,000 this weekend by working. Instead she
decides to spend the weekend chilling out. So A must be getting $2,000 worth
of leisure. Should she be taxed?
← NO! The leisure is outside of the market.
← HYPO: Should we tax people based on their abilities? A and B graduated from
BC. A went to NYC to work at $200,000/year. B got the same job, but turned it
down and went to work in NC for $100,000/year. B had the choice of making
$200,000. He gave up the $100,000 to have a different life. Since he had the ability
to earn the added $100,000, should we tax him on it?
← NO! The leisure is outside of the market.

← How are credits and deductions different? Do they result in different amounts of tax paid by
the same TP?
← $5 credit  reduces tax bill by $5
← TP has taxable income of $100, the tax rate is 10%, and TP has a credit of $5. Then he
will only pay $5 in taxes.
← $5 deduction  reduces taxable income by $5
← If TP has taxable income of $100, the tax rate is 10%, and TP gets a deduction of $5,
then he will only be taxed on $95 and pay $9.5 in taxes instead of $10 without the
deduction.
12
WNM
Tax 1 Ring (Fall 2017)

Recovery of Capital
Introduction to Basis
← Three Considerations
← Is there a taxable event by which gain or loss is realized?
← What is the amount realized on a sale, exchange, or other disposition of property?
15
← What is the adjusted basis of the property given up?
17
← §100116 Amount Realized (AR) – Basis (B) = Gain/Loss (Taxable Income)
← §1012: Basis of property: its cost; what the TP invested.
← Ex] A pays $15,000 for Cadillac worth $10,000; basis = $15,000
← When there is an exchange of property, the TP must determine the
basis of the property received in the exchange.
← Adjusted Basis (§1011(a))
← HYPO: If A buys a house for $200,000 and then spends
$50,000 on home improvement projects, what would be the
adjusted basis of the home? $250,000.
← §1001: Amount Realized: what the TP is paid when he sells the property
← HYPO: A buys land for $200,000 and sells it several years later for $300,000.
What is A’s gross income from the sale?
← Options:
← (A) A has no gross income from the sale.
← (B) A has $300,000 of gross income from the sale because that is
the amount A has available for consumption after the sale.
3. (C) A has $100,000 of gross income because A has $100,000
more than A paid for the property.
← Answer (C):
← (A) is incorrect because, selling for profit is too familiar a
business transaction to permit us to suppose that it was intended
to be excluded from taxation. Income derived “from all
sources” can be taxed. [Doyle]
← (B) is incorrect because, taxing the entire proceeds of a mere
conversion of capital assets also seems inconsistent because a
conversation does not always result in a gain. [Doyle] The initial
$200,000 used to buy the land had already taxed when it was
first earned. Taxing it again does not make sense.
← (C) is correct, because:
• – = / ( )
• = $300,000 − $200,000 = $100,00

← §61: Income for purposes of §61 is the realization of gain only after recovery of the
taxpayer's original capital investment. The government cannot tax beyond AR – B
(beyond that which Constitution authorizes).
Income is only that which exceeds your initial investment.
← HYPO: The official price of a Cadillac is $10,000; A pays $15,000 for the car and
later sells it for $15,000? Can A be taxed for the $5,000 difference? No, to do so
would be unconstitutional after Doyle (no new income here).
← HYPO: If you buy property and sell it, your initial investment (the basis) is
returned to you untaxed.
← Buy land for $1,000, sells it for $1,500.
← = $1,500 ( ℎ ℎ ℎ )
← − = −$1,000 ( ℎ )
← =$ ( → )
← = $1000 ( )

ii. Avoids double taxation


In tax accounting, adjusted basis is the net cost of an asset after adjusting for various tax-related
items. Adjusted Basis or Adjusted Tax Basis refers to the original cost or other basis of property,
reduced by depreciation deductions and increased by capital expenditures.

We do not adjust for inflation!

Basis: Investment in or cost of the property, item, etc.
13
WNM

Tax 1 Ring (Fall 2017)

Damage Payments
Payments made to the taxpayer to compensate for loss or damage suffered.

HYPO: Dean writes a book. The Boston Globe prints the book without his consent. Dean sues and wins. Are the damage payments income?

There is no simple answer! But, we must figure out what the damages are for before we can determine whether they will be taxed.

TEST: What are the damages replacing18
← ?
← If damage payments are received “in lieu of” receipts that would have been taxable
(reimbursement for lost profits, lost wages)
← Income → damage award is taxable
← If damages received “in lieu of” of non-taxable receipts or to compensate for
a loss of capital (destruction of the business (physical assets))
← Return of capital → damage award is NOT taxable
← If the damages are replacing something that the taxpayer has already paid
for/invested in, then it’s a recovery of capital until it exceeds that investment
(burden is on the taxpayer to show her basis).
← But compensation for loss of goodwill in excess of its cost is
income.

← Raytheon
← Facts: Raytheon (TP) sued RCA, claiming that RCA infringed on their patents, ruined their cathode-ray
tube business, and damaged their company's 'goodwill' (e.g. their brand name, market share, etc.). TP
won a $410k settlement. Out of the $410k TP received, they estimated the value of the patents at $60k.
TP filed taxes claiming the $60k as gross income and excluding the remaining $350k because it was paid
for the destroyed goodwill. The IRS claimed that the $350k that TP received for the settlement of the
suit was also taxable as gross income. TP argued that it wasn't gross income at all, but a replacement of
capital, which was not taxable. π claimed that they weren't getting income, but that they were just being
reimbursed for damages.
← Holding: The $350k represented a replacement of capital intended to reimburse TP for the loss of their
cathode-ray tube business. The Court suggested that the question to ask was, "in lieu of what were the
damages awarded."
o If the damages were for loss of profits due to an injury on your business, then the damages are
a substitute for lost profit and are taxable as gross income.
o On the other hand, if the damages were for loss of a capital item, then the damages are to replace
what you lost (aka replacement capital), and are not taxable as gross income.
o The Court also found that when RCA reimbursed TP for the loss of a business unit, that was
basically the equivalent of RCA buying the business unit from TP. Unfortunately for TP, that
meant that they had realized the value of the business, and would have to pay taxes on the
realized gain (aka the amount realized – adjusted basis) of the cathode-ray tube business. The
Court found that the adjusted basis of the business unit was almost nothing. So almost all of the
payment made to Raytheon ended up being taxable anyway, because it was made in excess of
reimbursement. [see 26 U.S.C. §1001].
← Damages treated like a forced sale.
13b
Clark
← Facts: Clark hired a lawyer who gave him some bad advice and Clark paid more in taxes than he needed
to. In order to make amends (and avoid a malpractice lawsuit), the lawyer reimbursed Clark for the
erroneous taxes that he paid. By the time the mistake was realized, it was too late to file an amended
return and get the money back from the IRS. The IRS claimed that the reimbursement was taxable as
gross income. Clark disagreed. Clark argued that this wasn't gross income, but just replacement of
capital.
← Holding: The Tax Court found for Clark. The Tax Court compared Clark's position at the beginning of the
series of events to the position at the end of those events. Clark had exactly the same amount of money
at the end as he had at the beginning. Therefore, he never gained any income, he was just put in the
place where he would have been had the lawyer not made a mistake.
← Notes: IRS cites Old-Colony. But the Court found for Clark because he is paying his own taxes! No one
paid Clark’s taxes. Clark paid his own taxes, the lawyer simply reimbursed him for the harm done when
Clark overpaid. Clark could not take a deduction in the first place.
← This set of cases is unique:
← Compensated with $20K for something that you had a right to enjoy TAX FREE to begin
with. Normally you should have this status tax free. Then someone harmed you. Then they
compensate for your harm and brought you back to the place you were in. the thing you
have the right to enjoy tax free.
The best possible legal tax return. You don’t have extra income
when you are being compensated back to that point.
14
WNM
Tax 1 Ring (Fall 2017)

← HYPO: Bush buys land on the coast of Maine in two parcels. Parcel 1 is 27 acres
and includes some coastal land and some non-coastal land for $1.4M. Three months
later Bush buys Parcel 2 (3 acres of non-coastal land) for $100,000. Bush then sells
24 acres of non-coastal land for $1.24M.
← TP: (makes an Inaja land argument)
← On his tax return Bush argues that this was a part sale of a
package, thus he gets a return of basis first!
← The pool of basis is $1.5M. And I have not gotten $1.5M back
yet! Therefore, I get to take this $ as part of my return of basis
until I get that the full $1.5M back.
← IRS:
← Where there is a partial disposition, the IRS will push for the
method in Option 3. It will want to split the non-coastal and
coastal land.
a
b
NC is about $33,333/acre ($100,000 for 3 acres)
Basis: $33,333 × 24 acres = $800,000

c $1.4M - $800,000 = $449,000 = GI


← HYPOs:
← A → Hires a perfect tax attorney. Pays no additional taxes than necessary.
← B → Hires an idiot. Ends up paying $20 extra.
← C → Hires a moral idiot. Ends up paying $20 extra. But the moral idiot
compensates him for the mistake by reimbursing him for the extra $20.
← Considerations:
← Should C be taxed on the $20 reimbursement?
a According to Clark, no, because the $20 is bringing C
back to the position he would have been in if not for the
mistake of the tax lawyer.
← Should B be allowed to exclude $20 in other income from his future
returns?
a No, we don’t really trust B. We don’t want people to just
exclude income willy-nilly because they think that they
overpaid last year. In C’s case, a third party is admitting
fault to a certain amount so we trust that it is correct and
allow that $ to go untaxed.
21
← Damage payments for human capital (i.e. product of your labor)
← Turns on voluntariness
← If the TP is part of the workforce → taxable
← If the TP was part of a labor camp or a prisoners of war → not taxable
← Ex] payments from the German government to citizens who had
been imprisoned in camps during the Nazi occupation were not
taxable because the damages were compensation for human rights
violations.
d. Personal freedoms are enjoyed tax free. Therefore, the
restoration to this baseline should not be taxable.
i. Replacing imputed income?? (possible way to
think about these damage payments)
← Personal injury22 payments:
Injury Tax Treatment
Physical Injury (e.g., broken leg) Entirely excluded from GI
Emotional Distress Generally, included in GI
Medical bills are excluded (e.g., TP goes to a trauma counselor)
If in conjunction with physical injury  emotional distress is excluded
Other (e.g., defamation) Entirely included in GI
Punitive Damages Entirely and always included in GI
← §104(a)(2): non-punitive damages for physical injury are excludable from GI

← There is no parallel concept of basis for human capital (income from wages).
← Actions for personal injury include claims for:
← Lost wages
← Loss in future earning power
← Reimbursement for medical expenses incurred or to be incurred in the future
← Pain and suffering
← Punitive damages

16
WNM
Tax 1 Ring (Fall 2017)

← Employee-provided insurance
← §104(a)(3): Individual insurance plan (employee-provided)
← RULE: payouts received through accident/health insurance or
personal injury or sickness (§104(a)(3))  Not Taxed
← Payments received through accident or health insurance are excludable
from GI provided that the policy was not financed by the taxpayer’s
employer or by employer contributions that are not includable in the
taxpayer’s income
← If your ER pays for your insurance and you get the benefit of excluding
that from income then you cannot exclude the payouts from that
insurance again!
← Applies to those policies that the TP pays for HIMSELF.
← Employer-provided insurance
← Policy: Congress has enacted a number of provisions excluding specific
fringe benefits of substantial value for the purpose of encouraging
employers to offer the benefits. These excluded fringe benefits include
health and accident insurance.
← §105: Employer-provided health and accident plans
← §105(a): include payout of employer-provided policy
← §105(b): exclude payments covering medical expenses
← §105(c)
← (1): exclude other payments not related to wages
← (2): If plan covers sick pay – (like wages) – include
← §105(b) and (c): TP cannot exclude all other payouts, except
when it’s a payout for medical care (§105(b)) or for the loss of
bodily functions (§105(c))
← §106: Contributions by employer to accident and health plans.
← Excludes any payments by an employer to a health or
accident plan. Thus, medical or accident insurance
premiums paid by an employer are excluded from
taxation.
← exclude premium paid by employer on your behalf
← Policy:
← Preference for ER provided over EE provided
← Preference for MP over AP
← Double Deduction:
(a) ER MP → buying is not treated as a benefit nor is the
payout
(b) ER AP → buying is not treated as a benefit nor is the
payout
Medical Plan Premium Paid Receipt of Payout
24
Employee provided Partially deductible under §213 Excludable under §104(a)(3)
Employer provided Excluded/Deductible under §106 Excluded under §105

Accident Plan Premium Paid Receipt of Payout


Employee provided No deduction Excluded under §104(a)(3)
Employer provided Excluded/Deductible under §106 Excluded under §105(c)
← HYPO: TP a 1% chance that he will suffer $100 of harm. So he are willing to pay $1
for insurance.
← Scenario 1: Tax-free world:
← $1 of premium for $100 of insurance coverage.
← Scenario 2: Premium payments are deductible AND the receipt of
any insurance are taxable. Tax rate is 50%. Based on his risk, TP
wants $100 of insurance.


TP will pay $2 for $200 in insurance.
$100 will be taxed ($200 - ($200 × .50) = $100)

← $1 will be deducted from TP’s returns.


← Scenario 3: Premiums are not deductible. Insurance receipts are
not included in income. TP wants AT = $100
← TP will pay $1 for $100 in insurance.

← The employer provided plans offer the TP a double benefit in both scenarios.
18
WNM
Tax 1 Ring (Fall 2017)

Tax Benefit Doctrine


§§186, 172, 111
← Tax benefit doctrine: if a taxpayer recovers as a loss or expense that which was deducted in
the previous year, the recovery must be included in the current year’s GI to the extent that it
was previously deducted.
← Annual Accounting Period:
← Sanford & Brooks25 established that each annual accounting period generally is treated
as a separate compartment so that the income and deductions of one year have no effect
upon other periods.
← Policy: important for administration. A tax system must produce revenue at
regular intervals. Waiting until the end of a transaction to collect taxes
would not result in a regular flow of taxes for the government.
← Capital Expenses:
← Dobson: D sold stocks in 1930 and 1931 at a loss. He received $29,000 in
partial damages in 1939 when he learned that the shares were fraudulent. He
did not report these damages as part of his GI. The IRS sued. TP never
recovered his original investment, even after receiving damages. It’s not as
if the TP is opening up old returns; it’s as if we’re looking back to those old
returns to get information as to how to tax the TP in the present. 26 Here,
the outlay in question (i.e. the purchase of the stock) was a capital expense
and if TP doesn’t recover it, then he shouldn’t be taxed on damages in a later
year (distancing from S&B). Thus, capital expenses are more protected
from tax liability than ordinary, general expenses (i.e. wages)
– TP is entitled to recovery of those capital expenses before he is taxed.
← Congressional attempts to alleviate annual accounting problems: Harshness of the IRS’s
rule about annual accounting is exemplified in Sanford & Brooks
← Net Operating Losses (NOL) §172: the excess net loss can be carried back 2
years or forward 20 years to be used on other returns (§172(b)(1))
← HYPO: In 2016, TP’s business earned $0 income, but suffered $20,000
in losses. Expense deductions would not benefit TP in 2016, so she did
not include them on her tax return. However, TP can carry the excess net
loss back 2 years or forward 20 years to be used on other returns.
← Requires NO transactional link between loss and income
← Cannot “hoard” NOLs in anticipation of high rate years
← Exclusionary Tax Benefit Rule: §111(a)-(c) and Reg. 1.111-1(a)(¶1): to the extent
that a previously deducted amount did not produce a tax savings, its recovery does
not constitute GI
← HYPO: if the taxpayer experiences a $2,000 theft loss in Year 1 but cannot
reduce its income and the property is returned to the taxpayer in Year 2, the
$2,000 is not income in Year 2 and does not need to be reported.
← Very broad rule → no time limit
← When both §172 and §111 are applicable, §172 trumps
← Deduction for recoveries of damages for antitrust/contract violations §186: if
you receive compensation for some wrong done to you, you must include the
compensation in GI, but you can also take a deduction for the amount of the
compensation or for the amount of uncompensated loss (whichever is less)
← Accomplishes the same end as §111 → the two rules are pretty much
interchangeable
← Sanford & Brooks:
← Facts: Sanford & Brooks had a contract to dredge the Delaware River for the government
between 1913 and 1916 and spent about $176,000 to do it. Except for 1914, the company ran
huge tax losses every year so they were not taxed. S&B stopped working and sued the
government to recover the excess losses that went outside what their deal was and recovered
$192,000 in 1920, but didn’t include it in their income. They claimed this money was a recovery of
capital on the transactions that spurred the suit and the corresponding damages because the
deductions in 1913-1916 were essentially wasted since they didn’t have any gross income at the
time and at the time there was no mechanism in the law to revise their previous returns.
← Holding: Court ruled that the compensatory damages were to be included in S&B’s GI for 1920.
The damages they received had to be included in GI for the year they were received rather than
the years during which the contract took place because Congress has elected to use annual
accounting for predictable revenue and administrability, which is not unconstitutional.
← This is what distinguishes Dobson from Sanford & Brooks.
19
WNM
Tax 1 Ring (Fall 2017)

Annuities
§72(a)-(c), (e)(1) - (3), (f)

ANNUNITIES DEFINED:
← Annuities: low-risk investments where the insured pays a company a lump sum at the start
of the contract and gets it back in fixed payments, either in a lump sum or over a period of
time.
← ex] TP is 50 years old and purchases an annuity for $100,000, the company might
agree to pay him $1,000/month for the rest of his life. If the TP lives for 33 years, he
will have received $396,000.
← Types:
← Term annuity: payouts come over a specific period of time
← Life annuity: payouts come for the rest of your life
← Deferred annuity: payouts are delayed
← Employer funded annuity
← Why set up an annuity?
← Annuities are often part of a retirement plan (especially life annuities) because
they continue paying out even after the TP no longer has other income. The TP can
structure the payout so that he will receive a certain amount.
i. Best to buy one early so it has time to grow
← Large life-altering injury → family may set up an annuity to guarantee life
payments so that the payments aren’t dependent on the family members being
there
← Life annuities are attractive because even if you live past your life expectancy the
insurance company manages this risk by collecting from customers who die sooner
than expected to fund customers who live longer than expected.
← Risks of an annuity:
← Insurance company granting the annuity could go under (depending on the type
of insurance, some states have backup pools to cover contracts if this happens)
← Inflation
← Risk of outliving your term annuity  payments stop

TAXING ANNUITIES:
27
1. §72 : each payout from an annuity is divided into capital and income:
d. ^_`abca @aechC noigck`k o

deCfg @hCbibjfC`k l`i`bjCm


= @hhcfg ^fpa`hC l`i`bq`k

← Term annuity:
← =( ) × (# ℎ )

HYPO: A pays B $1,735 in premiums. B agrees to pay A $1,000/year for 2 years.



The Old 3% Rule: (<1950) Of your annual annuity payment the amount that is taxable equals
3% of your premium price.
← HYPO: Premium = $100. Annuity payment is $10. $3 is taxable. $7 is not taxable.
← Problems with the 3% Rule:
← HYPO: Premium = $100,000. Annual payment = $4,884/year.
← Under the 3% rule: $3,000 was taxable $1, 884 was not taxable.
← Problem is he would have to live 53 years to recover his basis. But if you looked at his
actuarial table his expected life expectancy was 28 years. TP argued that this is
unconstitutional and arbitrary because there is no way he can recover his basis.
← The Court said, that TP does not need to recover his basis first!
← HYPO: In 1950: At age 20 A buys an annuity to kick in at age 60. In 1989: At age 59 B buys
an annuity to kick in at age 60.
← Relative to A’s premium, B’s premium will be very very high. Same age taxpayer
collecting the same payout over the same # of years could be paying very different
premiums.
← The bigger your premium the more of your own money is coming back to you! For A,
virtually everything is income! But for B, much of his payouts are a return of his own
money.
← The higher your premium relative to your payout, the more of your own money you are
getting back.
← A put so little cash in that most of her payout is coming from new money that was
made investing A’s original cash. B put in so much $ that most of his payout is
coming from his own money.

20
WNM
Tax 1 Ring (Fall 2017)
ii. = $1,000 × 2 = $2,000

i. $A,stu o

iii.
iv.
$v,www =$A,www
= $867.5 0

= $1,000 − $867.50 = $132.5

Year 1 Year 2
Principle (recovery of capital) $867.50 $867.50
Interest (income) $132.50 $132.50 $165
Total $1,000 $1,000
c. HYPO: A loans B $1,735 at a 10% interest rate.
Year 1 Year 2
Principle $826 $909
Interest $174 $91 $165
Total $1,000 $1,000
d. Although the reported income in the same in both cases, in the first case of an annuity, you are able to
postpone paying taxes. Annuities are also even payments of income (simplicity).
6. Life annuity: × ℎ
=

a. | 28

i. Die on time: The formula works perfectly if you fully recover your basis and no more by the
time you die (i.e., if you die “on time”). Just recovered the last bit of premium the day you
die.
29
ii. Die later : If you die later than the table predicts, you get realize a “mortality gain” which
is fully taxable since you have already recovered your basis under 72(b)(2).
1. After you hit your life expectancy, we no longer exclude any portion of the payout.
The entire payout is taxable after your reach your life expectancy.
iii. Die earlier: If you die earlier than the table predicts, you suffer a “mortality loss” and
§72(b)(3) allows a deduction if payout stop because of your death and you have not recovered
your basis/investment in the contract.
1. Deduction is offered.
b. HYPO: TP is 75 years old. He purchases a life annuity for $50,000 with a payout of $5,000/year for
life. The payments begin immediately. How much of the payout is taxable?
1. Life Expectancy: 12.5 years (IRS table 5 – pp. 1048)
2. ^_`abca @aechC noigck`k }

3. deCfg @hCbibjfC`k l`i`bjCm = @hhcfg ^fpa`hC l`i`bq`k

$uw,www o

Av.u $u,www = $u,www


← ( ) = $4,000

← Taxable = $1,000 (must report as taxable income)


← Not taxable = $4,000.
← Employer-provided annuity (§72(c) and §72(f))
← If your employer pays a portion of the premium and the portion they pay is a tax-free
fringe benefit.
← it doesn’t count towards the “numerator” (premium30) on the
left side for your tax responsibility
← HYPO: TP is 75 years old. If TP pays $40,000 towards his annuity and his ER pays
$10,000 towards his annuity, how much of the payout is taxed?
← If the $10,000 provided by the ER is a fringe benefit, then:
1. $•w,www o
=
Av.u $u,www $u,www

← ( ) = $3,200

← Taxable = $1,800

p.1049

Before §72(b) there was no mortality loss or gain. Therefore, if you die early you lose. And if you die late
you win. It’s a game of chance. This instability was removed by §72(b). Downside: If you think about this
practically, the person living longer is losing out! The amount they are paid out drops the year they exceed
their life expectancy because the full amount is now taxed.

Must be your own after tax income.
21
WNM
Tax 1 Ring (Fall 2017)


Deferred annuity31
← §72(e): to the extent that there is a growth in the premium the interest is
tax-free, but any of that interest that is pulled out of the annuity
prematurely is considered to be taxable.
← HYPO: TP pays $100,000 in 2017 for annuity payouts to start in 2037.
Through investment by the insurance company, the annuity has increased to
$110,000. Insurance company allows A to withdraw $10,000 from it.
← The $10,000 is not a recovery of capital, so it is fully taxed. If A
withdrew $11,000, $10,000 would be taxed, but the additional $1,000
would be tax-free because it is a return on your investment.

Life Insurance
§§101(a),(g), 79(a)
Life Insurance
LIFE INSURANCE DEFINED: 6
1. Life Insurance: 5

Premium ($)
a. The owner of a life insurance policy makes an investment in 4
the life insurance contract by making payments of 3
premiums. 2
b. Two types of insurance: 1
i. Term Life Insurance 0
ii. Whole Life Insurance Y1 Y2 Y3 Y4 Y5 Y6 Y7 Y8
2. Players: Years
a. Holder/Owner of the contract Term Life Whole Life
b. Insured (the person whose life is at stake)
i. Often the owner and the insured are the same
Figure 1: TP dies at Y8. TP’s term ends at Y3.
person.
← Beneficiary (the person who will
collect on the death of the insured)
← Term Life Insurance32 (“Bet”)
← Coverage for # of years. Payment increases with age (risk of death).
← The TP is making an annual bet with the life insurance company as
to whether the insured will survive the period of insurance
protection.
← If the owner of the policy dies, he “wins” the wager. His
beneficiaries will receive a tax-free payment upon his death
that is far in excess of the cost of the policy.
65 Dies  WINS
← If the owner of the policy lives, he “loses” the wager. He will
have lost the premium that he paid for the insurance
protection in that period.
65 Lives  LOSES
66 He might technically “lose” the wager, but he’s
probably pretty happy because he didn’t die and he
had the peace of mind that comes along with
having had insurance.
← The insurance company groups customers by age (risk of death)
because the lower your age, the less likely the insurance company will
have to pay out on the policy. Thus, premiums tend to be lower for
younger people and higher for old people for this reason.
← In calculating TP’s policy payments, the company figures out how
many of their customers are in TP’s age pool, how many are likely to
die that year, and how much money is needed to cover the payouts.
← TP’s policy payments increase as he ages because the risk of
death associated with age increases.


Deferred annuities were a way for insurance companies to market annuities beyond their intended
purpose. TP began using their annuities as tax-free investment vehicles.

Insurance Co. Perspective: co. pools 100 people all age 50. They the co. needs to figure out
how much of a premium to charge those 100 people. To do so, the co. needs to predict how many
people in the group is likely to actually die that year and then collect premiums from each of the 100
to cover payouts for those people.
22
WNM
Tax 1 Ring (Fall 2017)

← Whole Life Insurance33 (“Bet + Savings Account”)


← Coverage lasts until death of the TP. Flat payment (overpay while young 
underpay when old).
← TP’s premiums are locked in so he pays the same amount in premiums
throughout his entire life.
← Premiums still vary person-to-person based on age
← In the early years, the premium TP pays is higher than necessary to
cover the insurance company’s anticipated payouts for the group of
people in TP’s age bracket.
← The insurance company takes the “excess” premium (the
amount above an equivalent term life premium) and invests
it (premium begins to earn interest)
← In later years, the premium TP pays is lower than what is necessary to
cover the insurance company’s anticipated payouts for the group of people
in TP’s age bracket.
← Whole Life Insurance is a better savings vehicle than a traditional bank account
because the interest that the life insurance policy earns is not taxable!

TAXING LIFE INSURANCE:


← §101(a): payouts on a life insurance policy are excluded from the gross income of the
beneficiaries if the payout happens because the insured person died
← Policy: want to encourage TP to plan for his death because it’s expensive for the
people he leaves behind. The optics of taxing beneficiaries (widows and orphans of
breadwinner) is not very politically appealing.
← HYPO: A gets a life insurance policy on his own life and pays $5,000/years in
premiums. A dies two years later. His wife is the beneficiary and collects the money.
← Taxpayer’s tax treatment: no effect
← Beneficiary’s tax treatment: wife collects the money tax-free under
§101(a) because there was no transfer of the policy for valuable
consideration
← §101(a)(2): owner of the life insurance policy can sell (transfer) the life insurance
contract for valuable consideration
← When it gets paid out to the beneficiary of the policy, it is not taxed as income
provided the amount was paid into the contract already  treated like a regular
asset
← HYPO: A buys an insurance policy for $2,000. He then transfers the
policy to B. B pays an additional $1,000 in premiums. The only portion of
the payout that is tax free is the $2,000 PLUS any additional premium
payments made on the policy ($1,000).
← Basis: $2000 + $1000 = $3000  not taxed.
← §101(a)(2)(b): if you sell the policy to the insured, a partner of the insured, to a
partnership in which the insured is a partner, or corporation in which you are a
shareholder for valuable consideration, the whole payout is tax-free
← Policy: in generally, we don’t like to see life insurance policies sold
willy-nilly. However, was are OK with a sale to a partner.
← HYPO: TP gets a life insurance policy on his own life and pays
$5,000/year in premiums. Taxpayer transfers the policy for $15,000 to a
business partner. Partner continues to pay the premiums for another year
and names himself as the beneficiary. Taxpayer dies. Partner collects.
← Taxpayer’s tax treatment: no effect while he owned the policy,
but the $15,000 on the sale is taxable.
← Beneficiary’s tax treatment: Tax-free under §101(a)(2)(b).
← HYPO: A has a policy on his own life. A sells the policy for $15,000 to B, a co-
shareholder, in his small business. B then names himself the beneficiary and
continues to pay the premium for the next 3 years until A dies.
← Beneficiary’s tax treatment: co-shareholder does not fit neatly into the
categories in the statute. But an argument could be made that the co-
shareholder in the small business was really a partner.
← There is a potential tension between the rationale behind
the statute and the language of the statute.

← Solves the business model problem.


23
WNM
Tax 1 Ring (Fall 2017)

← HYPO: Harry has a life insurance policy on his whole life. He sells it to his next
door neighbor so that Harry can pay for a trip to Disneyland. When Harry dies, what
happens to the next door neighbor in terms of tax?
← Beneficiary’s tax treatment: follow 101(a)(2).
← Treated like an asset taxed on GI = AR – B.
← Taxpayer’s tax treatment: Harry sold an asset. He will be taxed on the
$ that he sold that asset for. (AR – B (premiums Harry has put in so far)
= GI)
← Why can’t Harry turn to 101(a)?
1. Because the payout to Harry is not by the death of the
insured. Harry was still standing when he sold the
policy.
← HYPO: B buys life insurance on his own life. He then transfers the policy (K) to
his daughter, BD. She transfers $45,000 in cash to B in return. BD names herself
the beneficiary. B dies right after the transfer and BD collects $100,000 on the
policy.
← Tax Treatment:
← 101(a)(2):
1. B: selling an asset (AR – B = $45,000 – B = GI)
2. BD: AR – B = $100,000 - $45,000 = $55,000 = GI
← 102 (Gifts): She paid for it so it’s hard to argue that this was a tax-
free gift!
← 102(a)(2)(b):
1. Could try to argue this. Unless she was a business
partner, it is unlikely. Consider the context.
← Why not avoid this problem, by naming BD the beneficiary and
the BD transfers $45,000? Well, it’s possible. But be careful that it
doesn’t look qui pro quo.
← Sale of insurance policy
← Seller: AR (sale price) – B (purchase price you paid + premium payments) = GI
(taxable)
← Buyer: purchase price is buyer’s basis; later, basis will be purchase price + premium
payments
← Accelerated death benefits
← §101(g): “accelerated death benefits” allow terminally or chronically ill TPs to
turn in their life insurance policy back to the insurance company or sell it to a
viatical settlement provider34 to get money to pay for medical treatment before
death.
← Terminally ill: must be certified by a physician as having an illness or
condition reasonably expected to die within 24 months
← Chronically ill: §7702B(c)(2) provides a complex definition of chronically
ill
← Taxation:
← §101(g)(1): (Cash out) If TP is terminally or chronically ill and cashes out
his policy to pay medical bills and other expenses while he is still alive 
NOT TAXABLE
← §101(g)(2): (Sell) Proceeds from the sale of a policy of terminally
ill TP to a viatical settlement provider regularly engaged in the
business  NOT TAXABLE
← HYPO: J is terminally ill. She sells her life insurance policy to pay for end of life
care.
← Under §101(g)(2), the amount she gets for the sale is tax-free!
← Purchased by employer:
← §79: to the extent that TP’s employer pays the premium on a policy worth up to
$50,000, it is tax free. If the policy is worth more, the first $50,000 of the payout is
tax-free and the rest is taxable.
← HYPO: ER purchases insurance on behalf of EE for $30,000. Taxable
Income = $0.
← HYPO: ER purchases insurance on behalf of EE for $80,000. Taxable
Income = $30,000.

←Viatical service provider = company in the business of buying life insurance policies. Policies must
be sold back to the insurance company or a viatical settlement provider because Congress didn’t want the
terminally ill to be taken advantage of and forced to sell their policies for less than their worth. Must be
registered.
24
WNM
Tax 1 Ring (Fall 2017)

Realization
The Realization Requirement
§§109, 1019 “to recognize or not to recognize”
← Realization (non-recognition35 on event): annual increase in the value of property is not
“income” until the event on which the gain is “realized” by having been severed from the
underlying investment
← Appreciation of an item’s value isn’t taxable until the “realization moment” (i.e.,
when the taxpayer sells, transfers, or otherwise disposes of it)
36 37
← Eisner v. Macomber : The distribution of a stock dividend , where a shareholder
received no actual cash or other property and retained the same proportionate share
of ownership of the corporation as was held prior to the dividend, is not taxable
38
income to the shareholder within the meaning of the Sixteenth Amendment . The
annual increase in value of property is not “income” until the gain is “realized”
(i.e. until it is severed from the underlying investment)
← Stock dividend takes nothing from the property of the corporation,
and adds nothing to the interests of the shareholders, therefore
taxing it would be unconstitutional.
← SH’s stake in the corporation (% of the pie) remains the same.
ii. Scenario 1: Co. can retain $  Not Taxed
–––––––––––––––––––––––––––––– dissents place the Constitutional line here
––––––––––––––––––––––
iii. Scenario 2: Stock dividends issued  Not Taxed
--------------------------------------------- majority places the Constitutional line here
-------------------------------
← Scenario 3: Option of cash or stock  Taxed
← Scenario 4: Cash is issued  Taxed
← HYPO: In Year 1, 5 shareholders each invest $10 into a corporation in
exchange for 10 shares of stock each. ($1/share). In Year 2, the corporation
does business and earns $950.
← Each shareholder now owns $200 in the corporation.
← In Year 3, the corporation decides to pay a stock dividend of 5 more shares
per shareholder. Each shareholder now has 15 shares.
Year 1 Year 2 Year 3
$ the Co. has $50 $950 + $50 = $1000
Pie breakdown $1/share $20/share $13.33/share
10 shares/SH 10 shares/SH 15 shares/SH
$10/SH $200/SH $200/SH

Not the same as exemption:
← Exemption: never question  something you will never be taxed on
← Non-recognition: deferral, we will pick it up later

Macomber is the only decision holding that an income tax statute enacted by Congress is
th
unconstitutional under the 16 Amendment.
← Facts: Macomber owns 2200 shares in a corporation. The co. issues a 50% stock dividend
(1100 additional shares for Macomber).
← Difference between capital and income
← Income is a gain derived from
capital, labor, or both o Capital is the
tree; income is the fruit.
← Ask: Has the taxpayer severed income from the tree/derived a benefit/realized
anything from the capital?
← Stock dividends are still apples on the tree; still pieces of paper representing
a claim that exists inside the corporation. Stock is just a representation of a
right to pluck money from the
corporation.
3 Equally divided dividends does not increase what you have in the company
Corporation = Pie
0 You have a piece of the pie (1/4); if that 1/4 gets sliced into more and more pieces (2/8,
4/16) not taxed
1 Pro rata does not change the proportion you have (Just gives the SH more paper
to signal the same interest)
Note: Companies usually give out stock dividends when they have liquidity/valuation problems.
Towne v. Eisner, a predecessor case ruled that stock dividends were not taxable. Congress then amended
the equivalent of §61 that was on the books at the time to rule make stock dividends taxable.

25
WNM
Tax 1 Ring (Fall 2017)

Helvering v. Bruun39: Under the section 22(a) of the Revenue Act of 1932, LL realizes a
taxable gain on property, the value of which has increased because it was improved
by a tenant.
§109 (Overruled Bruun): GI does not include income (other than rent) derived by a
lessor of real property on the termination of a lease attributable to buildings or
other improvements constructed by the lessee
§1019 (Overruled Bruun): the owner’s basis in the property doesn’t change due to
improvements made by someone else (§1019). Therefore, he is taxed on the
improvements.
0 However, under §1014 when TP dies, his property passes to his heirs at a
40
basis of the FMV of the property.

Benefit of Deferral
Deferral is the situation in which a taxpayer is able to defer paying tax on an item of income
Present Value = 2 different approaches:
0 Today TP has $98. At an interest rate of 2% in one year it will be worth $100.
0.0 Amount Today = $98
0.1 Interest Rate = 2%
0.2 Number of Years into the future = 1 year
0.3 Total Amount at future date = $100
1 I have a bill that costs $100 that will be due in one year. How much $ do I have to set
aside today such that I will have $100 in 1 year if the interest rate is 2%?
1.0 Amount Today = $98
Deferral has the effect of reporting income and then receiving an interest free loan from the
government to pay taxes on that income at a later point in time.
0 HYPO: TP has $50 income and owes $20 in taxes. However, a special rule
allows him to defer paying that tax.
0.0 So at the end of Y2 he will have to pay $20 but he pays nothing at the end
of Y1. So he has that $20 to spend in Y1  interest free loan from the
government.
0.0 The $20 is money I owe the government in taxes, but they are
letting me keep it all year (put it in the bank and let it collect
interest). At the end of the year, I’ll take out the $20 and pay my
taxes.
The deferral of tax on unrealized appreciation creates a tax advantage for investments
in relative to investments that return income in the form of current payment, such as
interest and dividends.
0 TP pays less tax on investments that are not taxed until they are realized.
1 By delaying paying taxes, TP pays less tax over all.
1.0 Therefore, exemptions and deferrals are the most beneficial for taxpayers.

Heverling v. Bruun:
0 Facts: LL repossessed land from T who had defaulted on lease. During the lease, T had torn down
an old building (w/ an AB = $12,811.43) and had built a new one (Value = $64,245.68). The lease
had specified that the LL was not required to compensate the T for these improvements. The
government argued that upon repossession, the LL realized a gain of $51,434.25. The LL argued
that there was no realization of the property because no transaction had occurred, and because
the improvement of the property that created the gain was not "severable" from the landlord's
original capital.
1 Holding: The Court did not reject the realization requirement. However, it ruled that taking
possession of the new building was a significant, discrete, and thus taxable event.
HYPO: TP buys a painting for $5 from an unknown artist. 50 years later the artist becomes recognized as
the best artist ever and the value of the painting increases to $15,000,000. You die and the value passes
to your kids for $15M tax free for both parties.
0 The concept of realization opens up for all sort so planning possibilities when combined with other
rules.
1 Cornerstone of tax planning.
26
WNM

Tax 1 Ring (Fall 2017)

5. HYPO: A earned $100 in wages in Year 1 and invests her after-tax wages in a savings account paying 10% interest. The savings account is closed after Y3 and the

proceeds are used for consumption (A takes B to a fancy dinner).


a. Income Tax World: Tax Rate = 50%

Year 1 Year 2 Year 3

Pre-Tax Receipt $100 $50 +(.10)($50) = $55 $52.5+(.10)($52.5)=$57.75

Tax $50 $2.50 (TP earned $5 in interest $2.63(TP earned 5.25 in interest taxed

which was taxed at 50%) at 50%)

After-Tax Take Home $50 $52.5 $55.13

b. Exemptions World: Savings accounts are not taxed, thus A is allowed to exempt the interest earned on the savings account.

Year 1 Year 2 Year 3

Pre-Tax Receipt $100 $50 +(.10)($50) = $55 $60.5

Tax $50 $0 (b/c it’s exempt!) $0

After-Tax Take Home $50 $55 $60.5


c. Deferral World: A can defer the taxes on the wages in Year 1 until Year 2, but the interest on the

savings account is still taxable.

Year 1 Year 2 Year 3

Pre-Tax Receipt $100 $100 +(.10)($100) = $110 $121


Tax $0 (income tax $55 (Interest and the $60.5
deferred) principle on the savings

account are taxable!)

After-Tax Take Home $100 $55 $60.5

The general principle illustrated by this example is that given a fixed rate of taxation, deferring the tax on an item of income is equivalent in effect to taxing that item and then

exempting from tax the income produced by investment of the after-tax proceeds.

6. HYPO: Suppose A gets a bonus of $100,000. He wants to invest the bonus and then live off of the after-tax earnings. Tax rate = 50%. Savings Account Rate = 10%
a. Scenario 1: Tax the bonus but exempt the earnings.

Year 1 Year 2 Year 3

Pre-Tax Receipt $100,000 $50,000 +(.10)($50,000) = $55,000 $55,000+(.10)($55,000)=$60,500

Tax $50,000 $0 (b/c interest income is not taxed) $0

After-Tax Take Home $50,000 $55,000 $60,500

b. Scenario 2: Defer taxes on the bonus until consumption when the entire amount that is pulled out is taxed.

Year 1 Year 2 Year 3


Pre-Tax Receipt $100,000 $100,000 +(.10)($100,000) = $110,000+(.10)($110.00)=$121,000

$110,000

Tax $0 $0 $60,500

After-Tax Take Home $100,000 $110,000 (deferred again) $60,500


27
WNM
Tax 1 Ring (Fall 2017)

Discount Obligations
§§1272(a)(1),(3), 1273(a)(1), 1276(a)(1), (b)(1), (b)(2)(A), 1278(b)(1)

Regular Bond
41
Formalized loan; extension of credit. Investment.
You loan them money; they owe you interest every year, then pay off the bond at the end.
0 Ex] TP loans Co. $1,000. Co. agrees to pay A back at 10% interest per year for 5
years. At the end of the 5-year term, A gets back his $1,000 plus interest at
10%/year.

Original Issue Discount Bond (OID)


Original Issue Discount Bond (OID): A bond issued by a company to an investor (TP) at a
discount (for less than the final payout).
0 A bond issued for less than the maturity value; debtor isn’t going to pay you
anything in the interim but pays all the interest at the end along with the principle.
0.0 Issued the instrument for an amount ($100) less than what the borrower
must pay at maturity ($133) . No interest payments are made to the lender
in between.
0.0 The interest is the time value of money.
42
Taxing OIDs:
0 §1272(a)(1): include in GI an amount equal to the sum of the daily portions of the
OID for each day during the taxable year on which the holder held such debt
instrument
1 Answers the question of how to tax the discount (the gap between what was
paid and what the taxpayer gets)
2 Determining the daily portions:
2.0 Determine the debt instrument’s yield to maturity (10%)
2.1 Determine the accrual period (Years 1, 2, and 3)
2.2 Determine the OID allocable to each accrual period
HYPO: Zero Coupon Bond. A loans Co. B $100 with the promise that at the end of 3 years,
B will pay A $133.
0 AR – B = $133 - $100 = $33 = Income
1 OID = $33
43
2 Yield to Maturity ≈ 10%

Regular Bond: You give $ to the borrower up front. The borrower pays you interest every year and at the
end of the term, the borrower pays you back.
0 Specified rate of return
1 Specific # of years

Interest income is the fee you are collecting from the borrower for the use of your $ for a certain period of
time. It is taxed.

History of Taxing OID:

Excessively generous to the TP


Contrast with stock: Stronger than appreciated stock because you are more likely to
get paid back. Method 2: 1969-84 – pro rata method.
Y1 = $11; Y2 = $11; Y3 = $11
Note: Borrower is taking a deduction of $11/year
Realization problem:
0 Liquidity issues  TP is being taxed on $ that he hasn’t yet received.
Congress felt that OIDs were enough like regular bonds to be taxed like regular bonds.
Economically, the $ is not being earned at $11/year.
Overstates interest in the early years! Claims $11 but the TP only earned $10. Accelerating the taxation.
Corporations taking deductions were very happy with this pro-rata approach. $11 in Y1 overstated
the interest deduction (accelerating). But as it turns out the people lending in this case were
university (non-taxpayers). So the government was losing.
28
WNM
Tax 1 Ring (Fall 2017)

Accrual Period OID/Accrual Period Basis


Year 1 $100 ´ .10 = $10 $110
Year 2 $110 ´ .10 = $11 $121
Year 3 $121 ´ .10 = $12 $133
Total $33 $133
NOTE: Unlike stock (which we noted in Eisner are not taxed until
appreciation), bonds are taxed before their value is realized.
EXAM TIP: Memorize this Example!
Explain the difference between RB and OID.
• Regular Bond: TP loans a corporation some money (let’s say $ 1000). The corporation takes the
money and agrees to pay the TP interest annually at a certain rate (let’s say 10%) for a specified
number of years (let’s say 10 years). At the end of those 10 years, the corporation must pay back
the $1,000. At the end of the 10-year term, TP has gotten back his $1,000 plus interest at
10%/year.
• Original Discount Bond: A bond issued by a company to an investor (TP) at a discount (for less
than the final payout) . TP loans a corporation some money (let’s say $100). The corporation takes
the money and agrees to pay $133 at the end of a specified number of years (let’s say 3 years). So,
at the end of the 3-year term, TP will get $133 back from the corporation. TP will not receive any
money from the corporation in the form of interest payments at the end of years 1 and 2. The
entire $133 payment is made at the end of the 3-year term.
41
Explain why the tax system moved from method 1 to 2.
• Under method 1, TP was only taxed once at the end of the 3-year term. But Congress believed
this method was far too generous to the TP.
o TP was very likely to get paid back at end of the 3-year term. We tax appreciation in stock
value, so it doesn’t make sense not to tax “appreciation” in these bonds as well.
o Congress believed that OIDs, were enough like RBs to be taxed like RBs.
• Under method 2, TP is taxed ratably.
Method 3: Yield to Maturity:
Investor lends $100 to corp. Corp has that $100 to use all year. Interest = 10% ($10). But the investor
doesn’t get that $10. They let the corp. keep the $10. So now the corp. has $110 for year 2! So at
the end of Y2 you owe me $11 (10% of $100). Going in year 3, you have $121 of my $$s and
you’re using it all year. So at the end you owe me $12 (10% of $121). So at the end of year three
you owe me.
($10+$11+$12 = $33) + $100 (principal) = $133.
You have nothing in your hand at the end of Y1. But we still tax you on $10 in Y1 even
though you got nothing. Y2 we tax you on $11 even though you got nothing.

29
WNM

@–—

$Atw

Tax 1 Ring (Fall 2017)

Market Discount (MD) Bond:


Market Discount (MD) Bond:
0 A buys a regular bond at a certain interest rate (10%). Before the end of the term A
sells the bond to another person, B, in the secondary market at a discount that
reflects the current market rates. When it is resold, it becomes as market discount
bond.
0.0 The effect of the discount the new bondholder gets equals the rate of
return (percentage) in the new market (e.g., 12%)44
0.1 The “discount” does not exist at issuance. It is created when the bond is
sold.
1 Rules:
1.0 If the second purchaser holds the bond to redemption, the discount piece is
not taxed until the end  at the end, it’s taxed as ordinary income
1.1 If the second purchaser doesn’t hold the bond to redemption (i.e., sells it
early), they have to calculate the accrued market discount as ordinary
income to the extent of the gain on the sale.
1.0 AMD = the portion of the discount that has been earned since the
time the second
bondholder acquired it to the time they sold it
2. #

= # ’

0.0 Doesn’t matter when the first bondholder bought it  only


matters when the specific bondholder whose AMD you’re
trying to calculate bought it
1.1 Gain on the sale of the MD bond is taxable as ordinary income to
the extent of AMD, but the AMD is a step, not a taxable moment.
Taxing MD Bonds: §1276(a)(1)
0 HYPO: Bond is issued at a rate of 10% for $1,000. When there are 10 years left on
the bond, it is sold to Harry for $870 who holds it for the rest of the term and then
cashes it out.
i. How much will Harry be taxed?
0.0 Harry will be taxed on the $100 that he receives each year.
0.1 Harry not going to be taxed on the full $1000, but Harry will be
taxed on the $130.
1.0 MD = $1,000 - $870 = $130
ii. When will Harry be taxed?
1. At the end of the bond term as ordinary income!
b. HYPO: Ratable Method. Bond is issued at a rate of 10% for $1,000. When there
are 10 years left on the bond, it is sold to Harry for $870 who holds it for 5 years,
then sells it to Carl for $900.
i. How much will Harry be taxed?
1. Harry will be taxed on the $100/year.
2. Harry will not be taxed on the full $900, but he will be taxed on:
a. MD = $1,000 - $870 = $130
b. = Awu = $65

c. $ = |

d. Harry is taxed on the gain from the sale of the MD bond as


ordinary income UP TO THE AMOUNT of the AMD 
all $30 are taxable an ordinary income.

In other words, in the current market if I were to go out and buy an identical bond (same quality
borrower on the same terms) I would be able to demand 12% interest.
0 Suppose I want to sell my bond now. Can I sell it for $1000?
Probably not! No one wants to buy the bond for $1000 because they can spend $1000 and get
a bond with
12% interest!
But if I lower my price, I can sell the bond!
0 I sell the bond for $870.
0.0 The new bondholder gets $100/year and gets $1000 at the end of the
term.
0.1 $100(# of years) + discount = 12% return on the
purchase price of $870. o The $130 is INTEREST!
If the interest is now 8% people will be lining up outside the door to have me sell it for $1000. But I wont
sell. I will only sell for what is = to an 8% return on their payment.

30
WNM
Tax 1 Ring (Fall 2017)

i. So in this case, he could have been taxed up to $65.


ii. When will Harry by taxed?
1. When he sells the bond to Carl.
c. HYPO: Bond is issued at a rate of 10% for $1,000. When there are 10 years left on the
bond, it is sold
to Harry for $870 who holds it for 5 years, then sells it for $800.
i. @–—
= u = $65
$Atw Aw
ii. $800 − $870 = −$ = |

iii. Harry is not taxed on the bond and can report a loss of $70 on the sale.
d. HYPO: Bond is issued at a rate of 10% for $1,000. When there are 10 years left on the
bond, it is sold
to Harry for $870 who holds it for 5 years, then sells it for $950.
i. @–—
= u = 65
$Atw Aw
ii. $80= |

iii. Harry is taxed on the gain from the sale of the MD bond as ordinary
income UP TO THE AMOUNT of the AMD  65 is taxable as ordinary
income, $15 is treated as a gain on the sale of a bond
1. Could this be Capital Gains?!

HYPO: Ford Motor Co. issues a bond for $5,000 to Jack, which pays 10% interest annually, and at the
end of 20 years pays $5,000 to the holder of the bond. After 10 years, Jack sells the bond to Marco for
$4,000. Marco after 5 years, sells the bond to Liz for $3,500. Ho are Jack, Marco, and Liz treated for
tax purposes.
Jack will be taxed on the $500 he receives every year in interest from the corporation for the 10 years he
holds the bond before selling it to Marco.

($5,000 x 10%) = $500 $500 x 10 years = $5,000 in taxable interest income

Jack also gets a loss on the sale of the bond for $1,000.

AR–B=GI
$4,000 - $5,000 = GI
$1,000 = LOSS
Marco will be taxed on the $ 500 she receives every year in interest from the corporation for the 5 years
he holds the bond before selling it to Liz.

($5,000 x 10%) = $500


$500 x 5 years = $2,500 in taxable interest income

Marco will also be taxed on his gain from the sale of the MD Bond as ordinary income UP TO THE AMOUNT
of the Accrued Market Discount. However, in this case, Marco sells the bond at a LOSS! Therefore,
Marco is not taxed on the bond and can report a loss of $500 on the sale.

MD = How much the bond pays out at the end of the term – How much the holder paid for the bond in the
secondary market

MD = $5,000 - $4,000 = $1,000


5
= = $500
$1,000 10

AR–B=GI
$3,500 - $4,000 = GI
-$500 = LOSS
Liz will be taxed on the $500 she receives every year in interest from the corporation for the 5 years she
holds the bond before cashing it out.

($5,000 x 10%) = $500 $500 x 5 years = $2,500 in taxable interest income

Liz will not be taxed on the full $5,000 payment when she cashes out the bond. Instead, she will be taxed
on the Market Discount (MD) portion of the bond.

MD = How much the bond pays out at the end of the term – How much the holder paid for the bond in
the secondary market

MD = $5,000 - $3,500 = $1,500


31
WNM
Tax 1 Ring (Fall 2017)

Capital Gains
§§1221, 1221, §1(h)

CAPITAL GAINS DEFINED


One of the big “hot button” issues in tax policy.
The IRC taxes taxes capital gains at a special capital gains tax rate, that is lower than the
ordinary income tax rate.
0 Rationale: If you own certain kinds of property and you sell that property you
should have to pay a lower tax rate on the gains.
1 Ex] §1221
Capital gains taxes are only triggered when an asset is realized, not while it is held by an
investor. That means he can own stock shares, for example, that appreciate every year, but
does not owe a capital gains tax on the shares until he sells them, no matter how long they're
held.
The most common capital gains are realized from the sale of stocks, bonds, precious metals and
property.
Downside:
0 If you sell at a loss then there might be some disadvantages
1 Overall, these are still very attractive.

TAXING CAPITAL GAINS


§1221: if an asset is a capital asset (generally an investment asset), then KG (capital gains) may
get a lower tax rate and KL (capital losses) are restricted in how the taxpayer uses them
0 Rationale (highly contested)  issue of realization:
0.0 An asset that appreciates in value isn’t taxed until the point of sale and
the taxpayer alone decides when to sell
0.1 Taxpayer is willing to sell any investment that has gone down in value, so
regardless of when you sell, you’ve lost the money, but selling it triggers a
loss on the tax return
0.2 Taxpayer will hold off on selling the investments that go up in value
0.3 Congress reduced the tax rate on certain assets to incentivize sale (because
that’s the taxable moment) and reduce the stagnation caused by the “lock-
in” effect
HYPO: Several years ago, TP earned salary and paid taxes on it. Until now, TP has been keeping
the after-tax money under her mattress, but has now decided to invest it. TP takes $100 of the
after-tax money and purchases an investment asset for $100 at the end of Year 1. During Year
2, the asset grows in value (the before tax rate of return on the investment is 10%). At the
end of Year 2, TP sells the asset for $110. During Years 1 and 2, the regular tax rate is 50%.
0 Income Tax World
Year 1 Year 2 Profit Rate of Profit
(Investment) (Disinvestment)
Pre-Tax cost of the $100 $110 (Because the $10 10%
investment (purchase investment rate is
price) 10%)
Tax $0 She’s not being $5 (50% of the gain
taxed here because on the asset) You’re
she’s already been being taxed on the
taxed on the salary $10 of income you
which she’s had made @ the 50% tax
under her bed. the rate on investment
act of buying does income.
not have an income
tax effect
After-Tax Cost $100 $105 $5 5%

32
WNM
Tax 1 Ring (Fall 2017)

Deferral World (Allowing TP to collect Basis before the sale of the Asset)
0 Deferral can also be achieved by accelerating the TP’s recovery of the initial
investment –– i.e. allowing the TP to recover basis before the asset has been
sold/exchanges.
Year 1 Year 2 Profit Rate of Profit
(Investment) (Disinvestment)
Pre-Tax cost of the $100 $110 $10 10%
investment (purchase
price)
Tax (savings) ($50) what you $55 (50% of the gain
would have paid in on the asset)
taxes without the
deduction.
After-Tax Cost $50 $55 $5 10%

c. Capital Gains Reduced Rate w/ No Deferral (KG Rate = 20%)


Year 1 Year 2 Profit Rate of Profit
(Investment) (Disinvestment)
Pre-Tax cost of the $100 $110 $10 10%
investment (purchase
price)
Tax $0 $2
After-Tax Cost $100 $108 $8 8%
AR–B=GI
$110 - $100 = $10
$10(.20) = $2 in taxes
owed
$110-$2 = $108

d. Deferral World (Capital Gains Reduced Rate w/ Deferral)


Year 1 Year 2 (Disinvestment) Profit Rate of Profit
(Investment)
Pre-Tax cost of the $100 $110 $10 10%
investment (purchase
price)
Tax (savings) ($50) (what you $22
would have paid
without the AR–B=GI
deduction; You $110 - $0 = $110
won’t be including $110(.20) = $22
this $50 in your $110-$22 = $88
income, so you can
invest it. But then
later you will be
taxed on the total
(principle + interest)
in Y2.)
After-Tax Cost $50 $88 $38 76%
Caused by a system with two tax rates:
0 Deduction = 50%
1 KG tax rate = 20%
33
WNM
Tax 1 Ring (Fall 2017)

Factors of Motivation
Windfalls 45
Windfall = an unanticipated benefit, not caused by any action taken by the recipient.
0 Cesarini: TP buys a piano and discovers $40,000 in cash inside of it46
1 HYPO:
1.0 Scenario 1: TP finds $1M in cash on the ground → TAXED (§61
“clear accession to wealth”)
1.1 Scenario 2: Ring buys an antique desk for $500. Her friend, an expert
in furniture, later informs her that the desk belonged to Abe Lincoln.
1.0 Does Ring suddenly have additional income?
0.0 No, we need to wait till she sells!
0.1 Yes, she’s richer because the desk is worth so much more!
1.2 Scenario 3: What if the desk contains gold nuggets?
2.0 IRS: Cesarini: cash equivalent found
2.1 TP: closer to oil, because you can’t take gold nuggets to the
store! Not quite as liquid. A next step is required. Cash is
instant income.
1.3 Scenario 4: What if the knobs on the desk are solid gold?
3.0 The value of the desk is more.
0.0 Part of the desk?
3.1 TP argues severability → desk NEEDS knobs. Unless and until
the TP removes the knobs and sells them or sells the desks he
cannot be taxed.
3.2 Now just one gold knob → valuation issue → VERY UNCLEAR
1.4 Scenario 5: Ring discovers oil under her house.
4.0 Requires a realization event  pulling oil out of ground,
selling it (and/or the property)
4.1 Distinguish from Cesarini → oil isn’t cash; severability
Taxed to various degrees, depending on the kind of windfall.
0 Lottery winnings or other cash windfalls are includable in GI under Glenshaw
Glass (“undeniable accessions to wealth”)
1 Other kinds of windfalls that are not immediately liquid in the same way as cash (e.g.,
oil found under your house, a doorknob in your house being made of solid gold) aren’t
GI until the realization event.

Gifts
§§102, 273, 274(b), 691(a)(1), 1014(a), 1015(a)&;
(e),1022, 1041(a)-(c); Treas. Reg. §§1.1001-1(e)(1);
1.1015-1(a)(2)

GIFTS DEFINED:
Gratuitous transfer
0 Ex] Ryan’s aunt gives him $100,000 in cash.
Act of generosity
Glenshaw Glass §61:
0 pretty broad
1 you’re a hell of a lot better off than before you found the $1M on the ground. There is no way to
peg the $1M as a return on your investment.

HYPO: You have a house. You’re digging a veggie garden. You find oil. Is this taxable?
Are you wealthier than before you found the oil? Ya!
So why not tax you as soon as you found the oil?
Liquidity problem!!!
If you were Cesarini’s lawyer how might you argue that the $ in the piano was different than the $ on the
street?
0 Could argue that C bought the $$$ when he paid for the piano.
34
WNM
Tax 1 Ring (Fall 2017)

TAXING GIFTS47:
§102(a): GI does NOT include income acquired as a gift, but imposes the tax burden on the giver
0 Rationale:
0.0 intra-family gifts generally do not generate additional economic income
0.1 gifts as imputed income –– intra-familial exchanges
0.2 encouraging generous behavior (wealthier members  less wealthy
members)
0.3 problems with taxing gifts:
3.0 valuation problems
3.1 administrability: awkwardness of asking the giver what the
FMV of the gift is (recordkeeping, documentation)
Limits on the exclusion:
0 §102(b)(1): income from property (gift, bequest, devise, or inheritance) is not
excluded from GI
0.0 ex] Interest, dividends
1 §102(b)(2): further denies the exclusion to gifts (either during life or at
death) of income from property (Irwin v. Gavit48)
2 HYPO:

3 Basic Ways to Tax Gifts:


0 Tax donor + donee
1 Donor: doesn’t get a deduction for giving a gift
2 Donee: gift included in income
Tax donee only [follow the $ approach]
0 Donor: deduction
1 Donee: only taxing where the $ lands after it moves through the system
Tax donor only
0 Donor: no deduction
Donee: not included in income
Notes:
Excludes from the income of the recipient (donee) of a gift the amount or value of the gift.
However, there is no such exclusion for the gift giver (donor).
Intra-family gifts do not generate additional economic income. Including gifts as part of gross income
would present significant valuation problems where gifts of property are involved and would
require distinguishing taxable gifts from tax-free support.
There is a gift tax, but that has nothing to do with whether gifts are income.
Irwin v. Gavit:
0 Facts: Grandfather left in his estate $ for his granddaughter. The father was allowed to collect
the interest income from the $ in the estate to use for the girl’s education until she turned 21
and could access the estate herself. The IRS determined that the interest was income to the
father. The Tax Court ruled in favor of the father that the interest was a gift and not income at
all.
1 Holding: the gift of interest from the fund was income to TP and thus should be taxed. This
was a gift of income, not a gift of property.
2 Notes: A  B (for x years)  Remainder to C
3 Is this a tax free gift? NO! The interest payment coming from the fund are taxed as
income.
Consider the Following:
If A had kept the property and simply said to B, “I will give you a gift of the income on this property
[which I, A, still own!!!] for five years,” then we would treat this transaction in the following
manner. We would say that A must report the income on that property during the five year
period, and then B is receiving a cash gift fully excludable as a gift under section 102. That is, we
would not see B as having a gift of an income interest that would be taxable. Why not? First, we
have a general doctrine we will explore more later, that bars taxpayers from trying to “assign”
their income to someone else to have that person report it. Thus, for example, if I promise my
niece that she can have half of my wages for 5 years, the tax system requires that I report the
wages first, and then I am viewed as making her a cash gift, which is a nontaxable gift of
property. We are worried about the abuse potential of letting a taxpayer divert streams of income
and try to get another taxpayer to have to report it as income, in cases where the original
taxpayer controlled the labor or retain the assets.
Second, in the classic” gift of an income interest” scenario [from A to B for 5 years, remainder to C],
because A is completely out of the picture, B is the most realistic person to have report the
income generated during the 5-year period – and we certainly don’t want it to go untaxed. In the
case where A retains the property, it is easier––and consistent with general ideas about not
letting you shift to obligation to report your income off onto someone else–– to have A first report
the income, and “then” make the gift to B. Of course if A has reported it as income, there is no
need, nor would it be appropriate to have B report it as income as well.

35
WNM
Tax 1 Ring (Fall 2017)

Scenario 1: A dies and leaves a trust with $100,000 to B.


0 Under §102(a), B has received a gift of property, so it is not taxable.
Scenario 2: A dies and leaves a trust with a corpus of $100K to B, under the
49
terms that B gets the interest income for 5 years with the remainder to
go to C.
0 B has received a “term interest” = the right to use (i.e., invest) the
$100,000, but not to spend it. B can keep the interest the trust
earns, but the interest is taxable.
0.0 Irvin and §102(b)
1 C gets to keep the $100,000 after 5 years and it is tax-free as a gift
of property under §102(a).
Scenario 3: A has $100,000 and gifts $10,000/year to her brother B for 5 years,
then the rest goes to A’s niece C.
0 Since A is holding the property, A has to report the $100,000 as
income. B and C’s gifts are tax-free under §102(a).

What is a “gift of income”? What is the classic example of a gift of income?


A gift of income is a transfer of property such as a trust that generates interest income for the donee.
The interest that the trust generates is income to the donee and is thus taxable. The money inside the
trust is a gift and is thus not taxable.

Appreciated and Depreciated Property:


0 §1015: Carryover Basis: When a gift is given, the donor’s basis becomes the
donee’s basis.
0.0 The basis of property received as a gift is generally equal to the donor’s
basis in the property. Therefore, if the donee subsequently sells the property
any appreciation in value of the property between the time the donor bought
the property and the time the donee sells it will be taxable.
0.1 Taft v. Bowers: When a gift is later sold, the taxable gain to the person who
received the gift is the difference in price between the sale price and the
original donor’s basis.50
1 Exceptions:
1.0 If the property has depreciated and sale of the property results in a loss, the
basis is the FMV of the gift.
0.0 Policy: prevents the donor from shifting an unrealized loss to the
donee.
0.1 If the gift depreciates in value and the donee later sells the gift at a
loss (donee’s sale price < FMV when donee acquired the gift) →
use FMV of the gift at time it was given as the donee’s basis
1.1 If donee’s sale price falls between the carryover basis and FMV when the
donee acquired the gift → NO GAIN OR LOSS (Treas. Reg. 1.1015-1(a)
(2))
2 HYPO: Basis Calculations:
2.0 Scenario 1: A buys Blackacre for $1,000, then gifts it to B, later,
when its value has depreciated to $800. B sells it for $700.

Remember: B can’t touch the principle of $100000 which will go to C after the 5 years. But B can eat
anything the $100000 accrues.
Taft v. Bowers
0 Facts: TP’s dad gave her shares of stock which were worth more when gifted than when they
were purchased by the dad. Later she sold them for more than their FMV when the gift was
made. The IRS wanted to tax the difference between the price at which TP sold the stock and
value of the stock when the father purchased them.
1 Issue: Can the IRS tax the increase in value of a gift after it has been give?
2 Holding: When a gift is sold, the taxable gain to the person who received the gift is the
difference in price between the sale price and the original donor’s basis.
HYPO:
1916: A purchased 100 shares of stock for $1000
1923: A gives the stock to B (now valued at $2000). B then sells the stock for $5000
IRS claims that B must pay income tax on $4000 which is the realized profit.
B claims he only needs to pay $3000
0 Only the appreciation during B’s ownership can be taxed. The appreciation during the A’s
(donor) ownership is not taxable income against B.
Outcome:
0 Sale price – Donor’s Basis = GI
$5000

$1000
= GI o
$4000
= GI
36
WNM
Tax 1 Ring (Fall 2017)
Donee’s sale price ($700) > FMV when donee acquired the gift ($800)
0 Therefore, the donee’s basis is the FMV of the gift when
the donee acquired it ($800).
1 The asset was a gift of depreciated property that continued
to depreciate.
AR–B=GI
$700 – $800 = -$100
iv. The donee is not taxed and gets a loss of $100.
0 Note: Donor’s $200 of loss disappears forever
Scenario 2: A buys property for $1,000, then gifts it to B, later, when
its value has depreciated to $800. B sells it for $5000.
0 Basis is $1,000 because the depreciation in value was made up for
in the sale price.
Scenario 3: A buys property for $1,000, then gifts it to B, after its value has
depreciated to $800. B sells it for $925.
0 Treas. Reg. 1.1015-1(a)(2) does not tell us how to calculate basis
when the sale price is in between the FMV when it was gifted and
the donor’s basis, but tells us that we do not get a gain or a loss.
HYPO: Mary gives her cousin a painting that she purchased several years ago for $3,000. At the time
Mary makes the gift, the painting is worth $6,000. What happens if the cousin sells the painting for
$5,000? $2,000? $7,000?
$5,000 $2,000 $7,000
The value of the gift appreciated between the time the donor purchased the AR–B=GI AR–B=GI
painting and the time the donor gifted the painting to the donee. $2,000 – $3,000 $7,000 – $3,000 =
= -$1,000 $4,000
Therefore, according to rule §1015, if the donee subsequently sells the property
any appreciation in value of the property between the time the donor bought Donee gets a loss Donee is taxed on
the property and the time the donee sells it will be taxable. (i.e., Donee Basis = of $1,000. his gain for $4,000.
Donor’s Basis = $3,000).

AR – B = GI = $5,000 – $3,000 = $2,000


Donee is taxed on his gain of $2,000.
HYPO: Millie gives her brother a painting that she purchased several years ago for $3,000. At the
time Millie makes the gift, the painting is worth $2,000. What happens if her brother sells the
painting for $5,000? $1,500? $2,200?
$5,000 $1,500 $2,200
Donor (M) Basis: $3,000 Donee (B) Sale Price: FMV @ Gifting < Donee (B) Sale Price < Donor (M)
FMV @ gifting (M  B): $2,000 (Dep.) $1,500 Basis
FMV @ gifting = $2,000 $2,000 < $2,200 < $3,000
Donee (B) Basis: $3,000 because the $1,500 < $2,000
depreciation in value was made up for Therefore, Donee (B) Basis Treas. Reg. 1.1015-11(a)(2) tell us that when the
in the sale price of $5,000. Therefore, = FMV @ gifting = $2,000 sale price is between the FMV @ gifting and the
the donor’s basis carries over the Donor (M) Basis, the Donee does not get a LOSS or
donee. AR–B=GI a GAIN.
$1,500 – $2,000 = $500
AR–B=GI B gets a LOSS of $500 Why? Neither the donee nor the donor has the
$5,000 – $3,000 = $2,000 opportunity to get a deduction for the depreciation
B will be taxed on $2,000 of GAIN in value while the donor held the property.
Therefore, it seems unfair to tax the donee’s gain.

Property acquired from a decedent (dead person):


0 §1014: Basis of property acquired from a decedent is the FMV of the
property at the date of the donor’s death
0.0 Rationale: Want to encourage people to only pass on appreciated
assets to their heirs and sell any depreciated assets before death so
you can report it as a loss on your last tax return.
0.1 HYPO: A buys property for $1,000, and the asset depreciates to
$900. If A gifts the property to A’s heirs on her death, their basis is
$900 and the heirs cannot deduct the $100 loss. If A sells the asset
before she dies, she can report the $100 loss on her last tax return.

37
WNM
Tax 1 Ring (Fall 2017)

1 “Step-up” in basis upon death: only fools die with depreciated assets.
That built in loss disappears. Hold appreciated property and sell depreciated
property so that you can collect on the tax return.
Holders of Life or Terminable Interest:
0 §273: amounts paid as income to the holder of a life or terminable interest
acquired by gift, behest, or inheritance shall not be reduced or diminished by
any deduction for shrinkage
51
0.0 Commissioner v. Early : because the original stock exchange
from decedent was a gift, the subsequent settlement for the life
estate was a gift, as well.
Marital Rights52:
0 §1041: property can be transferred tax-free between spouses or former
spouses (divorced; within one year of divorce) (§1041(a))
0.0 The transfer shall be treated as a gift
0.1 Carryover basis (§1041(b)) is used by the donee to calculate gain
when the “gift” is resold regardless of any
appreciation/depreciation in the FMV.
1 Transfer Before Marriage: [§1041 does not apply]
1.0 Farid53: TP paid valuable consideration for her shares.
Therefore, they were not a gift. They were an exchange.
Business Gift:
0 Putative gifts between partners in a business or commercial relationship 54.
1 Taxing Business Gifts:

Commissioner v. Early:
0 Facts: A wealthy woman gives her accountant interest in stock as a gift. She died a year later. Her
family, whom she hated, is not happy about this and they contest the will. The accountant settles
the dispute by giving the stock back in exchange for a life estate in the interest income. The
accountant sought to amortize the value of the life estate in subsequent tax returns but the IRS
wouldn’t let him.
o TP’s argument: TP thinks he should get a deduction because he has basis to recover
from purchasing the life interest in exchange for the stock her sold to the angry kids. This
was a taxable event; a sale; purchased income interest → has basis in the asset
equivalent to cost. As income is derived from the life estate he believes he should be able
to recover his basis.
o IRS’ argument: The IRS argues that the interest income is taxable income under
§102(b) because it is a gift of income.
1 Holding: because the original stock exchange from decedent was a gift, the subsequent
settlement for the life estate was a gift, as well.

HYPO: A owns Blackacre (Basis = $50. FMV = $400). B owns an airplane (Basis = $125. FMV = $400). A
and B exchange the land for the plane. Taxable event under §1001.
0 Taxation:
A: owns the land and now just sold it for the plane
0 AR – B = $400 - $50 = $350 = Gain
1 What is A’s basis in the airplane? $400
B: owns the plane and now just sold it for the land.
0 AR – B = $400 - $125 = $275 = Gain
1 What is B’s basis in the land? $400
Farid Es-Sultaneh v. Commissioner:
0 Facts: TP executed a pre-nuptial agreement with her soon-to-be husband. In the agreement, TP
“exchanged” her dower rights for shares in her fiancé’s corporation. The parties ultimately
divorced and the taxpayer received the shares in 1924 when the FMV was $10 each (but the
donor’s basis was 16¢). TP sold her shares for $19/share. Tax court said her basis in the shares
was 16¢.
o IRS: TP received the shares as a gift (16¢/share)
o TP: This is a sale. TP purchased the stock with her marital rights ($10/share)
1 Issue: what should TP’s basis be when she sells her stock?
2 Holding: TP wins! She gave fair consideration in relinquishing her marital rights which were
worth more than the stock. It was an exchange rather than a gift.
o FMV =
$10/share 
Basis o
$19/share =
$10/share =
Gain
3 Note: led to §1041: gifts between spouses

Consider former employees, independent contractors (legislative history is unclear)

38
WNM
Tax 1 Ring (Fall 2017)

0
Whether something is a gift is an issue of fact for the trial court to
determine  looks at whether the transferor’s intention
represents “detached and disinterested generosity”55 [Duberstein
56
and Stanton ]
0.0 Ex] payments to retired ministers are generally
nontaxable gifts in light of the close personal
relationship between the recipient and the congregation
0.1 Ex] payments by ER to EE upon termination  generally
taxable.
0.2 Ex] employment related payment during the holidays 
taxable.
0.3 Ex] ER gives EE turkey at Thanksgiving→ fails §102(c),
but probably a de minimis fringe
0.4 Ex] ER gives you a baby blanket → de minimis fringe
Employee Gifts:
57
0 §102(c): INCLUDE any amount transferred by an employer to,
or for the benefit of, an employee.
0.0 Family in Business Exception: exclude for GI gifts in the
employee-employer relationship when the parties are related
and the gift can be “substantially attributed to the familial
relationship,” not to the employment

1
There is no such thing as EE gifts! If you got it from your ER, it
is not a gift!
Employer deduction:
0 §274(b)59: Even if a business gift must be INCLUDED in the donee’s
income under §102(c), the business donor is limited to a $25
deduction for gifts to individuals.

No gift is ever truly detached or disinterested! We could interpret this as not expecting anything is return.
The Court is trying to create some kind of standard. It believes the IRS is being far too simplistic with it’s
“no business gifts!” rule. Not the common law meaning of gift, more casual, colloquial, kind of gift.
Duberstein:
0 Facts: After receiving some particularly helpful information from a business colleague,
Duberstein (D), Berman (B) decided to give D a gift of a Cadillac. Although D said he did not
need the car as he already had a Cadillac, but he eventually accepted it. Mohawk Co. later
deducted the value of the car as a business expense, but D did not include the value of the
Cadillac in his gross income when he filed his tax return, deeming it a gift (§102 excluded from
income). IRS claimed that D should have included the Cadillac as income on his returns and the
tax court affirmed. The appeals court reversed (no tax).
1 Holding: SCOTUS reversed the appeals court (taxable). When determining whether a transfer
of property is a gift, the focus should be on the transferor’s intent (question of fact  case-by-
case basis)
o Gifts result from "detached and disinterested generosity" and are often given out of
"affection, respect, admiration, charity or like impulses".
Contrast payments given as an "involved and intensely interested" act.
Stanton
Facts: Stanton resigned from employment to go into business for himself. As a "gratuity" the ER
awarded Stanton $20,000 in appreciation of the services rendered. While some directors testified
that Stanton had been well liked and the $20,000 was a gift to show that good will, there was also
some evidence given that Stanton was being forced out and the $20,000 was being given in
exchange for his resignation. Dist. Court judge said gift (not taxable). Court of Appeals reverses
(taxable).
Holding: SCOTUS remands to the DC, said issue is for the trier of facts. The trial judge made a
simple finding that the payments were a "gift".
Congress enacted §102(c) preventing EE from claiming that payments from an ER are gifts.
0 An exclusion for some “gifts in kind” were allowed as di minimis fringe benefits under §132(a)(4)
1 §139 Qualified Disaster Relief: amounts received from ER to reimburse for necessary medial,
temporary housing, transportation expenses incurred as a result of a Presidentially declared
disaster are excludable as gifts.
Proposed amendment:
0 HYPO: kids join in on Mom & Pop shop. Kids paid wages. Other things happen during the year
such as b-days and holidays. They get presents from their parents.
o How should these gifts to children be treated?
0.0 Extraordinary transfers to the “natural objects of ER bounty” will not be
considered business gifts!

The employer/business can usually deduct the amount of the gift as a business expense. However, because
the item is also excluded from the donee’s income, it will never be taxed. Therefore, §274 was adopted.
39
WNM
Tax 1 Ring (Fall 2017)
Rationale: Consider Duberstein; under §274, the business
can only take a $25 deduction for the gift → eliminates
any tax evasion!
0 Tips:
0.0 Tips are included in gross income. Giving tips is an
“involved and intensely interested” act.
1 Employee Death Benefits (to Spouse):60
1.0 Payments to surviving spouse equal to the deceased employees
wages until the end of the year (Carter) → GIFT (2d Cir.)
0.0 §102(c) may not apply because spouse is not an employee
Strike Benefits:
0 Strike benefits paid by union to all employees (some in cash; some in kind,
such as meals) → GIFTS, excluded
0.0 Keiser: deference to the finder of fact
0.0 “detached and disinterested” because the union paid
benefits to all employees, not just the union members;
this was union playing a social services role, NOT its
“union” role (like a public assistance programs)
1 Warning (narrowly applicable) → generally this would be a benefit
in kind (replacing wages) [don’t generalize this rule on the exam,
SCOTUS later moves away from calling something similar a gift]
§85: unemployment benefits:
0 taxable income (wages would have been taxed, so benefits should be too!)
Why are gifts generally not taxed? What is the difference between a gift and finding money
on the street?
Gifts are generally not taxed for policy reasons:
• Intrafamilial gift giving does not generate additional income. Therefore, we do not feel a
st
pressing need to tax these transfers. (e.g., mom and dad get Timmy a car for his 21
birthday. It’s all in the family.)
• There are also valuation and administrability problems with taxing gifts. It would be extremely
awkward for the TP to ask her aunt how much her precious antique earrings are worth in order
to include their value as income on his tax returns.
• We also refrain from taxing gifts because we would like to incentivize such generous behavior
and we hope that gift giving will result in the transfer of wealth between rich and poor family
members.
Payments to the spouses of deceased employees. If EE dies during the year, ER might give the EE’s
wife/husband $ = to the EE’s wages if (s)he’d lived for the rest of the year.
0 Carter and similar cases:
1 Two distinct trends in the law! SCOTUS doesn’t like this because it’s based on who can afford to
pay!
TC: taxable income
0 Cold people. Don’t care!  tax and be unhappy!
DC: not taxable income
0 Doesn’t see many tax cases, what they see are widows. Warm and fuzzy  not a
tax!
1 Government isn’t happy because the ER is deducting as a business
expense but the EE is not paying the tax!
40
WNM
Tax 1 Ring (Fall 2017)

Prizes & Awards


§§74, 274(j)
61
§74(a): expressly includes prizes and awards in gross income.
0 §74(b): EXCLUDE prizes and awards made primarily in recognition of religious,
charitable, scientific, educational, artistic, literary, or civic achievement, but ONLY
IF (1) no action on your part to enter a contest; (2) no future obligation for recipient;
and (3) the TP gives away the money to charitable or government
62
organization .
0.0 TP gets award  turns around and donates to charity  include in GI but
gets a deduction
0.1 Ex] Nobel Prize Winners must give away the prize money in order to avoid
being taxed
0.2 Ex] Hornung63 and Wills64
1 Examples:
1.0 TP wins a cash prize, donates the $ to charity
0.0 TAXABLE (§74(a)), but TP can then take deduction
1.1 We don’t tax Olympic Gold Medals even though it’s similar to the above
→ policy leaking in? There is no specific carve out for Olympic Gold
Medals.
1.2 Academy Awards?  artistic?

Overturned Washburn (held that a $900 cash prize won from a radio show by answering the phone was
excludable as a gift) and McDermott (held a prize awarded for an essay competition was excludable
from gross income)
Added after Hornung and Wills
§74(b)(3): if you donate the award to a charitable/government organization then we
will not tax you. o If (b)(3) did not exist and you gave the prize away  you
would get a deduction
o With (b)(3)  there is no way to keep a prize tax free.
0 If you keep your prize  taxed as income
1 If you give you prize away  not taxed (nets to $0)
Why didn’t they just get rid of (b)?
0 Under their formulation, there is $0 income under (b). But under a regular
charitable contribution regulation, you wouldn’t practically net to $0. (you might
only get ¾ of the prize as a deduction  b/c charitable contribution deductions
are limited)
1 Wanted to make sure that prizes that are given away really result in $0!
1.0 We don’t want to tax you for an achievement.
2 This is all
63
about the Nobel Prize!
Hornung v. Commissioner
Facts: Hornung claimed that the value of a Corvette he won for his performance in the NFL
championship shouldn’t have been included in his GI because it was a gift, or in the alternative
that it was excluded in §74 as an award for meritorious achievement. Hornung also argued that
the corvette was a gift under §102.
Holding: The court ruled that it was not a gift under §102 and was not excluded under §74 as a
prize/award.
0 The corporation that awarded the prize was not a disinterested party. They
advertised the giving of the Corvette so it wasn’t a gift.
The critical question was the nature of the activity Hornung claimed was being awarded and
the court was not convinced that the specific carve-outs in §74 were meant to encompass
athletic achievements.
Note: Can NOT be both a gift and an award  mutually exclusive concepts.
64
Wills v. Commissioner:
Facts: For breaking the MLB record for the most stolen bases, Wills was awarded the Hickok Belt, a
belt studded with precious gems given to athletes for athletic achievement. Wills argued that it
should be excludable under §74 and that it couldn’t be income because he would not have bought
this on his own, there was no utilitarian value, not useable—it’s a trophy (unlike a Corvette), there
was no FMV for it (since it was a trophy meant specifically for him so there would be no market to
sell it in), and finally that there would be liquidity issues in forcing him to pay taxes on it. Wills
also argued that he was awarded the belt for playing for an entire year (instead of one game like
in Hornung).
Holding: The court ruled that it was not excluded under §74. The belt did have FMV because the
individual gemstones had an FMV and if Congress had intended to exempt prizes and awards for
athletic achievement it would have included them in the specific carve-outs.
41
WNM
Tax 1 Ring (Fall 2017)

§74(c): EXCLUDES from income the value of an “employee achievement award65” if:66
67
0.0 the cost to employer does not exceed $400
0.1 the award is tangible/personal property (NOT cash)
0.2 the award is given for length of service achievement OR safety
achievement
0.3 award part of a “meaningful presentation” and does not create a
significant likelihood of disguised compensation.
1 Employer deduction for employee achievement awards
1.0 §274(j): ER deduction shall not exceed $400; satisfies other §74(c)
requirements
2 Examples:
2.0 ER gives EE $400 for “best attendance” → UNCLEAR (does it
satisfy §74(c) length of service? Is it given in cash?)
2.1 ER gives EE $500 watch for 20 years of work at a special luncheon → EE
EXCLUDES up to $400; ER DEDUCTS up to $400
2.2 ER sends EE $400 watch through interoffice mail  No
meaningful ceremony! Not excludable!
2.3 ER gives the EE a $400 watch for 25 years of service with the company 
Excludable

Scholarships & Fellowships


§§117, 127

SCHOLARSHIPS & FELLOWSHIPS


HYPO: TP gets $30,000 to attend BCLS. TP must be a degree candidate. TP gets to exclude the
$30,000 from income to the extent hat it is for tuition and related expenses except for room
and board.
How can you pay for school?
0 Borrow/loans: Eventually you will have to pay off your loans with after tax $$.
1 Work: Would not be exempt. Taxable. Coming out of your after tax income.
2 State university (subsidized  tuition less than FMV)
2.0 No special tax burden you bear by going to a state institution
3 Private school (alumni donation reducing the costs)
3.0 No special tax burden. The discounted piece is not taxable.
3.1 Sticker price $60,000. You pay $20,000. You’re not taxed on the $40,000
you’re not paying.

QUALIFIED EMPLOYER EDUCATIONAL ASSISTANCE PROGRAMS


§127: exclude from an EE’s income ≤ $5250/year for amounts paid by the ER under a
“qualified educational assistance program.”
0 QEAP includes tuition, fees, books, course supplies
0.0 Excludes: personal living expenses, recreational activity training
1 Applies to grad and undergrad
2 Must be established by a separate written plan of the ER for the exclusive benefit
of the EE and must be nondiscriminatory both with eligibility and
participation.
2.0 No more than 5% of the ER’s annual costs under the QEAP can
benefit officers, highly compensated individual or those with > 5%
ownership in the company.
Defined in §274(j) Employee Achievement Awards:
an item of tangible personal property which is:
(i) transferred by an employer to an employee for length of service achievement or safety
achievement, o (ii) awarded as part of a meaningful presentation, and
o (iii) awarded under conditions and circumstances that do not create a significant
likelihood of the payment
of disguised compensation.
Also consider if the award might be a di minimis fringe.
0 Policy: prevent ER from providing nontaxable disguised compensation, particularly to highly
compensated EEs, through nontaxable awards for regular job performance, such a
productivity, rather than other bonuses.
Acceptable double benefit tradeoff:
0 < $400 double benefit (exclusion for EE; deduction for ER)
1 > $400 double loss (excess is not deducted and excess is taxed)

42
WNM
Tax 1 Ring (Fall 2017)

2 Does not need to be related to the ER’s business. Outside what the EE does for work.
3 EEs can’t choose table compensation instead of educational assistance.
4 EEs can’t claim a deduction/credit w/ respect to the amount excluded from income
5 Policy: not fair to tax EE out of pocket for educational benefits from the ER
(restricts upward mobility)
Other Scholarships:
0 Miss America pageant  “scholarship” taxable because winner required to
perform services for the contest organizer
1 Athletic scholarships  §117 exclusion only if payment is not conditioned upon
participation in the athletic program.
1.0 Are viewed the same as academic scholarships.
ER-Provided Education Benefits:
0 Where an ER pays, directly or indirectly, education costs of its EE, §117(a) is not
applicable.
0.0 But in situations were the amounts are paid primarily for the benefit of the
ER, or if the rules relating to the qualified educational assistance fringe
benefits are met, the payments nevertheless may be excludable as tax-free
fringe benefits.

Social Welfare Payments


§§85, 86(a)

SOCIAL WELFARE PAYMENTS


federal and state governments provide a variety of welfare benefits to individuals. Subsidies can
be made:
0 directly in cash to low income/elderly individuals
1 to third party providers (e.g. medical care, housing, job training, education)
2 vouchers (e.g. food stamps)
3 in kind (e.g. public housing, health services, below costs public transportation)
4 substitutes for taxable income (e.g., unemployment compensation)
In general, benefits that are substitutes for wages are included in gross income, where as
benefits based upon financial need are excluded from gross income.
0 Policy: administrative grounds.
0.0 There is no excess. TP has no other assets. These welfare payments are
meant to cover the most basic necessities. Would be circular to then tax
them on it.

TAXING SOCIAL WELFARE PAYMENTS


§85(a): In the case of an individual, gross income includes unemployment compensation.
0 (b) Unemployment compensation defined. For purposes of this section, the term
“unemployment compensation” means any amount received under a law of the
United States or of a State which is in the nature of unemployment compensation.
§86(a): Social Security Benefits: We’re paying into the system through deduction in our
paychecks. These are forced retirement payments.
0 Modified Adjusted Gross Income
0.0 Mod AGI + ½ Social Security Payment
0.0 < $25,000  0% of the SS is taxable
0.1 = $25,000 to $34,000  50% of SS taxable
0.2 > $35,000  85% of SS taxable
Relationship of Categories
Qualified Disaster Relief Payments and Welfare are stand alones.
Gifts, Prizes, and Scholarships are mutually exclusive.
0 If you fail to make an argument that something is a prize/scholarship, then you
revert to §61 income status. You cannot then argue that it is a gift.
1 The §102 gift exclusion does not apply to prizes and awards or scholarships.
If an amount is a scholarship or fellowship, then it is not a gift.
Initial characterization of a payment is very important to determine which §applies.
43
WNM
Tax 1 Ring (Fall 2017)

Offsetting Liabilities
§§61(a)(12), 108(a),(b),(d)(1),(e)(5) & (f), 1017(b)(3)(A)

Cancellations of Indebtedness
TAXING DEBT: (We don’t!)
We do not tax $ that you borrow when you borrow it because you’ll have to pay it back. We
know the $ is not yours to keep.
0 HYPO: TP borrows $100,000 from his uncle for his education. Is the $ taxable
income?
0.0 Outcome: No, because TP must eventually pay it back.
1 HYPO: TP borrows $100,000 from his trust fund to throw a huge party. Is the $
taxable income?
1.0 Outcome: No, because TP is still borrowing.
2 HYPO: TP, the CEO, borrows $100,000 from his company. Is this $ taxable income?
2.0 Outcome: No, as long as the company is actually giving a loan that TP
must pay back it is not taxable income.
3 HYPO: TP has very high value stock that he would like to pass onto his kids, in
order to avoid taxation. He borrows $100,000 against the stock to live a comfortable
life. When TP dies the debt and the stock is passed onto his heirs.
3.0 Outcome: Family gets the stock, TP gets the benefit of borrowing
against the tax. But because he borrowed, the $100,000 he is not
taxed.

CANCELLATION OF DEBT (COD) INCOME:


TP owed a debt. However, for whatever reason, the creditor decided to forgive TP’s debt.
COD does not apply to price reductions for goods and services through negotiation.
Examples:
0 HYPO: Your parents loan you $1,000 to go towards law school. Some time pass
and they decide to forgive this debt.
0.0 Arguably a cancellation of debt  Taxable
0.1 TP can argue the $1,000 is a gift under §102  Not taxable
1 HYPO: You’re at the dentist and he charges you $100. You argue it’s at best
an $80 job and the dentist caves on the new price.
1.0 Not a cancellation of debt, because the new price was reached through
negotiation.
0.0 The dentist has $80 of income; you have an $80 dentil bill.
0.1 Adjustment of purchase price of services; not a loan debt.
2 HYPO: You’re at the dentist and he charges you $100. Instead of cash, you give him
a painting with a fair market value of $100 and a basis of $40.
2.0 Not a cancellation of debt. Your bill is $100 and you’re paying it with
something valued $100.
0.0 You have to realize $60 since you’re paying the bill with
appreciated property.
0.0 Amount realized: $100
0.1 Basis: $40
0.2 Gain/Taxable: $60 upon transfer → Could be capital gains
3 HYPO: Same as above. You give the dentist a painting with a fair market value of
$50 and as basis of $40. He accepts the painting. There is no negotiation over price.
3.0 COD! The dentist is basically loaning you $50 and then forgiving that debt!
0.0 COD Income = $50
3.1 The basis of the painting was $40. So, just like normal, you’ve realized
$10 since you’re paying the bill with appreciated value.
1.0 Amount realized: $50; Basis: $40; Gain/Taxable: $10 upon transfer
3.2 Total taxable: $60 ($50 COD income, $10 gain on sale of painting)
4 HYPO: Statute of Limitations. You’ve borrowed $100,000. The statute of
limitations runs on the ability to collect (State law sets a statute of limitations on
collections for various kinds of claims). Is this a cancellation of debt? Should we
say the statute of limitations has effectively cancelled the debt in the way we think
of in §61? IRS has successfully argued this is income!
4.0 Cancellation of debt through law.
44
WNM
Tax 1 Ring (Fall 2017)

TAXING CANCELATIONS OF DEBT:


§61(a)(12): If a debt/loan is forgiven, the unpaid amount of the loan is taxable as income.68
0 HYPO: Law school loans $50,000 to set up a café in the parking lot. Turns out
students love the yellow room and wont go outside. The law school cancels the
debt and says to the café, you don’t have to pay us back.
0.0 Outcome: the café will have to treat the cancelled debt of $50,000 as
income! The café has $50,000 of COD income.
1 §166: bad loss debt exception = when the lender cancels the debt in cases where
it’s clear they won’t get the money back; they can count it as a loss for that tax year.
Exceptions to COD:69
0 Insolvency Exception: [§108(a)(1)(B)] Discharge of debt won’t be income if the
70
debtor is insolvent or bankrupt.
0.0 Policy: It is unrealistic to expect a TP that cannot pay his loans to pay a tax
on the COD.
71
0.1 §108(d): Even Out the Exclusion: TP will loose future “tax
goodies” (e.g., credits, carryovers) to make up for the cancellation of
debt income that he is not being taxed on because he is insolvent.
1 Anti-basis Exception: When you are forgiven on a debt in which you did not
receive any $ upfront, you will not be taxed on the COD.
1.0 HYPO: TP promises under seal to pay ß $1M. The next day, TP realizes
this was so silly and cancels the debt. Does TP have $1M of COD?
0.0 No, because TP did not receive any $ upfront.
1.1 HYPO: TP required to pay $10,000/year in child support. TP is insolvent so
he does not pay the $10,000. After 3 years of not paying, the SOL on the
child support runs out. Does TP now have $ 30,000 of COD income?
1.0 TP will not have to pay taxes because he did not receive any funds
upfront.
1.2 HYPO: TP runs over π. π sues and wins $50,000 but does not collect the $
before the SOL on collection runs out. Does TP now have $50,000 of COD
income?
2.0 TP will not have to pay taxes because he did not receive any funds
upfront.
2 Unenforceable Under Law Exception:
2.0 Zarin:72 a debt that unenforceable under law is not taxable. (minority rule)

Kirby Lumber: must argue this case if you’re talking about COD!
0 Facts: TP issued bonds (debt) and bought back for less money than they were sold at. Cost Kirby
$11M to pay off a $12M debt.
1 Holding: Extra money is taxable → duty to repay is gone. We don’t tax the act of borrowing
because there’s an offsetting liability to replay. HERE, TP didn’t pay back the entire liability, so
they were taxed on what they didn’t pay back. Frees up assets and the duty to repay is GONE.
HYPO: BCLS loans $50,000 to a vendor to open a coffee cart in the parking lot. The coffee shop is
unsuccessful so the school forgives the loan. At the time of cancellation the coffee shop is insolvent (its
liabilities far exceeds its assets).
0 This is a COD. But, under §108 the IRS will not tax the coffee shop on the COD because the coffee
shop is insolvent.
1 Policy: its unrealistic to ask the coffee shop to pay tax on their $50K forgiveness because they can’t
pay it. Broader approach to fresh start. We think it’s less likely you’re going to pay the tax bill on
$50,000 when you’re insolvent.
TP is not taxed on his COD income!
Recall: you got that $50,000 to spend/use on the first place but we didn’t require that you pay taxes
on it because it was borrowed.
Wages: can TP exclude wages from his income because he is insolvent?
There is no §108 exclusion for wages!
Why not? We want to prevent those with quite a nice actual cash flow from avoiding taxes simply
because their
liabilities >> wages.
Similar to §§109, 1019 (response to Brun): Excluding taxation of building improvements by the
tenant but we’ll make the basis in it equal to zero so if you discharge it in the future, you’ll be taxed on
the full realization.
0 Version of deferral – not reporting it now, but the system will pick it up later.
Zarin: TP was a compulsive gambler in NJ and a regular in the casinos. He had a debt of $3.5 million to
one of the casinos and ultimately they settled with him for $500,000.
• TP: Does not want the settlement to be considered a cancellation of debt income
0.0 The debt is unenforceable under NJ law (at the time gaming debts were not enforceable
debts).
0.1 §108(e)(5): If seller finances the buyer of property, then later reduces the price
 treated as price negotiation  not taxed
45
WNM
Tax 1 Ring (Fall 2017)

Claim of Right & Subsequent Repayment


§1342
1 Claim of right: When does $ truly belong to the TP, such that he must report it as income?
0.0 Rule: Include it in your income, even you have to pay it back later.
0.0 “amounts received under a claim of right and without restriction as to their
disposition constitute GI and are taxable in the year of receipt even though
the taxpayer may be required ultimately to restore an equivalent amount”
0.1 North American Oil73
0.1 HYPO: TP gets a $50,000 retainer from his Client to provide future legal services.
1.0 The retainer is treated as pre-payment for TP’s services not a loan. It is
considered income unless the TP gives the money back.
0.2 HYPO: A landlord charges tenants a security deposit of $500 and prepaid rent of
$1,000.
2.0 Unlikely the LL will give back the prepaid rent so the $1,000 should be
included in income.
2.1 Expected that the LL will return the security deposit so the $500 won’t be
included in income.

2 §1341 Subsequent repayment: How is TP taxed if he is required to return part of his income in a
74
later year?

2.0 §165(d): This statute allows gambling losses to be deducted from any gambling gains as
income. So, losses should offset COD.
2.1 Anti-basis: Didn’t get actual money upfront!
IRS: Zarin has a $2.9 million worth of taxable COD income
Tax Court: It is taxable income. Ruled for IRS. Rejects all four arguments.
0.0 The taxpayer got something of value up front. Contract enforceability is irrelevant within tax
system.
0.1 §108(e)(5): Only applies to property!
0.2 §165(d): Technical argument: §165(d) requires two elements to use your gambling losses
on your tax return. Gambling losses can only offset gambling winnings. COD income ≠
gambling winnings/income.
0.3 Anti-basis: The taxpayer received the ability to gamble – this is value.
Appeals Court: reversed; Taxpayer wins. Obligation that TP owes to the casinos would not be a debt
under §108(d)(1). Not enforceable under state law. Contested liability  price reduction.
• Problem: arguable statutory misread! the definition of debt under §108(d)(1) is applicable only
to §108!

North American Oil:


• Facts:
0 1916 – TP’s property in receivership, but still generating income – unclear who controls
1 1917 – $ earned in 1916 is given to TP → court ends receivership; clear TP controls
2 1922 – litigation is ended in TP’s favor
Tax question: In what year should TP be taxed?
0 Tax at lowest rate in 1916 – TP’s preference. Rates increased in 1917 due to WWI and then
dropped again in 1922 → TP really doesn’t want to be taxed 1917 (unusual situation
because the TP does not favor a deferral to 1922)
Holding:
0 1916: No. The $ is clearly in dispute. TP does not have a claim to the $.
1 1917: Yes. The $ is no longer in dispute. The court gave TP the $ earned in 1916.
2 1922: No. Once TP got the $ he should have paid taxes on it. If the $ was taken back
for any reason, we’ll cross that bridge when we come to it by allowing you to take a
deduction then.
Policy: We don’t want TP to drag out litigation to avoid paying taxes.
Lewis:
Facts: Taxpayer gets a bonus in 1944. Later on after 1946, he has to return part of his bonus
because it was miscalculated. [Tax Rate was higher in 1944 than in 1946]. Taxpayer is netting
zero income so any tax amount is a problem.
0 Year 1 taxpayer gets $10,000 in income. He’s taxed at 70% ($7,000).
1 Year 2, taxpayer returns $10,000. The tax rate is 30% and he gets to deduct for $3,000 of
income.
2 So, TP has a net income of $0, but pays $4,000 in taxes because the rates have changed.
TP: argues that the $4000 in tax on $0 in income is unconstitutional.
Court: Hold in favor of the IRS. The tax system works on an annual basis. Unfortunately, that means
that sometimes things don’t work out perfectly from year to year ( see Sanford & Brooks).
Note: As a response to this holding, Congress passes §1341.
46
WNM
Tax 1 Ring (Fall 2017)

0 §1341 is special because it gives the TP a choice of response.


1 IF (three requirements):
1.0 Year 1: TP includes an amount in income [unrestricted right]
1.1 Year 2: TP “returns” that amount and takes a deduction (reduces TI)
1.0 [said differently, pays back the amount in year 1]
1.2 Deduction exceeds $3,000
2.0 If < $3,000 → MUST take deduction in Year 2
2.1 Policy: Only here to solve really egregious problems.
2 THEN (TP chooses either OPTION):
2.0 Traditional Method: Take deduction in Year 2 at the Y2 tax rate, OR
2.1 Alternative Method: Reduce tax bill by amount equal to additional taxes
paid in Year 1
1.0 Said differently, how much would TP’s tax bill would have
been in Year 1 → subtract that amount from the current tax
amount. [Not opening year 1 return, rather, letting you
subtract this overpayment in the current year.]75
Examples:
0 HYPO:
0.0 Y1 (t= 30%): TP pays taxes of $3,000 on a $10,000 bonus.
0.1 Y2 (t= 70%): TP is asked to pay the $10,000 bonus back and does so.
1.0 TM: TP gets a deduction of $7,000!
1.1 AM: TP gets a deduction of $3,000!
1 HYPO:
1.0 Y1 (t= 70%): TP pays taxes of $7,000 on a $10,000 bonus.
1.1 Y2 (t= 30%): TP is asked to pay the $10,000 bonus back and does so.
1.0 TM: TP gets a deduction of $3,000!
1.1 AM: TP gets a deduction of $7,000!
2 HYPO: On New Year’s Eve, the bank credits your account in error by
$1,000,000. They discover the error on New Year’s Day and takes off the
$1,000,000.
2.0 There might not even be a claim of right! Doesn’t seem like it
since you didn’t know it was there, take it out, and use it; No
need to report as income then deduct.

Embezzled Funds
TAXING EMBEZZLED FUNDS
RULE: TP is required to report income derived from unlawful sources [James76]
0 §61 is not limited to legal sources of income.
1 Repaying embezzled funds is not like repaying a loan because …
1.0 You did not intend to repay the funds in the first place
1.1 You never had a claim of right to the funds
CONCURRENCE: TP is not required to report income derived from unlawful sources.

The flip works too! So TP might get $4000 if


75
the tax rates reversed. Taxation method:
good when rates go up and bad when they go
down. Alternative method: good when rates go
down and bad when they go up.
76
James: held that money obtained by a taxpayer illegally was taxable income, even
though the law might require the taxpayer to repay the ill-gotten gains to the person from
whom they had been taken.
Include in Income? Convict?
Brennen Yes. Theft is on the reporting side because it is not like a loan which is a consensual transfer. No, because there is n
Stewart willfulness.
Warren Three categories of ways of getting $$
Clark Claim of right (yours to keep) Yes. Glenshaw  all
Theft (stole)  on the reporting side! accumulations of wea
Loan (borrow)  consensual transfer “however secured.”
Harlan New Trial
Black No. IRS should not be taking away from the victim. If the ∂ paid taxes on it, then the V wont No
Douglass get the full amount back.
Whittaker No. If the V cancels the debt then we can tax. No

47
WNM
Tax 1 Ring (Fall 2017)

0 Black: Including incoming (getting taxed) + returning embezzled funds = double


punishment
1 Whittaker: Treated like a loan. Exclude income from unlawful sources unless the
victim cancels the “debt” (embezzler gets to keep the funds)
McKinney: NOL §172  special provision shouldn’t be available to a bad actor, even though
the bad actor is required to report the income.
0 ∂ also wasn’t doing it consistently/regularly

DEDUCTING EMBEZZLED FUNDS


If TP is forced to return embezzled funds he does not get the benefit of choosing §1341. TP’s
deduction will be calculated according to the tax rate in the year in which the funds are
repaid.
0 Policy: §1341 is limited to a claim of right – TP does not have a claim of right over
funds he obtained illegally.
48
WNM
Tax 1 Ring (Fall 2017)

Tax Expenditures & Concept of


Income
Tax Exempt Bonds
§103(a): interest income from bonds issued by state and local government is not included income.
0 Policy: provides a source of low cost funding for state and local governments.
0.0 State and local governments can offer bonds at a lower interest rate.
Because interest from these bonds is not included as income on the TP’s
federal tax returns (whereas corporate bonds are), TP will be incentivized
to choose a state/local gov’t. bond over a corporate bond, even if the
state/local gov’t. bond has a lower interest rate (result in lower interest
income) than the corporate bonds.
1 Higher Income Individuals benefits more from Tax Exempt Bonds:
1.0 Because we do not have enough individuals in the higher income bracket
(H), we need to incentivize those in lower income brackets (L) to purchase
Tax-Exempt Bonds as well.
1.1 In order to incentivize Ls to purchase these bonds we need to increase the
interest rate.
1.2 Creates a sort of windfall for higher income individuals
HYPO: Suppose taxable bonds were paying 20% in interest, while nontaxable bonds were paying
14%, which would a TP prefer if the tax rate were 50%? If the tax rate were 30%
0 Scenario 1: 30% Income Tax Bracket; Bond = $1000
Interest Income Federal Tax After Tax Income
Taxable (Corp.) Bond $200 $60 $140
Interest Rate (i) = 20%
Tax-Exempt (Gov.’t) Bond $140 $0 $140
i = 14%77
Tax-Exempt 10% $110 $0 $110
$30 loss if TP buys a Government bond. The 30% TP will not be enticed to buy this! Because they could just buy a
corporate bond and have an ATI of $140 instead of $110.

b. Scenario 3: 50% Income Tax Bracket


Interest Income Federal Tax After Tax Income
Taxable (Corp.) Bond $200 $100 $100
Tax-Exempt (Gov.’t) Bond $140 $0 (IRS is giving up $100 $140 (HTP is getting $40
for benefit of S/LG) tax free  windfall)
Tax-Exempt $110 $0 $110
Still a $10 gain if TP buys a Government bond. The 50% TP will still be willing to buy it

Why does section 103 exempt the interest earned on state and local bonds from federal
income taxation? §103(a) exempts interest income earned on state and local bonds from federal
income taxation. By exempting the interest earned on state and local bonds the federal gov’t. is
essentially channeling tax dollars to state and local gov’t indirectly through the tax system. Using the tax
system to channel money to state and local gov’t. means that money comes without strings attached like
it would if it were coming through other federal spending programs.
Because of the tax exemption state and local gov’t can offer bonds at lower interest rates compared to
their private/corporate bond issuers whose bonds’ interest income will be taxed. For example, a TP in the
50% income tax bracket, who is willing to purchase a corporate bond at a 20% interest rate, would be
willing to purchase a tax exempt state/local bond at let’s say a 10% interest rate. This benefits
state/local gov’t. because they can compete for bondholders with corporate issuers while offering bonds
at a lower interest rate.

Under what circumstances might we be concerned that section 103 is not doing its “job” but
instead is providing a windfall to certain taxpayers?

Unfortunately, higher income individuals benefit from purchasing state/local bonds more than lower
income individuals.

Effectively subsidizes the state/local government by making it possible for the state to get $ at a lower
interest rate. Makes the 14% S/L bond competitive with a 20% corporate bond. The state is saving in
interest expenditures. It is as if the transfer is from the fed  state/local.
49
WNM
Tax 1 Ring (Fall 2017)

There are not enough high income TPs to fund state/local bonds completely. So we need to incentivize
lower income TPs to buy them too. In order to incentivize lower income individuals to choose state/local
bonds over corporate bonds, the state/local gov’t. need to raise the interest rate above what it would
take to get a higher income TP to purchase a bond. To make it worthwhile for a lower income TP to
purchase a state/local bond, the state/local gov’t. needs to increase the interest rate from the amount
the higher income individuals would have paid. This will create a windfall for the higher income
bondholders because by buying a tax exempt bond from the state/local gov’t at this higher interest rate
they are able to enjoy more tax free interest income than they would receive if they were to purchase a
corporate/taxable bond for the same amount.

Why does a state or local government issuing a section 103 qualified bond, not use the
lowest interest rate it can offer and still get someone to buy some bonds? (i.e. if someone will
buy the bond if it pays interest at rate of 10% when interest rate on taxable bonds is 20% then why pay
anything more than 10% interest?

Unfortunately, there are not enough high income TP (those who would buy a bond at a 10% interest
rate) to fully fund state/local gov’t. projects. So the S/LG has to offer higher rates that would be
attractive to lower income TPs in order to incentivize them to purchase these bonds also so the S/LG can
raise enough $$.

Tax Expenditure Budget


Government expenditures carried out through the tax system (rather than through direct
78
government spending; way of providing government financial assistance)
Tax Expenditures:
0 Exclusions from income (e.g., interest income from state and local bonds)
1 Deductions (e.g., charitable contributions)
2 Credits
3 Preferential rates (e.g., capital gains)
4 Deferral of tax (e.g., discount obligation bonds)
Special exclusions from GI, special deductions, tax credits, special accelerations of deductions
or deferrals of income are considered Federal Spending Programs.
0 A TP who is granted a special exclusion from income is in the same economic
position as if the tax has been paid on the fill amount of economic income (without
the exclusion) and the Treasury Dep’t. had returned a check to the TP equal to the
amount of the tax saving result from the income exclusion.79
1 Some items, although clearly income, are excluded from the tax base in order to
achieve some specific economic or social objectives.
1.0 Tax incentives  intended to encourage certain types of activities
1.1 Social objectives  relieving personal hardship

Concept of Income
H-S (pure) ––– Base-line income tax (exp.) ––– Code
0 Haig- Simons  most influential definition of income
0.0 Income = consumption +  accumulation to your saving this year
0.1 Focuses on the use
1 §61  focuses on the source.
We don’t use H-S because it taxes imputed income, appreciation in value, etc.
0 We don’t think we should tax those things though.
1We came up with a confusing base-line idea to meet the realities of our world
and our preferences. (realistic ideal)
Code: what we actually have doesn’t quite reach out realistic ideal.

History: Emerges in the 60s under Stanley Surrey. You have a tax system with rules and if you follow it that’s
what you’re supposed to pay. But sometimes there are special rules that creep in that are exceptions that allow
you to keep your money.
Better than the government taxing and then giving the $ back because the $ given back would then be
taxed as income. But since we are not going through this back and forth  the tax on the would be
income is also viewed as a tax expenditure and it is an additional benefit to the TP who gets to keep that
tax too.
50
WNM
Tax 1 Ring (Fall 2017)

Alternative Tax Base


1. Consumption Tax: TP is taxed on what he consumes (eats, spends) this year.
a. Does not tax savings. Only consumption.
b. Income Tax ≈ Haig-Simon Income Calculation = consumption + savings
i. CTW: Income = Consumption
c. HYPO:
i. Y1: $100 of salary. Spends $70.
ii. Y2: Spends $30.
Y1 Y2
Income Tax $100 $0
Consumption Tax $70 $30

2. Wage Tax: TP is taxed on the wages he brings home this year.


3. Our World: income = consumption + savings + tax
a. Ours is in theory an income tax world. However, upon closer analysis we might
actually conclude that it plays out more like a consumption tax world.
b. How does the tax rule you put in place, impact TPs behavior.
c. HYPO: TP earns $100 in Y1. TP can invest at 10%. Tax rate = 50%.
Y1 Y2 Ratio
No Tax $100 $110 100/110
Income Tax $50 $52.50 50/52.5
Consumption Tax $50 $55 50/55
80
Wage Tax $50 $55 50/55

4. Why are we concerned about alternative tax bases, if we’ve determined that we have an
income tax?
Well-known feat. of our sys. Income Tax Consumption Tax Our System
Owner Occupied Tax Tax No Tax
Tax Except Bond Tax Defer until interest Deviation from an income tax; more consistent
is eaten with a consumption tax.
Retirement Taxed as they Defer until interest Deviation from an income tax; more consistent
earn income is eaten with a consumption tax.
a. Overriding norm – is income tax. But we have exceptions that are closer to a
consumption tax
Why are we uncomfortable with a wage tax world? Since these two are the same, should we be
worried about a consumption tax world too? Why are they the same?
51
WNM
Tax 1 Ring (Fall 2017)

DEDUCTIONS
Deductions
IN GENERAL
What Are Tax Deductions? Simply stated, deductions reduce taxable income. Each deduction
reduces tax liability by the amount of deduction times the tax filer’s marginal tax rate. In contrast,
a tax credit reduces tax liability on a dollar-for-dollar basis because it would be applied after the
marginal tax rate schedule. An individual in a 35% tax bracket would receive a reduction in taxes
of $35 for each $100 deduction while an individual in a 25% tax bracket would receive a
reduction in taxes of $25 for each $100 deduction. Hence, the same deduction can be worth
different amounts to different tax filers depending on their marginal tax bracket. The tax savings
from deductions are generally equal to the tax filer’s marginal tax rate times the amount of the
deduction. So higher income tax filers typically benefit more than lower-income tax filers from
deductions.
“Deductions are a matter of legislative grace.”
0 TP does not have a Constitutional right to a deduction.81 Congress was
benevolent in writing deductions into the Tax Code, but has the power to
take them away at any time.
1 If you spent some $ to earn some $$$ the $ you spent is deductible (not the parallel to
basis).
1.0 Ex] wages, equipment, electricity bills, insurance, rent
1.1 You should get to recover/deduct the $ you spent at some point.
1.2 The rules help us know, if and when, we get to deduct our expenses.

EXPENSES
§162(a) Trade or Business expenses  deduction permitted in the year the TP incurs the
82
expense.
0 Ordinary and necessary expenses incurred, while carrying on any trade or
business are deductible.
§263 Capital Expenditures83  deduction permitted upon sale or over time for depreciation,
rather than in the year the TP incurs the expense.
0 Things that TP buys that will help him make $ in the future.
0.0 Ex] machinery, buildings, land
1 Policy: although Capital Expenditures are a business expense, we do not allow the TP
to deduct the expenses right away because the expenditure will continue generating
84
income for many years.
2 TP can recover the asset through sale or depreciation deductions in the future
§212 Profit seeking:
0 Expenses incurred in the production of income
1 An individual may deduct expenses if they are: ordinary + necessary + for
the production of income (profit seeking)
1.0 Ex] Cost of hiring a tax accountant→ deductible under §212(3)
1.1 Ex] Broker fees
1.2 Ex] Cost of WSJ subscription
§262 Personal expenses  deduction permitted only when a special exception exists.
0 Do not deduct personal, living, or family expenses unless there is a special
exception (e.g., home mortgage deduction; student loan interest) within the
Code.
0.0 Policy: unlike business expenditures, personal expenditures do not go
towards making $ for the TP, therefore, it does not make sense to deduct
them.

TP does have a Constitutional right to recover his Basis [Doyle Brothers: Basis is a Constitutional right  TP
has the right to a deduction of that basis from GI].
Immediately deductible  TP prefers §162(a) over §263.
0 Deferral/exclusion of income is always preferred and preferred sooner rather than later. TP can
take the money he would’ve spent on taxes and use it for investment growth over time!
(interest free loan from the gov’t concept).
Capital Expenditures have nothing to do with Capital Assets
Compare: a piece of machinery (CE) may work for 10-15 years, but the electricity bill (TBE) only keeps
the lights working for today
52
WNM
Tax 1 Ring (Fall 2017)

Business Non-Business
Business Activities Capital Exp. Profit Seeking Personal
Expenses §162 §263  future §212 §262  NO Deduction
“Ordinary and deduction “Ordinary and
Necessary” Necessary AND
Product of Income”
Loses §162(c) §165(c)(2) Some casualty loses
Bad Debts §166 §166(d) partial §166 partial deduction
deduction
Comments The electricity bill only keeps Match the expenditure Resulting income is confined Not producing any income.
the light on this month. The with the income. The to the year in which the Pure consumption.
expenditure only helps you incomes is spread out expense was incurred.
earn $ in this year, not over over time. The
time. equipment doesn’t help
you earn $ just this
year. It helps you earn $
over time.

Profit Seeking Expenditures


Ordinary and Necessary Business Expense
Ordinary: “Common & Accepted”; not capitalized
0 Is this kind of expenditure “normal” for a business in this industry?
0.0 Welch v. Helvering: paying off business debts that were previously
discharged is not an “ordinary and necessary business expense.” No one
else in the business does this. Instead it is more like a capital expenditure,
because the TP was hoping to reap the benefits of paying off those expenses
over time  no deduction is permitted under §162.85
1 The expense can be incurred only once; the expense does not have to be incurred
daily.
1.0 Ex] litigation expenses
2 Policy: trying to keep out capital expenses
Necessary: “Appropriate & Helpful”; not personal
0 Vitale: some expenses, such as hiring prostitutes, are just too personal to
86
warrant a business deduction.
1 Policy: trying to keep out personal expenses

Welch v. Helvering: (1933)


0 Facts: T. Welch and his father owned grain brokerage business that went bankrupt. The
corporation’s debts were discharged. T. Welch then opened a new business in the same industry.
He wanted to improve his reputation with former clients, so he chose to repay his father’s
discharged debts from their now bankrupt business. He then tried to deduct the repayments as a
business expense. IRS ruled that these payments were not deductible from income as ordinary
and necessary expenses in the course of business.
1 Holding: The expenses were not deductible under §162 because they were not "ordinary and
necessary business expenses.”
o The expenses were too personal to be business expenses and they were too
bizarre to be ordinary expenses. The Court suggested that ordinary and
necessary could be determined by looking at the prevailing practice in the
business world.
1.0 It was not the prevailing practice in the business world to pay off debts you
didn't have to pay off. This was more similar to a capital expenditure.
2 “Necessary” was defined as "appropriate and helpful [in] the development of the [taxpayer's]
business."
3 Barrier against §262: pushes back against personal expenses that might creep into the
business deduction. o Necessary = not personal
“Ordinary” = NOT capitalized
Vitale:
0 Facts: TP, an author, wanted §162 deduction for “research” relating to his book on prostitutes.
He argued that he was in the business/trade of researching prostitutes.
1 Holding: NOT a necessary expense → Even though being an author is the TP’s business/trade,
some expenditures are just too personal. (Not that the expense is personal, but simply that it
seems too personal).
53
WNM
Tax 1 Ring (Fall 2017)

Examples:
0 Ex] Business hires an MBA for $1000/month to consult on business decisions.
0.0 Ordinary and necessary (not personal)
1 Ex] Business hires a wizard for $1000/month to consult on business decisions.
1.0 Ordinary  Not that different from having an MBA! Not happening over
the long term.
1.1 Necessary  Yes, not personal
2 Ex] Business hires a religious advice for $1000/month to consult on business
decisions.
2.0 Ordinary  Regularly contributing, generating monthly income  okay
2.1 Necessary  more personal, because TP’s religious beliefs? [Ct. denied
the deduction b/c believed it was too personal]

PUBLIC POLICY LIMITS


Applies to things that would otherwise be ordinary and necessary.

§162(c)(1): No Deductions for Illegal Bribes or Kickbacks Given to Government Officials


0 HYPO: TP bribes the city in order to have ramp moved so the highway exits right
by his store. Can TP get a deduction on the bribe?
0.0 There are no deductions for illegal bribes, kickbacks, etc.
0.0 No conviction requirement: TP does not need to be convicted of
87
bribery to loose his deduction.
1 Bribing is like paying for a personal expense (i.e., going to the movies)
2 §162(c)(1) includes payments to a foreign government worker
2.0 Exception: if it is not unlawful under the foreign corrupt practices act, TP
can still deduct the payment. If it’s a foreign bribe, the payment is not
deductible if it violates the Foreign Corrupt Practices Act  response to
U.S. multinationals who live in places where it is understood that bribes to
public officials are the norm.
§162(c)(2): No Deductions for Illegal Bribes or Kickbacks to Others 88
0 HYPO: TP runs the housewares department at a department store. TP
accepts bribes from a manufacturer to select their product line.
0.0 James: Although the income comes from an illegal transaction, it is still
earned income so the department store manager must be taxed on it. BUT,
the manufacturer cannot take a deduction on the bribe because it was illegal.
§162(c)(3): No deduction for Medicare bribes
0 Even if the bribe is not illegal  no deduction
1 Policy: tax income funds this govt. program; don’t want bribes to increase the cost of
services
§162(f): No Deductions for Fines and Penalties
89
0 Policy: A deduction would defeat the purpose of the penalty.
1 If the fine is high enough, breaking the law won’t be economically rational anyways.
Too expensive to break the law; not enough savings. Fine outweighs any extra
income. But, if we allow a deduction then it might become rational to break the law
because of the tax savings.
2 HYPO: TP, a trucker, overloads his truck and is fined $1,100 as a result. TP earns an
additional $100 per overloaded trip. After ten trips, TP can make $1,000 extra. The
tax rate is 50%.
2.0 Without the deduction: It is not worth overloading because the fine is
$1,100.
0.0 So, TP looses $100.

Is this constitutional?
0 TP spends $7000 on a bribe and earns $10,000. §162(c)(1) tells us there is no deduction.
Therefore, the TP would be taxed on the full $10,000. Is this constitutional?
History: Didn’t exist before 1970. Gov’t. used a general public policy argument. TP usually won. So
Congress instituted the predecessor to §162 (required a conviction).
What bothers us about giving a deduction? Eases the fine, if a deduction is given. You’re supposed to
pay. Seems counterintuitive to give a deduction. Reduces the pain of the fine. Penalty is set to stop the
bad behavior (penalty must be high enough to make the bad act not worth it). If you get a tax
deduction, then it won’t stop the behavior. It undoes the penalty. But if you assume it is deductible, then
you can do the math and set the penalty at a higher rate. But still, it sends the wrong message to say
it’s bad and then give deduction which is a good thing.

Does the tax system have a moral role?

54
WNM
Tax 1 Ring (Fall 2017)

0.0 With the deduction: TP is earning $1,000. He get’s fined $1,100 but he
gets to deduct that from his income. It’s taxed at 50% so you’re taxed
$500. Get $500 in your pocket.
§162(g): Treble Damages
0 TP can usually deduct the full amount of settlement he pays in a lawsuit from his
income.
0.0 Exception: anti-trust lawsuits (2/3 is not deductible; 1/3 is deductible)
0.0 Why? In anti-trust lawsuits, the full amount of damages is
calculated by tripling the actual harm caused.
0.0 Actual harm = $1,000
0.1 Damage payment = $3,000
0.1 TP can deduct $1,000 of damages as business expenditures.
However, the remaining $2,000 is taxed.
1.0 Analogous to a fine or penalty since it is punitive.
Deductions for Illegal Acts:
0 How do we deal with deductions for expenses incurred in the process of illegal
acts or as a result of illegal acts?90
0.0 HYPO: You run an illegal prostitution business. Can you deduct your phone
bill?
0.0 James: All the earnings from the business should be reported as
income. Morality doesn’t matter to the tax system.
0.1 Rationale: Courts were hesitant to not allow the deduction
because using the phone in and of itself is not illegal – wouldn’t
be caught under §162(c).
1.0 States got around this by making it illegal to use the
phone to facilitate a prostitution ring.
1 §280E: No Deductions for Drug Dealers:
1.0 Tax system hates drugs, more than prostitution or any other illegal activities
1.1 A drug dealer can still recover his basis as AR-B=TI!
1.2 Ex] If you have a legal marijuana dispensary in Colorado, you
cannot deduct your employee’s wages as a business expense on
your Federal Income Tax Returns!
2 Legal fees:
2.0 If the legal fees originate from TP’s business, they are deductible under
§162.
2.1 Ex] Tellier91
§162(e): No Deductions for Lobbying. (influencing legislation, participation in a political
campaign, any attempt to influence the general public . . .)
0 Exception – TP can deduct expenses for lobbying local officials for business
purposes

SALARY
§162(a)(1): Deductions for Salary
0 Adds reasonableness requirement
1 Concerns with the “salary” category: businesses disguising dividends or gifts as
salary
1.0 There’s an incentive to disguise the dividends as more salary because they
can be deducted. In either case it’ll be taxable to the employee as income,
but the business will get a deduction when it’s called salary.
0.0 Dividends are not deductible by the paying business.
0.1 Reasonable salaries are deductible by the paying business.
1.0 Patton: salaries must be reasonable in order to be
deductible. Only a reasonable amount can be
deducted.92

There are expenses associated with running an illegal business. If the states make something illegal, then
getting a federal deduction for it is illegal too!
Tellier: TP wants to deduct his legal expenses from defending against a criminal charge. The court held
that since the criminal charges stemmed from Tellier’s securities business, the legal fees were expenses in
carrying on a trade or business under §162(a). The tax code does not concern itself with the lawfulness of
the income it taxes, so no public policy is offended when someone convicted of a crime seeks a lawyer in
their defense. The Constitution gives you a right to defend yourself, so not allowing you to deduct legal
fees associated with your business would be unfair.
Patton:
0 Facts: In the 30s, EE was regularly unemployed. From 1937 to 1940, TP company pays EE
about $1,200/year. In 1941, EE gets a new deal where he gets to keep 10% of profits. During
war, company gets huge GM contract. Therefore, in 1943, taxpayer gives employee $46,000 for
his services. EE is not a highly skilled worker.
55
WNM
Tax 1 Ring (Fall 2017)

§132(d): Working Condition Fringe:


0 Allows the EE to take a deduction when the ER provides the EE with
something in his working environment that EE would have been able to
deduct under §162 as an ordinary and necessary expense if EE had paid for it
himself.
0.0 Being an EE is considered a trade or business
0.1 HYPO: ER buys copy paper for the office. ER gets the deduction.
0.2 HYPO: ER pays EE an extra $100 in salary. EE uses the salary to buy
copy paper for the officer. EE must report the $100 as income, but can
then take a deduction on it!

PRODUCTION OF INCOME
§212: Production of income
0 TP can deduct the ordinary and necessary expenses of profit seeking activities
1 Profit seeking activities: “side job” something that does not take all of TP’s efforts.
1.0 Ex] Investing. Still profit-seeking, not personal – spending money to make
money
1.1 Ex] Stock broker:
1.0 when TP is working for his clients  trade/business
1.1 when he’s working for himself  “side job” (profit seeking)
2 §212(3) – TP can deduct expenses for paying his accountant to do his taxes.

Capital Expenditures
§263
93
§263: CAPITALIZE (do NOT deduct) the amount paid out for new buildings or
other permanent improvements/betterments made to increase the value of a
property or estate (in the same year as the expenditure)
a. Capital Expenditures contribute to an
increase in basis. i. The
deduction occurs at:
0.0 Over time (depreciation)
0.0 Ex] A machine the owner buys in Year 1 for $500,000 will
continue to produce income for the next 10 years.
Therefore, it does not fall under §162

Taxpayer company: We should get to deduct $46,000 as a business expense (employee’s


salary) under §162.
IRS: $46,000 is a problem but we’ll let you deduct $13,000 as reasonable. Government
thinks they were
paying the $46,000 so the taxpayer company would get a deduction, then the employee
would kick back the amount to the corporation to give to partners as dividends. Getting
a really big business deduction was worth giving a little money extra to the employee to
cover his taxes.
0 Is it really something else? Is it disguised? Not a relative, a friend, or owner of the co.
Doesn’t look like a disguised payment for something else he does.
Court: For the IRS. Affirms $13,000.
Dissent: Why is any of this being challenged? The employee is not related to the owners. The
taxpayer can pay the employee what they want. Contract rights assume fair bargaining power.
WARNING: Unfortunate terminology:
0 Do not confuse capital expenditures (the act of capitalizing an asset) with capital assets
(which get special treatments on their tax rate defined in §1221)
o Not connected concepts! Do not confuse them!
1 What must be capitalized is an enormous battle ground:
If a deduction is given depends on whether an expenditure is covered by §212/162 or §262
When a deduction is given depends on whether an expenditure is covered by §162/212 or
§263
0 TP will want to deduct from his income the amount he expended in order to
generate income in the year he spends the $$. Deduction here acts as a
deferral mechanism (recall: interest free loan from the gov’t.)
56
WNM
Tax 1 Ring (Fall 2017)

Ordinary & Necessary Business Expenses because


it will continue to generate income for many
years.94
0.0 Sale (or disposition)
0.0 Ex] Land TP buys in Y1 will continue to produce income
for as long as TP owns it. Unlike a machine, land does
not typically depreciate in value. Therefore, a point of
sale or disposition is required for a reduction.
1 Policy Rationale: The goal of a business deduction is to match income with the
cost of running a business.
Distinguishing capital expenditures (§263) and current deductions (§162)
0 car “Bright Line Test” for a capital expenditure
0.0 If the expense is less than $200 OR lasts less than 12 months it is a repair
deductible under §162
0.1 If the expense is for a betterment to property, restoring the property, or
adapting the property it is a capital expenditure under §263 [Reg.
1.263(a)]
1.0 Betterment:
0.0 TP is spending $ to fix a material defect
i. Mt. Morris Drive-In (defect in the drainage system)
0.1 Improvement reasonably expected to increase the
strength, quality, efficiency, or output of
property; or
0.2 Major addition (e.g. addition to a house)
1.1 Restoration:
1.0 Ex] TP spends $ to restore something to a
functional state after it has dropped into disrepair.
1.1 Ex] TP lost something (e.g. hurricane destroyed part of
home) and then paid to restore it; or
1.2 Money spent on a “major component” of property (what is
“major”?)
1.2 Adapting: Money spent to adapt to the property to a new purpose
2.0 Ex] a warehouse is converted into a showroom
1.3 Exceptions to §263:
3.0 Routine maintenance (small, regular repairs)
3.1 Small Taxpayer Safe Harbor (spend less than the lessor of
2% AB or $10K)
3.2 General Safe Harbor (sets $ thresholds)

Repair vs. Improvement


Repair: Costs of incidental repairs and maintenance of property are currently deductible.
Ex] Midland Empire Packing: the concrete lining to oil-proof a basement
Improvement: Costs that materially add to the value of property or substantially prolong its useful
life are nondeductible capital expenditures
Ex] Mt. Morris Drive-In Theatre Co: New drainage system for drive-in theater was an improvement and
96
thus a capital expenditure therefore no deduction is given
o Increases value
o Permanent improvement to property o Prolongs life of asset
Contrast: Wages and electric bills are considered recurring outlays that can be deducted in the same
year. The electric bill you pay in January to keep the lights on in your office only goes towards producing
income for your business in that one month.
Mt. Morris Drive-In Theatre Co. v. Commissioner: capital expenditure due to improvements
0 Facts: ∂ was forced to build a drainage system because the construction of the movie theatre had
resulted in flooding of the neighbor’s land. Issue was whether the $8,224 spent by the company in
1950 to construct a drainage system was deductible as an ordinary and necessary business
expense under §162 or as a non-depreciable capital expenditure under §263.
1 Holding: Both the tax court and 6th Circuit found that the drainage system was a capital
expenditure under §263. It was foreseeable that the company would have to create the drainage
ditch when they originally remodeled the land to create the drive-in movie and had they done it at
the same time as building the drive-in movie, it would be considered a capital expenditure. The
drainage ditch also adds value to the property, so it falls in §263.

57
WNM
Tax 1 Ring (Fall 2017)

was a repair and thus deductible under o Foreseeable at outset; something you knew about at
95
§162 purchase (not an “unforeseeable external factor”) → bought
• Maintenance land knowing about drainage problem.
• Completed to appease inspectors
• Unforeseeable, external problem

Contract Rights:
0 Ex] TP buys a fire insurance policy that protects his home for 3 years
0.0 Capital expenditure (3 years of protection)  not deductible in the
year the policy was purchased.
Intangibles:
0 Intangible assets that are the result of a business expense  do not necessarily
fall under §162
0.0 Court ruled that merger-related expenses are capital expenditures
because they reap huge long-term benefits [INDOPCO]97
0.0 IRS came out with “anti-INDOPCO” regulations in 2003 that are
more favorable to TO and allow for the inclusion of banking fees,
lawyers’ fees, etc. under §162
0.1 Self-constructed assets (creating an asset yourself instead of buying)
are capital expenditures because the IRS does not want to give
someone who builds their own stuff better tax treatment than someone
who contracts out to do it.
1.0 HYPO:
0.0 Scenario 1: TP buys a bulldozer for $100. The bulldozer
lasts 5 years + helps the TP earn income for 5 years on
various construction projects.
0.0.0 Capital expenditure → recover $20/year
over 5 years
0.1 Scenario 2: In year 1, TP uses the bulldozer to build
his own home (that has life of 40 years)
0.1.0 No $100 deduction in Year 1; depreciate
$20 over 40-year life of the house.
0.2 Intangibles are folded into your basis in the business so when you sell
it, you get back the expenses attributable to them
§263A: specifies what other expenditures have to be capitalized
0 ex] inventory to sell to customers

Capital Recovery
Determination of when TP gets to recover under §263
0 Worst case scenario: TP can only recover at the point of sale (could be a long time
)
Two separate inquiries
0 Did you suffer a loss?
1 Is the loss deductible? (consider limitations)
§165(a): generally, TP can deduct any business loss sustained during the taxable year

Midland: repairs are not capital expenditures


0 Facts: Midland ran a meatpacking company adjacent to an oil business that had been leaking oil
into the groundwater and Midland’s storeroom. Midland put in a sealant on the cellar floor to
protect it from water/oil seeping in and wanted to deduct it in the same year as a §162 ordinary
and necessary expense.
1 Holding: The tax court held that the sealant was essentially a repair so it was deductible as a
business expense under §162. The purpose of the sealant was only to permit Midland to
continue the use of the plant and basement for its normal operations, not to prepare the plant
for new or additional uses.
INDOPCO, Inc. v. Commissioner – resulted in the “anti-INDOPCO” regulations
0 Facts: INDOPCO was acquired by another Delaware corporation and hired firms (lawyers and
investment bankers) to facilitate the acquisition, then wanted to deduct the cost of the fees to
those firms under §162.
1 Holding: The tax court, circuit court, and SCOTUS all held that the expenditures were capital in
nature so they belonged in §263. The costs of the professional services contributed to future
benefits for the company and made these expenses an intangible asset, i.e., an outlay of the
business that you could only recover the cost of once you sold the business. Additional uses.
58
WNM
Tax 1 Ring (Fall 2017)

Losses
§165(c): TP can not deduct individual losses unless:
0 Losses occurred in trade/business
1 Losses were not connected to a trade/business, but were incurred in profit-seeking
transaction
2 Losses were not connected to a trade/business, but arise from theft or casualty loss
2.0 Ex] fire, storm, shipwreck, etc.
3 HYPO: TP buys house for $100k.
3.0 Scenario 1: Highway built nearby, sell at $30k loss.
0.0 Do not deduct → personal asset; does not matter if the highway
caused loss
0.1 Treas. Reg. §1.165(9)(a): Houses go down in value
because of personal consumption
3.1 Unless TP is in the trade/business of buying/selling
homes (relator  business loss) he does not get a
deduction
Scenario 2: If TP sells the house at a $150k → taxed
0 Well, that seems kind unfair

LIMITATIONS ON THE RECOVERY OF CAPITAL


Requires a realization event
0 §1001: an exchange gives rise to a realization event only if the properties are
“materially different” (i.e., legally distinct entitlements, different interests,
different properties, etc.)
0.0 Types of realization events:
0.0 Sale
0.1 Exchange
1.0 ex] Cottage Savings: Exchange of mortgage interests by
two banks triggers a realization event; the interests are
98
legally distinct entitlements.
0.2 Seizure of assets
2.0 Ex] S.S. White Dental: seizer by a foreign gov’t. of
property was a realization moment for the purpose
99
of recovery of a capital asset
1 Capital losses: loses on the sale of capital assets
1.0 §1211: Limited deduction are allowed for losses on the sale of capital
assets
0.0 Corporations  can only offset capital losses against capital gains
0.0 Carryover: Back 3 years or forward up to 10 years.
0.1 Corps. do not get lower interest rates on capital gains
(therefore a deduction is worth more to a corporation than
to an individual)
0.1 Individuals  if capital losses exceed capital gains, TP can
offset those losses against capital gains, AND against
ordinary income, capped at $3,000

Cottage Savings Association v. Commissioner:


0 Facts: During the savings and loan crisis, two S&L entities’ assets had decreased in value and the
S&Ls wanted to trigger a loss for tax purposes (wanted to reduce other income because they had
already lost the money) but didn’t want to have to report the losses to the banking regulatory
agency. The S&Ls exchanged interest (didn’t sell the whole pool of their loans, but exchanged
some of them because an exchange is considered a disposition) and argued that it was a sale
under §1001 (sale, exchange, or disposition that triggers an AR-B=GI analysis). The IRS said this
didn’t count as a realization event because the entities being exchanged did not materially differ in
economic substance, in fact they were basically identical!
1 Holding: SCOTUS ruled that the IRS was correct that the assets had to be materially different
but that the IRS’s threshold test was too subjective. The new test is whether they are “legally
distinct.” Since the two pools of loans included different loans and different borrowers, they
met the criteria for §1001 and triggered a loss.
S.S. White Dental: realization moment for the purpose of recovery of a capital asset
0 Facts: Taxpayer owned a business in Germany before WWI. During the war, his assets were
seized, and taxpayer wanted to be able to deduct the value of the assets like a forced sale. IRS
argued that it was just a fluctuation in value because it wasn’t clear that the taxpayer wouldn’t
get it back.
1 Holding: Court found in favor of the taxpayer because it was a clear disposition and loss.
Seizer in this context was enough. In the future, if the taxpayer got his business back or got $
from the German gov’t. he would have to treat it as income.

59
WNM
Tax 1 Ring (Fall 2017)

2 Policy: Gains on capital assets can have a lower tax rate than other assets,
but loses on these assets are restricted. The ability to use a loss on a §1221
asset as a deduction is limited!
2.0 There is a very high risk of planning potential with capital assets.
Owners of capital assets will maximize their tax benefits by
selling at certain times.
c. Personal losses
3 HYPO: TP buys a house for $100,000. 2 years later TP sells for
$70,000. Can TP take a $30,000 loss?
3.0 Pizza analogy: if you bought a pizza for $10 and ate ½ and then
sold the remaining ½ for $5 are you at a loss? No, you ate $5 worth
of pizza and sold the rest. Similarly, the value of the house dropped
because TP used the house.
4 HYPO: What if property values dropped because town zoning codes
changed?
4.0 Does not matter. The Pizza analogy still applies.
5 HYPO: What if TP sells the house for $120,000? The $20,000 in gain
is taxed! Admitted there is a lack of symmetry! “But, eh!” says the tax
system.
Gambling losses
0 §165(d) and Reg. 1.165-10: TP can only offset gambling losses against
gambling winnings in the same year
Sale to a related party
0 §267(a): Loss is disallowed if a property is sold at a loss to “related parties”
100
0.0 ex] Siblings, spouse, lineal descendants, ancestors
0.1 broad definition of property (anything TP sells)
1.0 HYPO: TP buys a car for $1000 and sells it to his
brother for $400. TP wants to deduct $600 from his
income on his tax return.
0.0 No deduction for TP!
1 §267(d): If property is transferred to a family member and that family
member sells that property at a gain → look at the family gain as a
whole.
1.0 Policy:
0.0 The sale price might not be reflective of the FMV.
0.1 The property “remains within the family.” The seller is not
truly parting with the asset. He may still have use of or
gain benefit from the property.
1.1 HYPO:
1.0 Scenario 1 (Gain): ß pays $1,000 for an asset and sells it
to his brother, Ω, for $900. Ω sells for $1,500. Although
Ω “realizes” $600 gain, he only has to REPORT a $500
gain (AR– B = $1500 - $1000 = $500) because we are
looking at the family as a whole/single unit in terms of
basis.
1.1 Scenario 2 (Loss): if Ω sells the asset at a loss ($300), ß’s
loss does not get added to Ω’s loss.
1.0 AR – B = $900 - $300 = $600
Wash sale: Buy stock → sell at loss → buy back
0 §1091(a) (Loss): Loss disallowed if TP buys/sells stocks within 30 days (on
either side)
1 §1091(d) (Basis): Basis of new stock (repurchased stock) = basis of
stock sold + (price of new stock – sale price of old stock)
1.0 HYPO: $100 basis in stock, sell for $90 ($10 loss) and buy back for
$85 in 2 days
0.0 Loss of $10 → Disallowed (within 30 days)
0.1 Basis: 100 + (85 – 90) = 95
2 Intrafamilial wash sale
2.0 McWilliams101 TP → marketplace → TP’s wife

NOTE: “related parties” is a term of art! (see §118 or §318 as example of how a brother is NOT a “related party”)
McWilliams v. Commissioner – intrafamilial wash sales
0 Facts: Taxpayer sold stocks in the marketplace to trigger a loss, then bought them back from the
marketplace using his wife’s account and wanted to apply §267.
1 Holding: Court disallowed the loss under §267 and treated the transaction as if the husband had
sold directly to the wife. However, most courts don’t do this unless this is real evidence of related
parties dealing and now kicks it over to §1041 (transaction between spouses).

60
WNM
Tax 1 Ring (Fall 2017)

0.0 Disallowed under §267 → as if marketplace never


happened
1.1 Alternative: Disallow under §1091 → husband/wife acting a single
unit
1.0 §267 kicks over to §1041: No G/L between spouses
g. Illegal activity
102
2 Mazzei v. Commissioner
3 Issue: TP robbed while conspiring to engage in forgery → seeks §165(c)(3)
theft loss
3.0 Majority: No loss → public policy: don’t reward someone engaged
in illegal activity
3.1 Concurrence: Not calculating tax income; voluntarily gave $$
away
3.2 Dissent: Loss! No room for public policy
I buy a car for $17,000. After a year I sell it for $12,000. Do I get to take a $5000 loss on my return?
Why? Why not?

Probably not. A car, much like a house or a pizza, decreases in value due to the owner’s use. The tax system
assumes that the dip in the car’s value is a result of the owner’s use of the car (i.e., wear and tear). Therefore, the
owner would not ordinarily get a deduction on the loss of $5,000 that he incurred upon the sale.

Bad Debts
In general:
0 HYPO: Bank lends $10,000 to Ω and Ω defaults on the loan, but Bank is able to
recover $2,000. Can Bank deduct the unpaid $8,000 as a bad debt?
0.0 It depends!
Business Debt: ordinary deduction
0 §166(a): When the lender cancels a bad debt because it is clear that the borrower
won’t be able to pay it back, the lender can deduct the loss during that year)
1 Employees: Being an EE is a trade or business
1.0 If an EE makes a loan to the company in order to keep his job  EE can
deduct the loss if the company defaults on the loan.
1.1 Exception: If TP is both an EE and an owner  capital loss
1.0 Generis and Whipple
2 Managing a business:
2.0 Managing a business ≠ having a trade or business
Non-business Debt:
0 §166(d) if the lender is an individual, and:
0.0 Borrower is a business  the lender can deduct
0.1 Borrower is a non-business  the lender must treat the bad debt as a
capital loss
1.0 Rationale: we are concerned about personal loans (e.g., ß loans
his brother $10,000 which his brother gambles away)

Depreciation
In general:
0 If an item (e.g., machine, vehicle) is depreciable, TP can deduct a percentage of
103
his basis in the item every year.
0.0 §167: In order to be depreciable, the item must be used in a “trade of
business” for the purpose of generating income.
STEP 1: Is it depreciable?
0 If the item is a §263 capital expenditure, ask: When can you use (“take”) the
deduction?

Mazzei v. Commissioner – illegal activity (no deduction)


0 Facts: Mazzei ran a counterfeiting scheme and brought $20,000 to NY, but lost it all in a fake
raid. Wanted to be able to deduct the $20,000 as a business loss. The IRS disallowed the loss on
the grounds that allowing it would be counter to public policy.
1 Holding: Court upheld the IRS’s decision since Mazzei’s conduct constituted an attempt to
counterfeit and allowing the loss deduction would constitute an immediate and severe frustration
of the clearly defined policy against counterfeiting obligations in the U.S. Code.
Note: some times (e.g., land, art, stock) are not depreciable and can only be realized at sale (or disposition)
61
WNM
Tax 1 Ring (Fall 2017)

i. Sale (do not depreciate)


1. Except perhaps landfills which have a limited useful life
ii. Over time (depreciation) → match the cost with income it generates
b. Focus on TP’s use in his “trade or business”
i. Simon: a professional musician’s bow is depreciable property. Although the value of the bow
increases as an antique, its value to a musician’s trade or business decreases due to wear and
104
tear.
2. STEP 2: Calculating Basis
a. RULE: Purchase price is basis (NO adjustment for inflation)
3. STEP 3: Depreciation methods
a. §1016(a): Reduce (adjust) the TP’s basis when he takes a deduction for depreciation expenses
b. Depreciation methods:
i. §168 Tangible Assets:
1. Recovery period  looks at the functional life of the asset historically. This was
done based on the individual, but now there are broad categories of assets and uses
within the regulations (e.g., residential apartment buildings)
a. Typically calculated as being shorter than the true useful life (policy choice
by Congress to make it easier on the TP  shorter the window, the sooner
you get the deduction)
2. Statute dictates which deduction method will be used
a. §168(b)(3) Ratable: straight line across the recovery period.
i. HYPO: Ω purchases a depreciable item for $2,000 with a recovery
period of 5 years.
Y1 Y2 Y3 Y4 Y5 Y6
Remaining Basis $2,000 $1,600 $1,200 $800 $400 $0
Amount Deductible $400 $400 $400 $400 $400 –

b. §168(b)(1) Double declining balance


i. Calculate the ratable deduction, double it, and then subtract that
figure from the balance every year.
ii. Can’t keep doing every year because you would never get down to
0, so check to find the point when it is better to switch to the
straight line method and then switch
iii. HYPO: Ω purchases a depreciable item for $2,000 with a recovery
period of 5 years.
1. Ratable rate = 20%; DD Rate = 40%
Y1 Y2 Y3 Y4 Y5 Y6
Remaining $2,000 $1,200 $720 $432 $216 $0
Basis
Amt. Ded. (R) $2000/5yr $1,200/4yr $720/3yrs $432/2yrs $216/1 –
= $400 =$300 =$240 = $216 = $216
Amt. Ded. (DD) 40% of $2000 40% of $1,200 40% of $720 40% of $432 – –
= $800 = $480 = $288 = $172.8
ii. For intangible assets (§167)
1. Intangibles with a definite life like insurance (at the end of a set number of years,
they no longer add value to the business), depreciate in a straight line over the course
of their definite life

Simon v. Commissioner – depreciation of a capital asset


0 Facts: Simon was an orchestra performer who bought violin bows for their tonal quality, not their
monetary value, for regular use as his professional equipment. He claimed a depreciation
deduction on his tax return that applied to 5-year property, The IRS argued that the useful lives
of the bows were indeterminable because the bows were treasured works of art and would
appreciate in value as antiques. Simon countered that the bows were necessary to his profession
and suffered wear and tear attributable to their use in his profession.
1 Decision: Court ruled in Simon’s favor and held that they fell within the meaning of §167
because they would depreciate over the course of their useful life (the period over which the
taxpayer could reasonably expect the asset to be useful to the taxpayer in his trade or business
or in the production of his income).
62
WNM
Tax 1 Ring (Fall 2017)

2. §197: Intangibles without a definite life amortize over 15 years


a. §197(d) lists types of intangible assets (e.g., workforce in place, goodwill,
etc.)105
b. Pros: Good for the taxpayer because it replaced the historical requirement
that you had to wait until the sale to deduct the basis
c. Cons: Bad for the taxpayer if the asset actually has a recovery period of less
than 15 years
3. Can only recover in a straight line (ratably)  double depreciation is limited to
§168
a. Ratable: straight line across the recovery period.
i. HYPO: Ω purchases an intangible asset for $2,000 with a definite
life of 5 years.
Y1 Y2 Y3 Y4 Y5 Y6
Remaining Basis $2,000 $1,600 $1,200 $800 $400 $0
Amount Deductible $400 $400 $400 $400 $400 –

iii. HYPO: TP purchases a restaurant with $505,000 in tangible assets and


additional intangibles (i.e., customer goodwill, etc.) for $600,000.
1. Look at the tangible depreciation first for the $505,000 under
§168 and decide whether to use the straight line or double
declining method
2. For the remaining $95,000, since goodwill is not finite like an
insurance policy, you amortize it over 15 years under §197

Business/Personal Borderline
Analysis:
0 Look at the motivation behind the expenditure? Where did it originate?
1 Categorize it
2 Look for the bright line tests within the category
3 Allocate costs that are deductible

Childcare
106
§21: credits 35% of employment related childcare expenses.
0 The percentage of the credit (which starts at 35%) decreases 1% for every $2,000
the TP’s adjusted gross income exceeds $15,000.
107
1 The decrease caps at 20%.
2 What is employment related childcare?
2.0 Care for the TP’s dependents (those under the age of 13 OR in need of
constant medical care)
2.1 Qualifying kinds of care:
Employment related Not-Employment related
• Nannies • Bartender
• Maids • Chauffer
• Day care • Overnight camps
2.2 Maid who doubles as a chauffer (dual purpose)
The amount of childcare spending that is creditable is capped. [The percentage of
the credit is capped AND the amount of the creditable spending is capped.]
0 Regardless of how much TP actually spends on childcare
1 $3,000 credit cap for 1 child
2 $6,000 credit cap for 2 or more children

Note: land can never be intangible asset


Credits are better than ordinary deductions. Tax credits provide a dollar-for dollar reduction of your
income tax liability. This means that a $ 1,000 tax credit saves you $1,000 in taxes. On the other hand, tax
deductions lower your taxable income and they are equal to the percentage of your marginal tax bracket.
Essentially, you are being guaranteed a 20% credit.
63
WNM
Tax 1 Ring (Fall 2017)

0.0 TPs cannot deduct/credit more than the lower wage parent earns.
0.0 Rationale: if the lower wage parent is not making enough to
afford the childcare, they should be providing the care
themselves.
0.1 Exception: unless the lower wage parent is a full-time student
§129: Dependent Childcare Assistance Program:
0 Similar to a cafeteria plan; offered through TP’s ER.
1 Allows TP to exclude from income up to $5,000 spent on childcare
1.0 The tax benefit comes through the ER. (ER does the
paperwork and verifies the documentation)
0.0 NOT the same as ER provided childcare.
TP can use both §21 and §129
0 However, there is a tradeoff.
0.0 The amount TP deducts under §129, reduces the cap in §21.
0.0 If TP excludes $5,000 under §129 and has one child, the cap for §21
is reduced to $0; if TP has two or more children, the cap is reduced
to $1,000
0.1 If TP excludes $2,000 under §129 and has one child, the cap for
§21 is reduced to $1,000; if TP has two or more children, the
cap is reduced to $4,000
0.1 General rule:
1.0 If the taxpayer’s tax rate is low, it is better to use only §21
1.1 If the taxpayer’s tax rate is high, it is better to use §129
HYPO: Parents with one child make $180,000 and spend $15,000 on childcare.
0 Percentage credit is reduced to floor at 20%
0.0 At the 1% decrease for every $2,000 the TP’s income exceeds $15,000 
TP would have an 82.5% decrease which is more than the 20% cap
1 If TPs have one child:
1.0 Credit is capped at $3,000. 20% of $3,000 = $600 credit.
1.1 If they spend $2,000 on childcare, they only get a credit for 20% of the
$2,000 (not the full $3,000).
2 If TPs have 4 children and spend $15,000, they get the credit for 20% of $6,000
($1,200).
HYPO:
0 TP1: $100,000 GI; 40% rate; 1 kid; $5,000 cost
0.0 Credit: $600 (20% x 3k)
0.1 §129  5K exclusion worth $2k
1 TP2: $45,000 GI; 15% rate; 1 kid; $5,000 cost
1.0 Credit: $600
1.1 §129  $5K exclusion worth $750
2 TP3: $30,000 GI; 10% rate; 1 kid; $5,000 cost
2.0 Credit: $810 (27% x 3K)
2.1 §129  5K exclusion worth $500

Clothing
§162: only deductible as a business expense if:
0 ER requires the EE to wear that particular clothing
1 The clothing isn’t suitable to wear outside of work
1.0 Pevsner: lays out an objective test: could the clothing item generally be worn
outside of work? We do not care whether the TP herself would have worn the
108
items outside of work.
Pevsner v. Commissioner – clothing
0 Facts: Taxpayers managed a Yves Saint Laurent and was expected to wear expensive clothes
to advertise the product. She deducted the cost of the clothes as an ordinary and necessary
business expense under §162(a). The IRS disallowed the deduction, but the tax court ruled she
was entitled to it.
1 Holding: The 5th Circuit reversed the holding of the Tax Court because she could feasibly wear
them outside of work. The court used an objective, not subjective, test of adaptability.
Although many expenses are helpful or essential to one’s business activities (e.g., commuting
expenses, cost of meals at work), these are inherently personal and are disallowed under §262.
Clothing is a business expense only if it’s specifically required as a condition of employment, it isn’t
adaptable to general usage as ordinary clothing, and it is not worn as ordinary clothing.
64
WNM
Tax 1 Ring (Fall 2017)

2
TP has not worn the clothing outside of work
109
2.0 Examples: clown costume; fire fighting gear; hazmat suit
What about ER provided clothing?
0 If the ER’s logo is on the clothing and TP is not allowed to wear it anywhere else,
TP might consider it a §132(d) working condition fringe
0.0 if TP bought the clothing  TP could have deducted it
1 It depends on what the uniform is:
1.0 if it’s a $15 shirt, it’s probably a de minimis fringe
1.1 if it’s $500 suit, probably not
2 HYPO: firefighter’s uniform; in part provided by the ER and in part the EE is
required to go out and buy it. It is entirely deducted.
2.0 The part provided by the ER is deductible as a §132(d) working conditions
fringe.
2.1 The part bought by the EE is deductible under §162 trade or business
expense.

Travel & Moving Expenses


Travel
§162(a)(2): provides a deduction for money spent on reasonable and necessary traveling
expenses (i.e., hotels and meals) when the TP is away from home in the pursuit of a
trade or business
0 ex] attorney working in NYC is sent to SF for a three week trial  travel expenses
deductible
1 Note: failure to satisfy any element destroys the deduction
2 Travel for work:
2.0 “Away from home”
2.1 Deductible only if the TP is far enough from home such that an overnight
stay or substantial rest is required (focuses on time, not distance)
[Correll]110
3 Commuting expenses:
3.0 Generally not deductible  the choice of where to live is generally personal so
if TP wanted to TP could avoid the costs of a daily commute by living closer to
111
his workplace [Flowers]
3.1 Exceptions:
1.0 if TP gets an assignment outside of his municipality  could be
deductible
1.1 Driving around to see patients  deductible
4 If TP has to pay for transportation during the job because of the job (e.g.,
TP is a traveling salesman), the costs are deductible
4.0 What if TP uses her home as a full-time business location? Not having a
home office, but completely running your business out of your home.
0.0 Then travel from home to anywhere business-related is deductible
Temporary employment  taxpayers engaged in temporary jobs are accorded special tax
treatment in which the taxpayer is considered to be in “travel status” and travel expenses
paid or incurred in connection with the temporary assignment are deductible

Scrubs? Flight attendant uniforms? Apparently people wear these outside of work? Lol wtf?
Correll v. Commissioner – “away from home” clarification for travel expenses
0 Facts: Taxpayer was a traveling salesperson who left the house early in the morning and would
have breakfast and lunch on the road, then come home for dinner. Wanted to be able to deduct
the cost of breakfast and lunch he ate on the road everyday.
1 Decision: The court disallowed the deduction under §162 because even though he was in the
pursuit of business, he was not sufficiently “away from home.” Even though he travelled far, he
never travelled so far that he needed to stay overnight. The court established that day trips are
not enough.
Commissioner v. Flowers – travel expenses
0 Facts: Taxpayer lived in Jackson, MS but was unwilling to move to Mobile, Alabama for his job.
He made an arrangement with his employer whereby he could continue to live in Jackson on the
condition that he pay his travelling expenses. On his tax return, he wanted to deduct the
expenses incurred in making trips between the two. The tax court disallowed the deduction.
1 Decision: The court affirmed the tax court’s decision and defined his “home” as the place where
he worked. Put that way, Flowers was not actually travelling for work. There was no necessary
relationship between the travel expenses and the Flowers’s employer so they were §262 personal
expenses. He had incurred the expenses because of his personal preference for staying in Jackson.
o Away from “home” not away from work.
65
WNM
Tax 1 Ring (Fall 2017)

2 But travel back “home” from a temporary job is not deductible [Hantzis]112
HYPO: Monday through Friday, TP works in Boston. On the weekends, TP works a different
job in Maine. Are his meals and lodging in Maine on the weekend deductible?
0 NO! Boston job isn’t sending TP to Maine; TP is choosing to have two separate
jobs; TP has a fulltime job in Boston; he travels away from Boston for the
pursuit of a different trade or business; Maine is a travel destination!
1 YES! No requirement of that travel must be for the same trade or business.
There is a business connection to both places!
For a two-worker family:
0 Each spouse has a tax “home” and when one of the spouses is required to travel
some distance to their employment, the fact that the other spouse has a tax home in
the same city doesn’t convert the first spouse’s travel costs into deductible travel
expenses
1 The only way this works is if at least one spouse has a job in two cities  only way
to make travelling back to his/her family can qualify as a business expense.
2 HYPO: Husband and Wife live in NYC. Then Wife takes a job in DC. Wants to
take a deduction for her apartment and meals.
2.0 Analyzed person by person, not family unit.
2.1 Her only job is in DC. She should thus live there.
Temporal element:
0 Must be a post that lasts less than a year!
Meals: even when a §162 deduction is met, §274 could still cap the deduction.

Moving Expenses
§217: allows for a limited deduction of moving expenses
0 Applies to:
0.0 EE changing jobs with the same employer,
0.1 EE entering the workforce for the first time,
0.2 EE starting a job with a new employer, and
0.3 Self-employed people starting a new trade or business or moving to a new
location
1 Requirements:
1.0 New place of work has to be at least 50 miles farther than the TP’s former
residence
1.1 Only applies to permanent or indefinite (not temporary) moves 
individual has to be employed full-time for at least 39 weeks during
the year following the move
1.0 Rationale: sneaky old people trynna move to FL
2 The direct expenses of a move are fully deductible (cost of transporting the
individual and his family, household effects, and reasonable lodging en
route), but meals are not.
§82: employer reimbursement of moving costs is taxable
0 Exception: working condition fringe benefit allows TP to exclude the
reimbursement if it covers expenses that would normally be deductible under
§217

Travel & Entertainment


Outlay must first meet §162. Then check §274 for restrictions.
§274(a): activities and entertainment are deductible if directly related to business
(expense occurred immediately before/after a substantial meeting)
0 ex] dinner following a Board meeting
1 ex] take a client to a bar to discuss the case

Hantzis v. Commissioner – temporary employment


0 Facts: Hantzis was a 2L from Harvard who got a summer job in NYC but lived in Boston and
wanted to deduct the cost of travel between Boston and NYC (cost of apartment, meals, etc.)
because she had a husband in Boston. IRS said no because she didn’t need to travel back to
Boston for business; once she took the job in NYC, that’s where her home was.
1 Decision: The court affirmed because there was no business reason to duplicate the
expenses. The heart of the §162 deduction for travel and moving costs is that there is a
business reason for being in two places!
66
WNM
Tax 1 Ring (Fall 2017)

§274(d): “anti-Cohan” rule. Expense calculations must be accompanied by documentation 113


§274(h): attendance at conventions, cruises
0 §274(h)(1): Cannot deduct for expenses outside of North America or the
Caribbean unless TP provides a good reason
1 §274(h)(2): Conventions on cruise ships are restricted  if the expense is
deductible, it has to be for a convention held on U.S. ship leaving from a U.S. port
1.0 Note: the IRS will generally assume that TP went on the cruise convention
for fun
1.1 Policy: if we are giving you this deduction, might as well make sure
you’re patronizing US tax payers.
2 §274(h)(7): no deduction shall be allowed under §212 for profit seeking activities
(e.g., investing)
2.0 Policy: cannot turn your vacation into a deductible “investment” cruises
§274(n): caps what TP can deduct to 50% of the the expenses on meals and entertainment only
114
0 Meals aren’t always deductible [Moss]
1 Policy: There is no duplication of cost like there is for housing costs. If you’re
eating out in NYC, you’re not eating what’s in your fridge in Boston. Inherently
personal value. Admittedly, the 50% cap is arbitrary.
2 Examples:
2.0 Partner takes an associate for lunch to discuss their career  ok
2.1 Partner meeting once a month  ok
2.2 Partner takes an associate for lunch every day  not ok

Hobby Losses
Hobby losses (activities TP is not engaged in for a profit)
0 §183(a): generally, hobby losses are not deductible
1 §183(b)(1): TP can deduct things that would normally be deductible without
regard to whether the activity was engaged in for profit (e.g., casualty losses)

History:
0 Prior to 1962, the deduction of entertainment expenses was governed by the
ordinary and necessary standard in §162 making it relatively easy to deduct even
the most personal of expense expenses
0.0 Sanitary Farms Dairy, Inc. v. Commissioner – pre-§274
0.0 Facts: Taxpayers operated a corporation and the corporation officers made a
decision to send the executives on safari in Africa and use the footage from the
trip as a way to advertise the corporation. The corporation deducted the
amounts as advertising expenses. The IRS disallowed the deduction.
0.1 Holding: The court ruled for the taxpayers because it found that the trip was
meant to be for advertising so it met the ordinary and necessary requirement of
§162 and the safari was clearly for business purposes since the company made
good on their promise to use the footage in advertising.
1.0 Primary purpose view; court knows that the owners were
enjoying themselves, but they recognize that the owners got the
most for work. Perfect combo (thing you love + adv. for business)
1 Cohan: Judge Learned Hand ruled that where the TP had spent substantial sums on tax-deductible
entertainment but kept no account of the expenses, the TP was still entitled to the deduction.
1.0 Congress responded with §274, which placed substantial restrictions on the deductibility
of many items.
0.0 Remember, to get to §274, must still have to go through §162
Moss v. Commissioner – business meals
Facts: Moss was a partner in a law firm in which the firm’s lawyers met for lunch each day at a
nearby restaurant to discus matters related to the firm’s litigation. The IRS challenged these
deductions and the tax court disallowed them.
Decision: The court held that although meals can be deductible under §162(a), this particular daily
lunch was not a necessary business expense because the meal was not an organic part of the
meeting to coordinate the work of the firm and the firm didn’t need the lunch to cement its
relationship within the members. To determine if business lunches are deductible is a matter of
degree and circumstance and frequency  daily is too much, even if you’re taking clients.
o No clients involved
o Daily is pretty frequent
67
WNM
Tax 1 Ring (Fall 2017)

0 Deductions you can get even if you are not in business (personal
loses not connected to business that are deductible elsewhere in the
Code)
§183(b)(2): TP can deduct the amount that would be deductible if the activity was
engaged in for profit only to the extent that the GI derived from this activity
for the taxable year exceeds any deductions allowed in §183(a)115
0 HYPO: TP spends $120 on his hobby and make $100 off of it in
income. Of that $120 TP spent, $10 is allowed to be deducted elsewhere
in the code.
0.0 TP: would want to subtract the $10 from the $120 first and then
deduct the $100 so TP could deduct $110 total.
0.1 IRS: TP must stack the other expense first and use the expenses
against his hobby income. Ultimately TP can only get a $100
deduction.
a. Forced to up  so truly capped at $100.
§183(d): we presume that if TP’s activity is profitable 3 out of 5 years, TP’s
hobby is actually a profit-seeking activity.
0 Special rule for horse-breeding: if profitable 2 out of 7 years  profit
seeking activity
116
0.0 [Bessenny] pre-§183 case

Legal Expenses
Must originate as a product of the business to be deductible under §162 or §212
0 Tangential effect on the business is not enough.
1 Focus on the origin of the claim, rather than the effects of losing the suit.
i. Gilmore117 and Patrick118: divorce is too personal. Look at the origin on the
claim.

Education Expenses
Deductible if:
0 (1) one maintains or improves a skill needed in business: updated training (e.g.,
CLE credits for lawyers), new practices, new technologies, etc. or (2) meets the
express requirement of the ER or the law for keeping a job.
1 Cannot be merely a minimum requirement for the job (e.g., having a bachelor’s
degree)
2 Cannot be training that qualifies TP for a new trade or business (e.g., law school
tuition)
3 HYPO: Accountant believes going to law school will better his understanding of
accounting. Once law school is over, he will return to accounting. Can he get a
deduction expense?
i. No, qualifies the accountant to enter into a new trade or business

Similar to how TP’s gambling losses can only be deducted to the extent of TP’s gambling gain
Bessenny:
0 Facts: Horse breeder wants to deduct the net cost of running her unprofitable business.
She’s a very wealthy woman and has other income. IRS was only willing to offset costs
against income earned.
1 Holding: she loses because all she talks about is the love of her horses and not about the profits.
2 Compare: Skiing is not the same as breeding horses because it cannot make any money.
Farming is a business. Occasional $ can be generated.
U.S. v. Gilmore – legal expenses
0 Facts: Gilmore wanted to be able to deduct legal expenses associated with his divorce and
argued that it should be governed by §212 as an ordinary and necessary profit-seeking profit
expense because if his ex-wife got too much in the divorce it could affect his reputation and he
could lose his job.
1 Decision: The court disallowed the deduction because the deductibility of the legal expenses
turned on the origin and nature of the claims themselves (whether they arose as a result of the
taxpayer’s profit-seeking activities), not whether they woul have a consequence on the
taxpayer’s profit-seeking activities.
Patrick – legal expenses
0 Facts: In the division of assets stage of the divorce proceedings, Patrick was in the middle of a
property settlement with is wife where he was fighting to keep his newspaper company and
wanted to deduct it as a profit-seeking expense under §212.
1 Decision: The court disallowed the deduction because the deductibility of the legal expenses
turned on the origin and nature of the claims themselves (whether they arose as a result of the
taxpayer’s profit-seeking activities), not whether they would have a consequence on the
taxpayer’s profit-seeking activities. Here, the claims still originated due to marriage, so they
were personal expenses under §262.
68
WNM
Tax 1 Ring (Fall 2017)

HYPO: City has a shortage of teachers and thus lowers the educational
requirements to teach to a bachelor’s degree.
i. Minimum education requirement. Therefore, cannot deduct the last year of
college.
HYPO: Business school  in business working for a company, take two years off to
go to business school, come back to the same company and position. (fact
specific)

Approaches
How do we draw the line between business and personal?
0 Motivational analysis (Sanitary Farms (pre-; 162)
1 Categorical answers
2 Bright lines drawn by the court (Correll; Pevsner)
3 Allocation (§183 Hobby Rules (Stacking approach))
4 Origin of the claim (legal expenses; Gilmer)

Personal Deductions
§262: do not deduct personal, family, or living expenses
a. Policy: if everything was deductible, there would be nothing left for the IRS to tax

Deduction No Deduction
Interest • Interest expenses by individuals (not used to • Personal interest (e.g., house, car, credit cards)
purchase capital assets) • Interest expenses related to tax-exempt activity (cannot
• Investment interest (up to the amount of deduct interest on debt incurred to purchase a tax
investment income) exempt bond)
• Home equity loans
• Mortgage debt on qualified residencies
Taxes • State and local taxes • Federal taxes
• Sales tax
• Property tax
• Foreign tax
Casualty • §165(c)(1) trade or business • Limitations:
Loses • §165(c)(2) profit-seeking endeavor • Loss must exceed $100
• §165(c)(3): some casualty loses are deductible • only permitted to the extent that it exceeds 10% of
even for personal losses the TP’s AGI
o natural disasters • Cannot be carried forwards or backwards
o accidents
o theft
Medical • Prescription drugs/insulin • TP does not receive a deduction if the expense is
Expenses • medical care (amounts paid for the diagnosis, reimbursed by insurance.
cure, mitigation, treatment, or prevention of
disease)
• Counseling (sometimes)
Charitable • Not if done quid pro quo (in exchange for something)
Contributions • Donating $$ to a single professor at a university
• Limitations:
• §170(b): caps the deduction to 50% of TP’s AGI
for individuals
• §170(c): Must be to an organization, typically a
501(c)(3)

Interest
Applies to interest on loans (i.e., on money that TP borrowed)
Business Interest [for individuals, not corporations]
0 §162: deduct interest expenses if they incur interest on their loans for the business
unless the loan is for a capital asset.
i. If it is a capital asset  §263
Investment Interest [borrowed to buy stock]
0 §163(d): for investment interest (e.g., borrowing money to buy stock), deduct up to
the amount of
69
WNM
Tax 1 Ring (Fall 2017)

TP’s investment income


0.0 Rationale: deductible as a business interest because TP cannot be in the
trade or business of investing [Yaeger]
0.1 Note: Does not have to be investment income from the same asset
Personal interest (house, cars, credit cards)
0 §162(h)(2): lists of expenses that do not qualify for deduction
1 Deductible expense:
1.0 qualified residences (mortgage debt) up to $1,000,000 (for up to two houses,
combined)
1.1 Home equity loan up to $100,000
Interest expenses related to tax-exempt activity [Arbitraged]
0 §265(a)(2): cannot deduct interest on debt incurred to purchase a tax-exempt item
0.0 Policy: Congress was concerned about TP borrowing to buy tax-
exempt bonds, then deducting the interest (double benefit that
creates a profit after the tax)
HYPO: TP has $600 and is looking to borrow money. TP would like to spend $300 on a
vacation; invest $500; and spend $400 on a new oven for his bakery. How should TP
borrow in order to deduct most favorably?
0 Divide the $600 in half and spend $300 of it on the vacation, and the other $300 on
the
1 investment.
2 Borrow an additional $600 and split it into $200 to go towards the investment (the
loan interest is deductible up to the gains made on your investment) and $400 to go
towards an oven for his business (fully deductible as a business expense, probably a
capital asset under §263)

Taxes
§164(a): Cannot deduct federal income tax from tax return, but can deduct other kinds of taxes
0 Types of deductible taxes:
0.0 State and local taxes
0.0 Sales tax (if your state doesn’t have an income tax)
0.1 Property taxes
0.1 Foreign taxes
1 Needs to be YOUR taxes (you can’t pay parents’ property taxes and get a federal
deduction)
1.0 Loophole: → “join” parents on property title and loan; make gift so they a
get deduction

Casualty Losses
§165(c)(2): (individual + business/profit-seeking) deduct individual losses incurred as part of:
0 §165(c)(1) trade or business
1 §165(c)(2) profit-seeking endeavor
2 §165(c)(3): some casualty loses are deductible even for personal losses

§165(c)(3): casualty loses are deductible even for personal losses


0 Rationale: Treated like a negative windfall. H-S perspective. If positive
windfalls are taxable, negative windfalls should be deductible.
1 Only deductible to the extent that TP did not have insurance coverage
1.0 Insurance: If TP has insurance and is reimbursed for the total amount of
the theft, he does not receive a deduction on the stolen amount.
1.1 No insurance: If TP does not have insurance, the deduction reduces his AGI
1.0 §165-7(b)(1): TP gets to deduct the lesser of his basis or the
decline in the FMV of the asset that was stolen/destroyed
0.0 HYPO: TP bought house for $50,000. Value declines to
$40,000 and is then destroyed in a fire ($0).
0.0.0 Deduction: $40,000 because that is the
value that is destroyed by the casualty event.
0.1 HYPO: TP bought house for $50,000. Value increases
to $60,000 and is then destroyed in a fire ($0).
0.1.0 Deduction: $50,000
70
WNM
Tax 1 Ring (Fall 2017)

1.1 Cannot deduct if the asset declined in value due to personal use

Types of casualties covered by the statute


0 Natural disasters (fire, storm, ship wreck)  Event must be sudden
0.0 Termite damage  no deduction
0.0 because they take a long time to cause damage
0.1 Property damage from summer of heavy rains  no deduction
1.0 because foreseeable
0.2 Property damage from a sudden flash flood  deduction
2.0 because unforeseeable
1 Accidents: (consider TP negligence/fault)
1.0 Carpenter: husband drops his wife’s wedding ring into the garbage disposal
 deductible
0.0 Quick, one swift movement119
1.1 Keenan: husband threw his wife’s jewelry away wrapped in Kleenex  no
deduction
1.0 Result of TP’s negligence/fault
1.1 Difficult to prove it was an accident
1.2 White: husband smashed car door against wife’s finger and the diamond on
her wedding ring was crushed  deduction
2.0 Proof because wedding band remains
2.1 Suddenness of the accident
2 Theft
§165(h) Limitations:
0 Loss must exceed $100
0.0 Immediately deduct $100 from each loss
0.1 Policy: want to avoid di minimis loses
1 Net casualty loss is only permitted to the extent that it exceeds 10% of the TP’s AGI
1.0 Subtract 10% of AGI from total casualty losses.
1.1 Low income TPs have an easier time getting casualty losses
1.0 Policy: a fire tends to be harder on the poor than the rich, so they
should get a better deduction (besides, the rich probably have
insurance)
2 Casualty losses cannot be carried forwards/backwards
3 HYPO: TP incurs $7,000 in damages to his damage, $50,000 in damages to his
house, and $60 in damages to his wedding ring.
3.0 Get $7,000 for car + $50,000 for house + NOTHING for ring (less than
$100) = $57,000
3.1 AGI $100k.
1.0 10% AGI = $10k → $57,000 - $10,000 = $47,000
3.2 AGI $ 400k.
2.0 $40,000  $57,000 - $40,000 = $17,000

4 Policy:
4.0 Gov’t. acting in part as an insurer because the TP does not have insurance.
The deduction is granted in proportion to the TP’s income so that if a loss is
really big for you, we are going to help out.
Carpenter v. Commissioner – casualty loss due to accident
0 Facts: Taxpayer wife placed her diamond ring in a glass of ring cleaner to soak and her husband,
not knowing what was in the glass, poured the glass’s contents down the kitchen sink and turned
on the garbage disposal, thereby damaging the ring. Taxpayer claimed a casualty loss on her
return.
1 Decision: The court allowed the loss as a casualty loss under §165(c)(3) because the evidence
showed that her husband would not deliberately and knowingly have put the ring in the
garbage disposal, so the event was inadvertent and accidental and the damage to the ring arose
from fortuitous events over which the taxpayer had no control.
71
WNM
Tax 1 Ring (Fall 2017)

Medical Expenses
§213: TP does not receive a deduction if the expense is reimbursed by insurance.
0 §§104 and 105 limit tax-free receipt of medical insurance benefits to payments that
are not attributable to deductions allowed under §213
§213(d)(1)(A): allows deductions to be taken for medical care (amounts paid for the
diagnosis, cure, mitigation, treatment, or prevention of disease)
0 Need not be paid to a licensed physician or healthcare provider or a hospital
1 Counseling is deductible if the treatment has a primary purpose of preventing or
alleviating a physical or mental defect
1.0 However, not everything the care provider recommends (e.g.,
shopping therapy) are deductible
§213(b): allows deductions for prescription drugs and insulin
0 Smoking cessation pill would qualify, but the patch would not because no
prescription is needed.
When medical expenses are paid with borrowed funds, the “amount paid” is deductible when the
TP becomes indebted to the third party, not when the loan is actually repaid (Rev. Rul. 78-
39, 1978-1)
Restraints on the deduction:
0 Cannot take the deduction unless the aggregate medical expenses exceed 10% of
TP’s AGI
0.0 favors low-income taxpayers
Deductible Lacking Bright Line Test Not Deductible
121
• injuries from a car accident • Plastic surgery → if to • Illegal medical treatments
• Meals and lodging that improve the structure or • Costs incurred for someone other
accompany in-patient hospital function of your body than TP122 (e.g., cost of boarding
care 1.213-1(c)(1)(v) (if medical • Acupuncture → if school for kids of sick parent)
care is principal reason TP is in recommended by a doctor • Shopping Therapy (Just because
the hospital) • Home elevator for heart your doctor says something is good
patient → To extent cost for you, important, or even
exceeds value added to necessary, does not mean it is §213)
120
home

Charitable Contributions
§170(a): Allows a deduction equal to the FMV of the cash or property TP donates to a qualified
charity.
0 Policy: charities provide social value, we want to encourage TP donations123
1 Limitations:
1.0 §170(b): caps the deduction to 50% of TP’s AGI for individuals
1.1 §170(c): Must be to an organization, typically a 501(c)(3)
1.0 Ex] Cannot be directed at a single professor at the University

Ask: Does the elevator raise TP’s home value?


The difference is a §213 expense if you paid more than the increase in home value
HYPO: Original Home Value ($100k), Elevator Cost ($80k), New Home Value ($140k) →
§213 deduction:
0 $140k - $100k = $40k
1 $40k - $80k = - $40k = DEDUCTION
In the regs.
DO NOT deduct care for children or school resulting from sick parent (Och)
Boarding School replacing them for her (job as Mother), and it has nothing to do with her directly;
it’s NOT for the patient.
Note: Had she gone to the sanatorium may have turned out differently.
Och is making a Pevsner style argument. But the outlay needs to be medical and connected directly
to the patient’s health. That was not the case with the boarding school.

Also covers religious entities. Donating to a charity, shouldn’t be viewed as consumption (H-S definition). We
did not accept this in the gift context, but we do recognize it here. Charities like higher tax rates because the
deduction rate is also higher. Ppl more likely to donate.
72
WNM
Tax 1 Ring (Fall 2017)

0.0 No deduction if the contribution quid pro quo → look at timeline (Dowell)
0.0 Dowell: Deduction allowed for a couple who accepted into a
retirement community before donating large amounts of $$ to the
community (easy standard to abuse)
a. Policy: A charitable donation implies TP is not getting
anything in return
How much is deductible?
0 Cash → deduct face value
0.0 Requires substantiation from the charity if the amount is more than $250
1 Services → cannot deduct
1.0 Rationale: valuation problem
2 Can deduct other related expenses (e.g., transportation costs, toll fees, etc.)
Property → complicated analysis:
0 §170(e)(1)(A) Ordinary Income Assets: if built-in gain would be
taxed at ordinary rates (ordinary assets and capital assets held for less
than year)
0.0 deduct basis
0.1 e.g., inventory, painting I made and sold, I’m not a dealer in stock
§170(e)(1)(B) Capital Assets: if built-in gain would be taxed at a preferential
capital gains rate (capital assets held for longer than 1 year)
0 Tangible personal property:
0.0 Used by the donee in its tax exempt function → DEDUCT
FMV*
0.1 Not used by the donee in its tax exempt function →
DEDUCT BASIS
1 Non-tangible personal property:
1.0 donated to a “private foundation124” → DEDUCT BASIS
1.1 not donated to a “private foundation” → DEDUCT FMV*

Ordinary
Income Deduct
Asset (e.g, Basis
inventory)

taxed at the Limited to


Held for < 1
OI Rate Basis
ANALYSIS
used by the
donee in its Deduct at
tax exempt FMV
Capital Tangible function
Assets Personal
Property not used by
the donee in Deduct at
its tax exempt Basis
function
taxed at a Donated to a
preferential Non- private Deduct at
Held for >1
rate Tangible foundation Basis
Personal Not donated
Property to a private Deduct at
FMV
foundation
Intellectual deduct at
Property FMV

Receive funding from a small group (e.g., family); highly regulated. We want to prevent families from
abusing these organizations to enrich themselves without doing actual philanthropic work that deserves a
deduction.
73
WNM
Tax 1 Ring (Fall 2017)

Tax Rate
0 Charities want a higher tax rate
0.0 HYPO: Charity A trying to convince TP to donate $1k.
0.0 If the tax rate is 70%, TP donates $1k to charity → $700 deduction
(costs TP $300)
0.1 If the tax rate is 30%, TP donates $1k to charity → $300 deduction
(costs TP $700)
1 HYPO: A donates appreciated stock to the law school  full FMV is deductible.
1.0 It’s a capital asset as long as A isn’t a stock-dealer. A has held the stock for
longer than a year. It’s not tangible personal property and it’s not being donated
to a private foundation.
2 HYPO: A donates medical antiques to the law school It’s a capital asset as long as A isn’t
a dealer in antiques. A has held the antiques for longer than a year. It’s tangible personal
property, so you would have to make an argument that the law school is using it for its
charitable purpose (teaching a course in medical law?). If it meets that hurdle, since it’s
not being donated to a private foundation, you could deduct the full FMV.
Fraud prevention: the amount TP deductions should be substantiated by the charity
Constraints on exempt organizations:
0 Ex] Bob Jones University (not a qualified charity because of its practices of racial
discrimination)
74
WNM
Tax 1 Ring (Fall 2017)

COMPUTATION OF
TAX
Tax Rates
The IRS adjusts certain things for inflation every year to be able to calculate your income in §151
0 note: not all numbers get adjusted for inflation
Different rate schedules apply depending on:
0 Who you are?
0.0 Married, head of household, or single
0.0 Head of household = unmarried person, with dependents
0.1 Single = not married, not head of household
0.1 Married people cannot file as single individuals  can file separate returns,
but the IRS treats the two returns as a joint return split into two parts
1 How much money do you make?
1.0 U.S. uses a progressive rate schedule  as you earn more $$, the next
chunk of $ is taxed at the next highest rate
1.1 HYPO: TP earns $300 for the year. The first $100 is taxed at 10%, the
second at 15%, and the third at 20%, rather than the entire $300 being
taxed at 20%
$100 10% $10
$100 20% $20
$100 30% $30
Total Tax $60

Calculating Taxable Income


STEP 1: Find gross income (GI)  §61 income
STEP 2: Calculate the “above the line” deductions125 under §62 and subtract that the
126
deductions from GI to find adjusted gross income (AGI)
0 §62 is a list of deductions that are otherwise provided in the code  the section
just tells you that now is the time to take them

Above-the-live deductions reduce a tax filer’s AGI (the line). Above-the-line deductions are sometimes also
called adjustments to income, because they generally represent costs incurred to earn income. In contrast,
itemized and standard deductions are sometimes referred to below-the-line deductions, because they are
applied after AGI is calculated to arrive at taxable income.

Policy Rationale: Why do we have “above the line” deductions?


GI v. AGI comparison:
TPA is a lawyer with a gross income of $100,000. TPB is a small business owner with a gross income of
$400,000 but business related expenses (e.g., EE salaries, rent) of $300,000.
GI System AGI System
Lawyer $100,000(.10) = $10,000 $100,000(.10) = $10,000
if the Lawyer’s medical expenses exceed if the Lawyer’s medical expenses exceed
$10,000 he can deduct them from his $10,000 he can deduct them from his
income. income.
Small Business Owner $400,000(.10) = $40,000. $400,000 - $300,000 = $100,00
If the SBO’s medical expenses exceed
$40,000, then only he can deduct them $100,000(.10) = $10,000
from his income. Under this system, if the SBO’s medical
expenses exceed $10,000 he can deduct
them from his income.
Under the GI system, the SBO needs to have huge medical expenses before he can get a deduction, while the lawyer gets
deductions sooner. But if we look closely, the two TPs are very similar individuals.
As you can see the AGI system is much more fair because it treats two otherwise similar taxpayers the same!

75
WNM
Tax 1 Ring (Fall 2017)

NOTE: the actual deductions are found elsewhere in the code!


Largely consists of highly politically motivated deductions

§62 Trade or Business deductions, limited by §274 Travel/Entertainment/Meals:


Trade or business deductions (NOT services by TP as employee)127
Certain specific trade or business deductions that employee pays for in
connection with his services as an employee
128
0 Reimbursed expenses of employees
1 Certain expenses of performing artists
2 Certain expenses of officials
3 Certain expenses of elementary and secondary school teachers
4 Certain expenses of members of reserve components of the Armed
Forces of the United States
Losses from sale or exchange of property
Expenses on property held for the production of rent or royalties
Depreciation: §62 if Trade or Business, §212(2) if on property held for rent
Moving expenses
Alimony
Interest on educational loans, §212
Higher education expenses

Above the line


Below the line

STEP 3: Subtract personal exemptions from AGI


0 §63(a) Personal exemption = deduction for everyone in the household
0.0 Helps to create the “zero bracket” for those who aren’t required to pay taxes
at all
0.1 Subject to a phase-out once TP’s income exceeds a certain level
1.0 In 2015, if TP is married, once TP’s income hits $309,900, his
personal exemptions of $4,000 per spouse/dependent begin to
phase out
1.1 By the time TP reaches $432,400 as a married couple, the
exemption completely disappears.
1 Decide whether to take a standard deduction or do §63(b) itemized deductions
and subtract them accordingly.
1.0 General rule
0.0 If you own a home, itemizing is usually worth it because of the
mortgage interest expense and property taxes
0.1 If you don’t own a home, you should probably use the standard
deduction
2 Standard deduction v. itemized deductions: If a standard deduction would be
more than itemizing your assets (putting them through the “egg” test)  then
choose the standard deduction.
2.0 Standard deduction = set amount set by the tax law every year
0.0 Can’t document; can’t find; If SD is higher than ID  take SD
0.1 You can still use the standard deduction even if you don’t have any
expenses
1.0 it isn’t considered lying or trying to subvert the system
0.2 2017:
2.0 $12,600 married filing jointly or surviving spouses
2.1 $6,300 for single tax filers and married filing separately
2.2 $9,300 for tax filers who qualify as the head of a household
2.1 Itemized deductions (aka, “below the line” deductions)  use the “egg”
test (work right to left)

Of the §162 deductions you can deduct them all except those of being an EE.
consist of expenses paid or incurred by the taxpayer, in connection with the performance by him of
services as an employee, under a reimbursement or other expense allowance arrangement with his
employer.
76
WNM
Tax 1 Ring (Fall 2017)

Note: because §§67 and 68 limit the amount you can deduct,
“below the line” deductions are less desirable (but obviously
something is better than nothing)
§67 governs the miscellaneous itemized deductions (right side of the
egg)
0 ex] trade or business costs of being an employee (e.g.,
paying to go to a conference), most investment
expenses, costs of preparing your tax return
1 A deduction is only allowed to the extent that the
aggregate of the expenses in this category
exceeds 2 % of your AGI.
1.0 HYPO: if the expenses are $110 and 2% of AGI
is $100, only $10 is deductible
1.1 Typically, very hard to meet and if you do,
it’ll be a very small number
§68 governs most itemized deductions (middle part of the egg over to
the right).
0 §68 overall limitation on itemized deductions.
0.0 To the extent a taxpayer’s AGI exceeds the
specified threshold (e.g., for a joint return, in 2017,
that would be $313,800) the taxpayer loses
itemized deductions (covered in §68) in an amount
equal to
3% of the amount by which the taxpayer AGI
exceeds the threshold.
0.0 HYPO: On a joint return in 2017, if
the TP’s AGI is $313,900, then the
TP’s AGI exceeds the threshold by
$100, so the taxpayer loses itemized
deductions equal to 3% x the excess,
i.e. $100= $3.
1 Also applies to §67 deductions to the extent that you got
anything after that hurdle
1.0 HYPO: the $10 that survived §67 is still subject to
§68
Only three deductions to survive both §67 and §68  they already
have internal limitations on your ability to take the deduction
0 §213 medical expenses
1 §165(c)(3) casualty losses and §165(d) gambling losses
2 §63(d) investment interest
3 Once your AGI hits a certain threshold, your itemized
deductions are phased out  doesn’t apply to a large
majority of TPs
3.0 Basic rule: for every dollar your AGI is over the
threshold, you lose 3¢ of your otherwise eligible
deductions (the two right sections of the egg)

Leverage
Nonrecourse Borrowing
If borrower refuses to pay or does not pay the loan, the lender’s only recourse is to seize the
specific property against which the borrower borrowed (borrower can satisfy the debt at any
time by turning over the property). The lender, for example, cannot seize or seek to claim
other property or assets of the borrower to satisfy the debt.
Borrowing Against Appreciation
0 HYPO: TP buys property for $5, which appreciates to $100. TP borrows $100
nonrecourse against the property to get a $100 loan to pay for his vacation to
129
Disney.
0.0 How should that borrowing be treated? Recall, that earlier in the semester,
we determined that borrowing money was not a taxable event. Why should
nonrecourse borrowing be any different?

HYPO: If after TP has taken out a nonrecourse loan on the property, it appreciates to $200 then TP is not
going to surrender the property to settle a $100 debt. Instead he might sell it and settle the debt of $100
and keep the remaining $100.

HYPO: TP borrows $100 NR to buy prop for $100. Then property drops in value to $70. So instead of paying
$100, he will turn over the property to the lender in satisfaction of his debt. The lender bears the loss (and the
risk) in a nonrecourse debt.
77
WNM
Tax 1 Ring (Fall 2017)

0.0 Well, you could argue that because the borrowing here is
nonrecourse, this transaction is more like a sale (with a call option)
and less like a real loan; in a sense, with a nonrecourse loan, the
borrower gets a $100, and effectively has only put the asset at stake.
0.1 The borrower can legitimately walk away and surrender only the
asset (thus this looks more like a sale) – and if the borrower
wants to “keep” the pledged property, the borrower just needs to
pay the $100 (this is the call option piece, the borrower can
reclaim the asset by paying the lender the $100).
0.2 Of course, unlike a “real” sale, the borrower is holding onto the
property and using it, until the borrower fails to pay the $100.
0.3 The bigger point, however, is that in the case of a nonrecourse
loan, the LENDER bears the risk that the pledged property will
decline in value – i.e. if the property that is currently worth $100
declines in value to $80, the borrower will logically decide it is
better to surrender the property (now worth only $80) instead of
keeping the property and paying the lender $100. Given this
change in who bears the risk with respect to the value of the
pledged property, it is not implausible to argue that this transaction
is more like a sale, and less like a traditional loan
1 Woodsam: Court held that taking out nonrecourse debt is not a realization
(taxable) event!130 Nonrecourse debt is just like regular debt.
Effect of Borrowing on Basis:
0 If you borrow nonrecourse to buy an asset, how much is your basis?
i. HYPO: TP borrows $100 nonrecourse to buy an asset for $100.
0.0 What is the basis in the asset?
0.0 $100, because the borrowed money is “yours” to create
basis in any thing you buy with it.
0.1 $0, because TP really put no money at stake (because the
only money TP put into the asset was the bank’s money
that TP doesn’t actually have to pay back, because TP can,
instead, just walk away from the property and have that
satisfy the debt).
Crane:131 TP has property worth $260,000 subject to a nonrecourse debt of
$260,000. TP inherited this package, held it for a while, taking $28,000 in
depreciation deductions, and then sold it (with the Mortgage attached) for
$2500 in cash.

Woodsam: TP argued that the act of NR borrowing is almost like selling. Therefore, the gain (inc. from $5 to
$100) should have been reported at taxed. The Court disagreed!
Crane:

Taxpayer IRS Magruder Concurrence


AR $2,500 $262,500 $2,500
($260,000 [discharge of the mortgage] + $2,500 [cash])
Basis $0 $232,000 ($28,000) negative basis
($260,000 [value of the mortgage] – $28,000 [deductions taken])
Gain/Loss $2,500 $30,500 $30,500
Comments The “problem” When you sell property subject to a nonrecourse mortgage, The borrowed
here is that the your amount realized includes any debt assumed by the nonrecourse money
TP’s buyer, even a nonrecourse mortgage (in part because probably should not
“explanation” we’ve seen in prior cases, such as Woodsam, that we are count to create basis,
does not explain not drawing distinctions between recourse and nonrecourse thus the TP was correct
how the mortgages). Thus, the AR on the sale of the property was that the basis was $0.
taxpayer could the $260K mortgage assumed plus the $2,500 in cash – thus
have taken as total of $262,500. However, to be consistent, this view However, to the extent
depreciation implies that when the property was acquired originally with the TP took depreciation
deductions the mortgage, that mortgage money counts towards the deductions they MUST
totaling $28,000 basis. Thus, the basis was initially $260K and then was reduce basis, so if we
if the basis had reduced by the depreciation deduction of $30,500 so that at start with a basis of $0
been zero at the the time of sale the basis was $232,500. (moreover, it and then reduce it by the
outset!! seems that the taxpayer implicitly accepts this view because $28,000 of depreciation
the TP was taking depreciation deductions). deductions taken, then
we must say that the TP
78
WNM
Tax 1 Ring (Fall 2017)

0.0 How should the taxpayer be taxed? The court agreed with the
IRS and took the view that when you sell property subject to a
nonrecourse mortgage, the amount realized includes any
cash/property received from the buyer, plus the face amount of
the nonrecourse mortgage on the property that is still
outstanding.
0.0 A corollary to this basic conclusion of the court was that
determination that when you borrow nonrecourse, that
borrowed money counts as “your” money and goes
towards the basis.
1 HYPO: TP purchases a $10M building for $1000 of his own money
and the remainder in borrowed proceeds. How should TP be taxed?
1.0 Outcome: TP’s basis in the building is $10M (and TP can use
that amount as his basis for calculating his depreciation!) It
does not matter whether the loan is nonrecourse.
Takeaways:
0 Initial borrowing to buy property is included in basis, regardless of
whether the loan is recourse or nonrecourse (basis is still purchase
price).
0.0 Subsequent borrowing against appreciated property is excluded from
basis of that property; the property’s basis does not suddenly increase.
(borrowing is not treated as a realization event)
1 Adjusted basis = original basis – depreciation deductions
Disposition of Mortgaged Property:
0 Tufts: Amount realized on the sale/disposition (including surrender to lender) of
property subject to a
nonrecourse mortgage includes: cash/property received PLUS the value of any
debt taken off your hands, even if the outstanding debt is greater than the value of
132
the property at that point in time.

The IRS’ view is consistent with the economic reality, as had a basis at the time of
demonstrated by the numbers above. sale of negative $28,000.
• Economic Reality/Cash flow: TP spent no money to get the property, and at the “end of the day,” disposed of it and
received $2,500; thus overall, owning this property was a net positive in the amount of $ 2500.
If the tax system is going to accurately track this economic reality, then when we consider all of the
different tax implications of TP having owned this property, it should reflect a net positive picture of
$2500.
• Overall tax impact: the taxpayer received the property, held it for several years and taking
depreciation deduction and then sold it:
Taxpayer IRS Magruder Concurrence
Amount Realized -$28,000 -$28,000 -$28,000
Basis $2,500 $30,500 $30,500
Gain/Loss -$25,500 $2,500 $2,500
• Clearly the taxpayer’s view of how the transactions should be taxed is not consistent with the
economic reality because if we followed the taxpayer’s treatment, including the view that there was
only $2500 of gain on the sale of the asset, then the tax system would, overall, be treating the
taxpayer as if owning the property had been a net LOSING experience in the amount of $25,500.
This is clearly NOT true; it was a positive experience in the amount of $2,500.
Tufts: Facts: TP borrows $100,000 nonrecourse to buy asset for $100,000. TP then takes depreciation deductions
of $60K; TP repays $10K of principal. Because of unfortunate market conditions, the FMV of the property is now
$70,000. TP has decided to surrender the property to the lender in satisfaction/cancellation of the note.
Nonrecourse Debt $100,000
Basis $100,000
Depreciation $60,000
Repaid Principal $10,000
FMV of property (now) $70,000
Nonrecourse Debt (now) $90,000
Basis (now) $40,000
How is the taxpayer treated on surrender of the property in satisfaction of the debt (a disposition event)?
0 Economic Reality/Cash flow: TP has out of pocket spent $10K (the amount of repaid principal)
– otherwise the TP has not parted with any of its own money. How much cash has the TP taken in
by virtue of being owner of the property $0. Thus, owning this property was economically at real
world loser in the amount of $10,000.
Overall tax impact:
TP IRS
Amount Realized $70,000 $90,000 ($100K loan - $10K repaid)
Basis $40,000 $40,000
79
WNM
Tax 1 Ring (Fall 2017)

CHOICE OF TAXPAYER
1. Who should be treated as a taxpayer? Who should be allowed to file a joint tax return?
2. History:
a. Pre-WWII, all taxpayers filed individual tax returns:
i. HYPO: Husband earns $10; wife earns $0. They argue that their income should be split up so
that each spouse earns $5 (preferable under the rate structure, because then both spouses
would be under a low income tax bracket).
$0-$5 20%
$5-$10 35%

ii. Poe v. Seaborne and Lucas v. Earl: popular cases from the era that attempted to make the
iii. above argument
1. Poe wins on the argument. Court found that since the couple lived in a community
property state, the money Poe earn became his wife’s as well under state law.
2. Lucas lost, because he and his spouse did not live in a community property state and
instead had entered into a contract that mimicked the law in a community property
agreement. Court ruled that TP can't use a contract to shift income for tax purposes.
Cannot assign away your income before it is yours in the eyes of the tax law. Must
report $10, then you can give away the after tax income to your wife.
b. Post-WWII, married couples were required to file joint tax returns:
133
i. Marriage bonus: If one spouse makes a lot more than the other (H: $0; W: $10)
ii. Marriage penalty: If both spouses have pretty much the same income (H: $5; W: $5)
1. Mapes: TP had to pay a marriage penalty because they were being taxed more than
they would if they could file as single people
a. Argued that the tax was unconstitutional because it hurt the woman's
earnings (seen as secondary to the husband's) – discourages secondary
income
b. Court said that since the Code doesn't have a distinction between primary
and secondary income based on gender – it isn't universally hurtful to every
married couple, just Mapes (not directed at women)
2. Boyter: Couple divorced at the end of each year so that they could file their returns
134
separately and be charged a lower tax rate.
a. Tax status is determined by martial status at the end of each year.
b. A divorce for tax purposes alone is a sham and will not be recognized. The
couple must file jointly.
3. Divorce:

Gain/Loss $30,000 $50,000


Previous depreciation deductions -$60,000 -$60,000
Gain on disposition $30,000 $50,000
Overall net tax effect -$30,000 -$10,000
• Holding: If the mortgage money counted as part of your basis costs (i.e. if the borrowed money was
used to create your basis in the asset, which it did in our case here) then when the property is
disposed of with the mortgage attached, that mortgage remaining must count in the amount realized.
Essentially if the debt counts in creating basis, it must count towards the amount realized when being
assumed regardless of whether at the moment of disposition the asset itself if worth less than the
outstanding mortgage. We can tell this is the right answer because if we follow the TP’s view, we get
an answer from the tax system that is NOT consistent with the economic reality.
Can’t meet these three goals at the same time. That’s why we have lingering marriage penalty problems:
• Progressive tax
• Equality in taxation between workers (married and unmarried)
• Preventing the marriage penalty/loss.
Marital status is determined on 12/31.
80
WNM
Tax 1 Ring (Fall 2017)

Property is transferred and alimony can be paid


0 §1041 Property:
0.0 If you transferred it to your spouse before §1041, it could be
considered an exchange
0.1 After §1041, transfers between spouses aren't taxable events, full
carryover of basis
1 §71(b) Alimony: Has to be cash, has to be received by spouse under
contract, contract can't say it's not includable/deductible, can't live in the
same household, and payment can't be made after the death of the
135
recipient.
1.0 Exception: §71(c) child support is not deductible for the payer
and not includable for the recipient
1.1 §215 If you pay alimony, it's deductible (above-the-line) -- if
you receive it, it's income
§151 personal exceptions for dependents: If you have a dependent, you get a
personal exemption under §151.
0 In a divorce setting, who gets to claim the child?
0.0 §152(e)(2) Custodial parent gets to under the tax law.
0.0 “Private ordering”: the custodial parent can agree to
“give” the deduction to the non-custodial parent.
0.0 Rationale: Non-custodial parent may be in a
higher tax bracket so the custodial parent may
give it away to be nice. Negotiate for more
alimony.

CAPITAL GAINS &


LOSSES
Why do we care about this separate category of assets?
0 For the individual, selling a capital asset at a gain might get you a good rate if you
hold the asset for more than a year
1 For the corporation, no good rate for corporations on capital gains
136
Loss restrictions
0 §1211: KLs must be used against KGs
0.0 Not good for individuals because they tend to have fewer KGs;
Corporations don’t care because they usually have both
1 If you still have losses after your KLs are used against your KGs
1.0 Individual: can use the loss against up to $3,000 of ordinary income
1.1 Corporation: excess losses can be taken back 3 years or forward 10 years
Rationales for having lower KG rates for individuals and disfavoring KLs for everyone
0 Lock-in Effect: doesn’t allow TP to diversify because if they do diversify they will
have to pay a tax. We don’t want people locked into their capital assets. We want
them moving in and out of different investment opportunities.
0.0 The rationale with the strongest foundation; however, if we were really
concern about this, there are better ways to address this problem.
0.1 Our regime isn’t really targeted towards this. The rules don’t require you to
reinvest. You can sell and go on a cruise. No reinvestment requirement or
time period.
1 Encourages investment
2 Counteracts inflation
3 Restrictions avoids having people cherry-pick which assets to sell and is
balanced out against the treatment of KGs
4 Bunching effect: a middle class TP sells off their primary investment asset, the
gain is lumped into one year, and the TP is taxed as if they are a high-income TP.
4.0 Imperially, we know that this is not what actually happens. Those who own
capital assets are extremely wealthy and they receive those gains year after
year.
4.1 If we were worried about bunching, there are other ways to tackle this issue
5 Historic effects of the Great Depression

Consistency by the two parties is the driver!


Not good! As TPs we don’t like this!
81
WNM
Tax 1 Ring (Fall 2017)

Capital gains rate (lower


Asset held for >1 year
tax)

Individual TP

Capital asset + sold at a Asset held for < 1 year Ordinary tax rate
gain

Corporate TP NO special rates

(1) Use KLs against KGs;


Individual TP (2) Use against up to $3,000 OI;
(3) Carry rest back/forward
Capital asset + sold at a
loss
(1) Use KLs against KGs;
Corporate TP
(2) Carry rest back/forward

Just some fun facts about capital assets:


Capital gains are profits from the sale of a capital asset, such as shares of stock, a business, a
parcel of land, or a work of art. Capital gains are generally included in taxable income, but in
most cases are taxed at a lower rate. o A capital gain is realized when a capital asset is
sold or exchanged at a price higher than its basis.
o A capital loss occurs when an asset is sold for less than its basis.
Capital gains/losses are not adjusted for inflation.
Capital losses may be used to offset capital gains, along with up to $3,000 of other
taxable income. The unused portion of a capital loss may be carried over to future
years.
The donee’s basis for a capital asset received as a gift equals the donor’s basis.
However, the basis of an inherited asset is "stepped up" to the value of the asset on the
date of the donor’s death. The step-up provision effectively exempts any gains on assets
held until death from income tax.
o Gains on an asset held until death may be passed on to heirs with the tax forgiven; if the
asset is then sold, the gain is sales price less market value at the time of death, a
treatment referred to as a “step-up in basis.”
C corporations pay the regular corporation tax rates on the full amount of their capital
gains and may use capital losses only to offset capital gains, not other kinds of income.
82

Das könnte Ihnen auch gefallen